solutions to exercises for physics for physiotherapy ... - solutions.pdf · solutionsto exercises...

139
SOLUTIONS to Exercises for Physics for Physiotherapy Technology 203-946-DW Fall 2019 by Chris Roderick, Ph.D. Created 2019 August 16th. Version α10, T E Xify-ed November 28, 2019. Continuous-update version

Upload: others

Post on 13-Mar-2020

10 views

Category:

Documents


0 download

TRANSCRIPT

Page 1: Solutions to Exercises for Physics for Physiotherapy ... - Solutions.pdf · SOLUTIONSto Exercises for Physics for Physiotherapy Technology 203-946-DW Fall 2019 by Chris Roderick,

SOLUTIONS to Exercises forPhysics for

PhysiotherapyTechnology

203-946-DW Fall 2019

by Chris Roderick, Ph.D.

Created 2019 August 16th.Version α10, TEXify-ed November 28, 2019.

Continuous-update version

Page 2: Solutions to Exercises for Physics for Physiotherapy ... - Solutions.pdf · SOLUTIONSto Exercises for Physics for Physiotherapy Technology 203-946-DW Fall 2019 by Chris Roderick,

Why practice?

Learning is about changing how you think. The most effective way to do that is to start by knowing how you thinknow. So always begin solving a problem by writing out what you think the answer will be like. Yes!, make a guess! Butafter that, do procede using the appropriate methods, and don’t skip steps. At the end, compare your result with yourinitial guess. Are they different? If yes, how do they differ? Are they different by a few percent? Or are they completelyopposite?

That will be the place to pause and reflect on how you are thinking about these situations, and figure out what youneed to change in your thinking. I’m here to help with that step, but it will go much faster if you contribute towardsidentifying where you need the help. Doing the exercises is the place where you work on that analysis.

So now, let’s get to work.

ii

Page 3: Solutions to Exercises for Physics for Physiotherapy ... - Solutions.pdf · SOLUTIONSto Exercises for Physics for Physiotherapy Technology 203-946-DW Fall 2019 by Chris Roderick,

Contents

0 Preparation 1

0.1 Units . . . . . . . . . . . . . . . . . . . . . . . . . . . . . . . . . . . . . . . . . . . . . . . . . . . . . . . . . . . . . 1

0.1.1 Lengths . . . . . . . . . . . . . . . . . . . . . . . . . . . . . . . . . . . . . . . . . . . . . . . . . . . . . . . 1

0.1.2 Areas & Volumes . . . . . . . . . . . . . . . . . . . . . . . . . . . . . . . . . . . . . . . . . . . . . . . . . . 1

0.1.3 Rates . . . . . . . . . . . . . . . . . . . . . . . . . . . . . . . . . . . . . . . . . . . . . . . . . . . . . . . . . 1

0.1.4 Masses & Forces . . . . . . . . . . . . . . . . . . . . . . . . . . . . . . . . . . . . . . . . . . . . . . . . . . 1

0.2 Vectors . . . . . . . . . . . . . . . . . . . . . . . . . . . . . . . . . . . . . . . . . . . . . . . . . . . . . . . . . . . . 2

0.2.1 Components, Magnitudes & Angles . . . . . . . . . . . . . . . . . . . . . . . . . . . . . . . . . . . . . . 2

0.2.2 Sums of Vectors . . . . . . . . . . . . . . . . . . . . . . . . . . . . . . . . . . . . . . . . . . . . . . . . . . 3

0.2.3 Vectors that Sum to Zero . . . . . . . . . . . . . . . . . . . . . . . . . . . . . . . . . . . . . . . . . . . . . 5

0.3 Logarithms . . . . . . . . . . . . . . . . . . . . . . . . . . . . . . . . . . . . . . . . . . . . . . . . . . . . . . . . . 7

0.3.1 Basics . . . . . . . . . . . . . . . . . . . . . . . . . . . . . . . . . . . . . . . . . . . . . . . . . . . . . . . . 7

0.3.2 Solving Equations with Logarithms . . . . . . . . . . . . . . . . . . . . . . . . . . . . . . . . . . . . . . 8

1 Forces 9

1.1 Free-Body Diagrams: Forces in Static Equilibrium . . . . . . . . . . . . . . . . . . . . . . . . . . . . . . . . . 9

1.1.1 Gravity and Tension . . . . . . . . . . . . . . . . . . . . . . . . . . . . . . . . . . . . . . . . . . . . . . . . 10

1.1.2 With an External Force . . . . . . . . . . . . . . . . . . . . . . . . . . . . . . . . . . . . . . . . . . . . . . 13

1.1.3 Gravity and Contact . . . . . . . . . . . . . . . . . . . . . . . . . . . . . . . . . . . . . . . . . . . . . . . . 16

1.1.4 Level Surfaces . . . . . . . . . . . . . . . . . . . . . . . . . . . . . . . . . . . . . . . . . . . . . . . . . . . 16

1.1.5 Inclined Surfaces . . . . . . . . . . . . . . . . . . . . . . . . . . . . . . . . . . . . . . . . . . . . . . . . . 18

1.1.6 More General Cases . . . . . . . . . . . . . . . . . . . . . . . . . . . . . . . . . . . . . . . . . . . . . . . . 21

1.1.7 Indeterminate Problems . . . . . . . . . . . . . . . . . . . . . . . . . . . . . . . . . . . . . . . . . . . . . 22

1.2 Solving problems using the Process . . . . . . . . . . . . . . . . . . . . . . . . . . . . . . . . . . . . . . . . . . . 24

1.3 Pulleys . . . . . . . . . . . . . . . . . . . . . . . . . . . . . . . . . . . . . . . . . . . . . . . . . . . . . . . . . . . . 43

1.3.1 A Single Pulley . . . . . . . . . . . . . . . . . . . . . . . . . . . . . . . . . . . . . . . . . . . . . . . . . . . 43

1.3.2 Two Pulleys . . . . . . . . . . . . . . . . . . . . . . . . . . . . . . . . . . . . . . . . . . . . . . . . . . . . . 44

1.3.3 Three or more Pulleys . . . . . . . . . . . . . . . . . . . . . . . . . . . . . . . . . . . . . . . . . . . . . . . 45

1.3.4 Two or more Ropes . . . . . . . . . . . . . . . . . . . . . . . . . . . . . . . . . . . . . . . . . . . . . . . . 45

2 Torques 47

2.1 Torque : Qualitative Exercises . . . . . . . . . . . . . . . . . . . . . . . . . . . . . . . . . . . . . . . . . . . . . . 47

2.1.1 Sign of Torque . . . . . . . . . . . . . . . . . . . . . . . . . . . . . . . . . . . . . . . . . . . . . . . . . . . 47

2.1.2 Line of Action . . . . . . . . . . . . . . . . . . . . . . . . . . . . . . . . . . . . . . . . . . . . . . . . . . . . 47

2.2 Torque : Quantitative Exercises . . . . . . . . . . . . . . . . . . . . . . . . . . . . . . . . . . . . . . . . . . . . . 48

iii

Page 4: Solutions to Exercises for Physics for Physiotherapy ... - Solutions.pdf · SOLUTIONSto Exercises for Physics for Physiotherapy Technology 203-946-DW Fall 2019 by Chris Roderick,

2.2.1 Single applied Force . . . . . . . . . . . . . . . . . . . . . . . . . . . . . . . . . . . . . . . . . . . . . . . . 48

2.3 Solving Problems using the Process . . . . . . . . . . . . . . . . . . . . . . . . . . . . . . . . . . . . . . . . . . . 52

2.3.1 Solving for Forces using Torque . . . . . . . . . . . . . . . . . . . . . . . . . . . . . . . . . . . . . . . . . 52

2.3.2 Biomechanical Equilibrium . . . . . . . . . . . . . . . . . . . . . . . . . . . . . . . . . . . . . . . . . . . 61

3 Materials 81

3.1 Stress . . . . . . . . . . . . . . . . . . . . . . . . . . . . . . . . . . . . . . . . . . . . . . . . . . . . . . . . . . . . . 81

3.2 Strain . . . . . . . . . . . . . . . . . . . . . . . . . . . . . . . . . . . . . . . . . . . . . . . . . . . . . . . . . . . . . 82

3.3 Stress-Strain Curves . . . . . . . . . . . . . . . . . . . . . . . . . . . . . . . . . . . . . . . . . . . . . . . . . . . 83

4 Energy 89

4.1 Thermal Energy . . . . . . . . . . . . . . . . . . . . . . . . . . . . . . . . . . . . . . . . . . . . . . . . . . . . . . 89

4.1.1 Energy Transferred . . . . . . . . . . . . . . . . . . . . . . . . . . . . . . . . . . . . . . . . . . . . . . . . 89

4.1.2 Temperature Change . . . . . . . . . . . . . . . . . . . . . . . . . . . . . . . . . . . . . . . . . . . . . . . 91

4.1.3 Temperature, initial or final . . . . . . . . . . . . . . . . . . . . . . . . . . . . . . . . . . . . . . . . . . . 92

4.1.4 Equilibrium Temperature . . . . . . . . . . . . . . . . . . . . . . . . . . . . . . . . . . . . . . . . . . . . 94

4.2 Kinetic Energy . . . . . . . . . . . . . . . . . . . . . . . . . . . . . . . . . . . . . . . . . . . . . . . . . . . . . . . 96

4.3 Work . . . . . . . . . . . . . . . . . . . . . . . . . . . . . . . . . . . . . . . . . . . . . . . . . . . . . . . . . . . . . 97

4.3.1 Problems . . . . . . . . . . . . . . . . . . . . . . . . . . . . . . . . . . . . . . . . . . . . . . . . . . . . . . 98

4.4 Power . . . . . . . . . . . . . . . . . . . . . . . . . . . . . . . . . . . . . . . . . . . . . . . . . . . . . . . . . . . . . 99

4.4.1 Power, Energy, and Price . . . . . . . . . . . . . . . . . . . . . . . . . . . . . . . . . . . . . . . . . . . . . 99

4.4.2 Problems . . . . . . . . . . . . . . . . . . . . . . . . . . . . . . . . . . . . . . . . . . . . . . . . . . . . . . 100

5 Waves 102

5.1 Periodic Waves . . . . . . . . . . . . . . . . . . . . . . . . . . . . . . . . . . . . . . . . . . . . . . . . . . . . . . . 102

5.1.1 Period & Frequency . . . . . . . . . . . . . . . . . . . . . . . . . . . . . . . . . . . . . . . . . . . . . . . . 102

5.1.2 The Fundamental Relationship for Periodic Waves . . . . . . . . . . . . . . . . . . . . . . . . . . . . . 104

5.1.3 Graphs of Waves . . . . . . . . . . . . . . . . . . . . . . . . . . . . . . . . . . . . . . . . . . . . . . . . . . 105

5.2 Sound . . . . . . . . . . . . . . . . . . . . . . . . . . . . . . . . . . . . . . . . . . . . . . . . . . . . . . . . . . . . . 114

5.2.1 Intensity, Power, and Area . . . . . . . . . . . . . . . . . . . . . . . . . . . . . . . . . . . . . . . . . . . . 114

5.2.2 Intensity, Energy, and Time . . . . . . . . . . . . . . . . . . . . . . . . . . . . . . . . . . . . . . . . . . . 116

5.2.3 Sound Level . . . . . . . . . . . . . . . . . . . . . . . . . . . . . . . . . . . . . . . . . . . . . . . . . . . . . 118

5.2.4 Sound Attenuation . . . . . . . . . . . . . . . . . . . . . . . . . . . . . . . . . . . . . . . . . . . . . . . . 120

6 Electricity 122

6.1 Electric Current, Voltage and Power . . . . . . . . . . . . . . . . . . . . . . . . . . . . . . . . . . . . . . . . . . 122

6.1.1 Current, Charge and Time . . . . . . . . . . . . . . . . . . . . . . . . . . . . . . . . . . . . . . . . . . . . 122

6.1.2 Voltage, Charge and Energy . . . . . . . . . . . . . . . . . . . . . . . . . . . . . . . . . . . . . . . . . . . 124

6.1.3 Electrical Power . . . . . . . . . . . . . . . . . . . . . . . . . . . . . . . . . . . . . . . . . . . . . . . . . . 125

6.1.4 Problems . . . . . . . . . . . . . . . . . . . . . . . . . . . . . . . . . . . . . . . . . . . . . . . . . . . . . . 126

6.2 Electric Circuits . . . . . . . . . . . . . . . . . . . . . . . . . . . . . . . . . . . . . . . . . . . . . . . . . . . . . . 127

6.2.1 Ohm’s Law & the Simple Circuit . . . . . . . . . . . . . . . . . . . . . . . . . . . . . . . . . . . . . . . . 127

6.2.2 Circuits with Resistors in Series . . . . . . . . . . . . . . . . . . . . . . . . . . . . . . . . . . . . . . . . 129

6.2.3 Circuits with Resistors in Parallel . . . . . . . . . . . . . . . . . . . . . . . . . . . . . . . . . . . . . . . 130

6.3 Time-Varying Currents . . . . . . . . . . . . . . . . . . . . . . . . . . . . . . . . . . . . . . . . . . . . . . . . . . 131

iv

Page 5: Solutions to Exercises for Physics for Physiotherapy ... - Solutions.pdf · SOLUTIONSto Exercises for Physics for Physiotherapy Technology 203-946-DW Fall 2019 by Chris Roderick,

6.3.1 Alternating Current . . . . . . . . . . . . . . . . . . . . . . . . . . . . . . . . . . . . . . . . . . . . . . . . 131

v

Page 6: Solutions to Exercises for Physics for Physiotherapy ... - Solutions.pdf · SOLUTIONSto Exercises for Physics for Physiotherapy Technology 203-946-DW Fall 2019 by Chris Roderick,

vi

Page 7: Solutions to Exercises for Physics for Physiotherapy ... - Solutions.pdf · SOLUTIONSto Exercises for Physics for Physiotherapy Technology 203-946-DW Fall 2019 by Chris Roderick,

Chapter 0

Preparation

0.1 Units

0.1.1 Lengths

EXR 0.1.01 1 m = 100 cm = 1 000 mmEXR 0.1.02 1 cm = 0.01 m = 10 mmEXR 0.1.03 3.205 m = 320.5 cm = 3 205 mmEXR 0.1.04 0.829 m = 82.9 cm = 829 mmEXR 0.1.05 7 cm = 0.07 m = 70 mmEXR 0.1.06 237 cm = 2.37 mEXR 0.1.07 15 mm = 1.5 cm = 0.015 mEXR 0.1.08 29.45 cm = 294.5 mm = 0.2945 m

EXR 0.1.09 14 in = 35.56 cmEXR 0.1.10 42 in = 3.5 ft = 1.067 mEXR 0.1.11 88 in = 2.235 mEXR 0.1.12 1 ft = 30.48 cmEXR 0.1.13 5’6” = 5 ft 6 in = 1.68 mEXR 0.1.14 25 cm = 9.8 inEXR 0.1.15 1.00 m = 3 ft and 3.37 inEXR 0.1.16 1.956 m = 6 ft and 5 in

0.1.2 Areas & Volumes

EXR 0.1.17 1 m2 = 10 000 cm2

EXR 0.1.18 400 cm2 = 0.0400 m2

EXR 0.1.19 0.0820 m2 = 820 cm2

EXR 0.1.20 37 cm2 = 3 700 mm2

EXR 0.1.21 1 in2 = 6.45 cm2 = 645 mm2

EXR 0.1.22 1 ft2 = 144 in2 = 929 cm2

EXR 0.1.23 200 ft2 = 18.58 m2

EXR 0.1.24 50 cm2 = 7.75 in2

EXR 0.1.25 7 L = 7 000 cm3 = 7 000 mLEXR 0.1.26 375 mL = 0.375 LEXR 0.1.27 0.520 L = 520 mLEXR 0.1.28 1 m3 = 1 000 000 cm3 = 1 000 LEXR 0.1.29 1 in3 = 16.39 cm3

EXR 0.1.30 1 ft3 = 28 317 cm3 = 28.3 LEXR 0.1.31 10 L = 0.353 ft3

EXR 0.1.32 625 mL = 38.1 in3

0.1.3 Rates

EXR 0.1.33 1 m/s = 60 m/min = 3 600 m/hEXR 0.1.34 1.28 m/s = 77 m/min = 4 620 m/hEXR 0.1.35 2.5 m/s = 150 m/min = 9 000 m/hEXR 0.1.36 90 km/h = 25 m/sEXR 0.1.37 100 km/h = 27.8 m/sEXR 0.1.38 10 m/s = 36 km/h

EXR 0.1.39 343 m/s = 1 235 km/hEXR 0.1.40 5.0 ft/s = 5.5 km/hEXR 0.1.41 50 mL/s = 3.0 L/minEXR 0.1.42 8.33 L/s = 0.500 m3/minEXR 0.1.43 2 L/min = 33.3 mL/sEXR 0.1.44 0.370 m3/min = 6.17 L/s

0.1.4 Masses & Forces

EXR 0.1.45 3.2 kg = 3 200 gEXR 0.1.46 487 g = 0.487 kgEXR 0.1.47 13 g = 0.013 kgEXR 0.1.48 18 lb = 8.2 kgEXR 0.1.49 145 lb = 65.8 kg

EXR 0.1.50 80 kg = 176 lbEXR 0.1.51 72 kg weighs 706 NEXR 0.1.52 8.56 kg weighs 84.0 NEXR 0.1.53 150 lb weighs 667 NEXR 0.1.54 169 lb weighs 750 N

Exercises for PPT α10 November 28, 2019 1 Ch.0 Preparation

Page 8: Solutions to Exercises for Physics for Physiotherapy ... - Solutions.pdf · SOLUTIONSto Exercises for Physics for Physiotherapy Technology 203-946-DW Fall 2019 by Chris Roderick,

0.2 Vectors

0.2.1 Components, Magnitudes & Angles

From Components to Magnitudes & Angles

For each of the vectors below find its components, then calculate its magnitude and the angle it makes with the +x-axis.(The angle measured counter-clockwise is positive.) In these exercises the grid size is 1 cm.

EXR 0.2.01

x

yAx =+5 cm (1)

A y =+4 cm (2)

A = 6.403 cm (3)

θ = 38.7 (4)

EXR 0.2.02

x

yAx =+5 cm (5)

A y =+5 cm (6)

A = 7.071 cm (7)

θ = 45.0 (8)

EXR 0.2.03

x

y Ax =−6 cm (9)

A y =+3 cm (10)

A = 6.708 cm (11)

θ = 153.4 (12)

EXR 0.2.04

xy

Ax =−4 cm (13)

A y =−3 cm (14)

A = 5.000 cm (15)

θ =−143.1 or 216.9 (16)

EXR 0.2.05

xy

Ax =+2 cm (17)

A y =−5 cm (18)

A = 5.385 cm (19)

θ =−68.2 or 291.8 (20)

EXR 0.2.06 Find the angle between this vector andthe +y-axis:

x

yAx =+5 cm (21)

A y =+4 cm (22)

A = 6.403 cm (23)

θ = 51.3 (24)

Angles Between Vectors

For each of the pairs of vectors below find the angle between them.

EXR 0.2.07

x

y

~A

~B Ax =+6 cm Bx =+2 cm (25)

A y =+3 cm By =+5 cm (26)

θA = 26.6 θB = 68.2 (27)

The angle between the vectors is θB −θA = 41.6.

EXR 0.2.08

Exercises for PPT α10 November 28, 2019 2 Ch.0 Preparation

Page 9: Solutions to Exercises for Physics for Physiotherapy ... - Solutions.pdf · SOLUTIONSto Exercises for Physics for Physiotherapy Technology 203-946-DW Fall 2019 by Chris Roderick,

x

y

~A

~B

Ax =+6 cm Bx =+2 cm (28)

A y =−4 cm By =+3 cm (29)

θA =−33.7 θB = 56.3 (30)

The angle between the vectors is θB −θA = 90.0.

EXR 0.2.09

x

y~A

~B

Ax =+2 cm Bx =−7 cm (31)

A y =+5 cm By =−2 cm (32)

θA = 68.2 θB = 195.9 (33)

The angle between the vectors is θB −θA = 127.7.

EXR 0.2.10

xy

~A

~B

Ax =−7 cm Bx =−2 cm (34)

A y =−2 cm By =−5 cm (35)

θA = 195.9 θB = 248.2 (36)

The angle between the vectors is θB −θA = 52.3.

From Magnitudes & Angles to Components

0.2.2 Sums of Vectors

In this set of exercises we will practice the summation of vectors. In each exercise you will need to find the componentsof the vectors involved. Be explicit and careful with the units of the quantities you are using and calculating.

Sum of Pairs of vectors

In the exercises below each vector is a position, and each grid square corresponds to one centimetre (1 cm) of distance.

For each of the pairs of vectors below calculate their sum. Find its components, its magnitude, and the angle itmakes with the +x-axis. (The give vectors can be called ~A and ~B. Call the sum ~C = ~A+~B.)

EXR 0.2.11

Exercises for PPT α10 November 28, 2019 3 Ch.0 Preparation

Page 10: Solutions to Exercises for Physics for Physiotherapy ... - Solutions.pdf · SOLUTIONSto Exercises for Physics for Physiotherapy Technology 203-946-DW Fall 2019 by Chris Roderick,

x

y~A

~B

~CAx =−5 cm Bx =+8 cm Cx =+3 cm (37)

A y =+6 cm By = 0 cm Cy =+6 cm (38)

The magnitude and direction of the vector ~C areC = 6.71 cm and θC = 63.4.

EXR 0.2.12

x

y

~A ~B

~CAx = 0 cm Bx =+5 cm Cx =+5 cm (39)

A y =+4 cm By =+4 cm Cy =+8 cm (40)

The magnitude and direction of the vector ~C areC = 9.43 cm and θC = 58.0.

EXR 0.2.13

x

y

~A~B

~CAx =+7 cm Bx =−4 cm Cx =+3 cm (41)

A y =+4 cm By =+4 cm Cy =+8 cm (42)

The magnitude and direction of the vector ~C areC = 8.54 cm and θC = 69.4.

EXR 0.2.14

x

y

~A

~B

~C

Ax =+3 cm Bx =−8 cm Cx =−5 cm (43)

A y =−7 cm By =+4 cm Cy =−3 cm (44)

The magnitude and direction of the vector ~C areC = 5.83 cm and θC =−149 (or θC = 211).

EXR 0.2.15

x

y ~A

~B

~C

Ax =+5 cm Bx =−2 cm Cx =+3 cm (45)

A y =+2 cm By =−8 cm Cy =−6 cm (46)

The magnitude and direction of the vector ~C areC = 6.71 cm and θC =−63.4 (or θC = 297).

EXR 0.2.16

x

y~A

~B

~C

Ax =+10 cm Bx =−4 cm Cx =+6 cm (47)

A y =+7 cm By =−6 cm Cy =+1 cm (48)

The magnitude and direction of the vector ~C areC = 6.08 cm and θC = 9.46.

Exercises for PPT α10 November 28, 2019 4 Ch.0 Preparation

Page 11: Solutions to Exercises for Physics for Physiotherapy ... - Solutions.pdf · SOLUTIONSto Exercises for Physics for Physiotherapy Technology 203-946-DW Fall 2019 by Chris Roderick,

Comparing Sums

In the exercises below each vector is a force, and each grid square corresponds to one newton (1 N) of force. For the setsof vectors below find the pair of vectors whose sum (the resultant) has the greatest magnitude.

EXR 0.2.17

x

y~A

x

y

~B

x

y

~C

With three vectors, there are three possible pairs.

The first pair gives a resultant ~F1 = ~A + ~B that has components F1,x = −1 N and F1,y = +7 N, and magnitudeF1 = 7.07 N.

The second pair gives a resultant ~F2 = ~B+ ~C that has components F2,x = −3 N and F2,y = −2 N, and magnitudeF2 = 3.61 N.

The last pair gives a resultant ~F3 = ~C + ~A that has components F3,x = +4 N and F3,y = +1 N, and magnitudeF3 = 4.12 N.

The pair with the resultant of greatest magnitude is ~A and ~B.

EXR 0.2.18

x

y ~A

x

y~B

x

y

~C

With three vectors, there are three possible pairs.

The first pair gives a resultant ~F1 = ~A+~B that has components F1,x = 0 N and F1,y =+8 N, and magnitude F1 = 8 N.

The second pair gives a resultant ~F2 = ~B+ ~C that has components F2,x = −5 N and F2,y = −7 N, and magnitudeF2 = 8.60 N.

The last pair gives a resultant ~F3 = ~C + ~A that has components F3,x = +3 N and F3,y = +7 N, and magnitudeF3 = 7.62 N.

The pair with the resultant of greatest magnitude is ~B and ~C. (Note how the resultant of the two largest vectorsdoes not have the largest magnitude.)

Sums that are Zero

In the exercises below each vector is a force, and each grid square corresponds to one newton (1 N) of force. For the setsof vectors below find the missing vector that would make their sum equal~0 N. (Remember that the vector~0 N is thevector whose components are each 0N.)

0.2.3 Vectors that Sum to Zero

In the exercises below each vector is a force, and each grid square corresponds to one newton (1 N) of force. For thesets of vectors below find the missing vector that would make their sum equal~0 N. (Remember that the vector~0 N isthe vector whose components are each 0N.) In the case of two vectors being given (which we can call ~A and ~B) find thethird vector ~C such that ~A+~B+ ~C =~0.

Exercises for PPT α10 November 28, 2019 5 Ch.0 Preparation

Page 12: Solutions to Exercises for Physics for Physiotherapy ... - Solutions.pdf · SOLUTIONSto Exercises for Physics for Physiotherapy Technology 203-946-DW Fall 2019 by Chris Roderick,

EXR 0.2.19

x

y

~A

~B ~C

Given that Ax =−2 cm, A y =−7 cm,Bx =−5 cm, and By =+3 cm,we can solve the vector equation~A+~B+ ~C =~0 −→ ~C =−(~A+~B)by its components

Cx =−(Ax +Bx) (49)

=−((−2 cm)+ (−5 cm)

)=+7 cm (50)

Cy =−(A y +By) (51)

=−((−7 cm)+ (+3 cm)

)=+4 cm (52)

EXR 0.2.20

x

y

~A~B

~C

Given that Ax =+3 cm, A y =+2 cm,Bx =−2 cm, and By =+3 cm,we can solve the vector equation~A+~B+ ~C =~0 −→ ~C =−(~A+~B)by its components

Cx =−(Ax +Bx) (53)

=−((+3 cm)+ (−2 cm)

)=−1 cm (54)

Cy =−(A y +By) (55)

=−((+2 cm)+ (+3 cm)

)=−5 cm (56)

EXR 0.2.21

x

y

~A

~B~C

Given that Ax =+4 cm, A y =−4 cm,Bx =+1 cm, and By =+2 cm,we can solve the vector equation~A+~B+ ~C =~0 −→ ~C =−(~A+~B)by its components

Cx =−(Ax +Bx) (57)

=−((+4 cm)+ (+1 cm)

)=−5 cm (58)

Cy =−(A y +By) (59)

=−((−4 cm)+ (+2 cm)

)=+2 cm (60)

Exercises for PPT α10 November 28, 2019 6 Ch.0 Preparation

Page 13: Solutions to Exercises for Physics for Physiotherapy ... - Solutions.pdf · SOLUTIONSto Exercises for Physics for Physiotherapy Technology 203-946-DW Fall 2019 by Chris Roderick,

EXR 0.2.22

x

y

~A~B

~C

Given that Ax =+2 cm, A y =+3 cm,Bx =−3 cm, and By =+1 cm,we can solve the vector equation~A+~B+ ~C =~0 −→ ~C =−(~A+~B)by its components

Cx =−(Ax +Bx) (61)

=−((+2 cm)+ (−3 cm)

)=+1 cm (62)

Cy =−(A y +By) (63)

=−((+3 cm)+ (+1 cm)

)=−4 cm (64)

EXR 0.2.23

x

y

~A

~B

~C

Given that Ax =+5 cm, A y =+2 cm,Bx =+1 cm, and By =+4 cm,we can solve the vector equation~A+~B+ ~C =~0 −→ ~C =−(~A+~B)by its components

Cx =−(Ax +Bx) (65)

=−((+5 cm)+ (+1 cm)

)=−6 cm (66)

Cy =−(A y +By) (67)

=−((+2 cm)+ (+4 cm)

)=−6 cm (68)

EXR 0.2.24

x

y

~A

~B

~C

Given that Ax =−2 cm, A y =−5 cm,Bx =−4 cm, and By =+3 cm,we can solve the vector equation~A+~B+ ~C =~0 −→ ~C =−(~A+~B)by its components

Cx =−(Ax +Bx) (69)

=−((−2 cm)+ (−4 cm)

)=+6 cm (70)

Cy =−(A y +By) (71)

=−((−5 cm)+ (+3 cm)

)=+2 cm (72)

0.3 Logarithms

The square-root function tells you “if x =py, then y= x2.” In a similar way the logarithm function tells you “if x = log(y),then y = 10x.” The logarithm (or just “log” for short) has some algebraic properties. You know that for the square-rootp

a×b =pa×p

b. For the log there are the rules loga×b = log(a)+ log(b) and log cn = n× log c. The exercises in thissection practice these properties.

0.3.1 Basics

EXR 0.3.01 log(100) = 2EXR 0.3.02 log(1000) = 3EXR 0.3.03 log(500) = 2.70EXR 0.3.04 log(50) = 1.70EXR 0.3.05 log(5) = 0.7EXR 0.3.06 log(1000000) = 6EXR 0.3.07 log(10000000) = 7

EXR 0.3.08 log(3000000) = 6.477EXR 0.3.09 log(9.900×105) = 5.996EXR 0.3.10 log(100×1000) = 5EXR 0.3.11 log(100)+ log(1000) = 5EXR 0.3.12 log(15) = 1.176EXR 0.3.13 log(3)+ log(5) = 1.176EXR 0.3.14 log(1000

/10) = 2

Exercises for PPT α10 November 28, 2019 7 Ch.0 Preparation

Page 14: Solutions to Exercises for Physics for Physiotherapy ... - Solutions.pdf · SOLUTIONSto Exercises for Physics for Physiotherapy Technology 203-946-DW Fall 2019 by Chris Roderick,

EXR 0.3.15 log(1000)− log(10) = 2EXR 0.3.16 log(15) = 1.176EXR 0.3.17 log(30)− log(2) = 1.176EXR 0.3.18 log(7

/3) = 0.368

EXR 0.3.19 log(7)− log(3) = 0.368EXR 0.3.20 log(3

/7) = −0.368

EXR 0.3.21 log(137/

137) = 0EXR 0.3.22 log( 1

100 ) = −2

EXR 0.3.23 log(5) = 0.7EXR 0.3.24 log(25) = 1.4EXR 0.3.25 2× log(5) = 1.4EXR 0.3.26 log(2) = 0.3EXR 0.3.27 log(8) = 0.9EXR 0.3.28 3× log(2) = 0.9EXR 0.3.29 log(

p10) = 1/2

EXR 0.3.30 log(√p

10) = 1/4

0.3.2 Solving Equations with Logarithms

One last note: Since 10x > 0 for any value of x, there is no “±” when we use the logarithm to solve an equation. Also forthat reason expressions like log(−7) have no meaning (just like

p−7 is not a number).

EXERCISE 0.3.31

Exercises for PPT α10 November 28, 2019 8 Ch.0 Preparation

Page 15: Solutions to Exercises for Physics for Physiotherapy ... - Solutions.pdf · SOLUTIONSto Exercises for Physics for Physiotherapy Technology 203-946-DW Fall 2019 by Chris Roderick,

Chapter 1

Forces1.1 Free-Body Diagrams: Forces in Static Equilibrium

In these exercises we will practice the construction and use of Free-Body Diagrams (FBDs) to reason about the forcesacting on objects in static equilibrium. For the purposes of practice we will limit ourselves to two-dimensional systems.Unless explicitly stated each system is being viewed from the side so that gravity acts straight downwards on thediagram.

In each exercise construct the Free-Body Diagram (FBD). A recommended first step is to make a list of the thingsthat the object is interacting with. Name those things, beginning with the Earth, and then name the other objects orsurfaces which the object is touching (like the floor) or is attached to (like a rope). Look carefully at the diagram (ifgiven), and read carefully any descriptive text of the situation (if given), during this step.

After constructing the FBD, check that the forces do sum to zero. This check is a qualitative diagram used toconfirm that equilibrium is possible. If it is not possible to sum the forces in your FBD by adjusting them, then returnto your list of interactions, and think more carefully about the situation. (There is no point to move past this step todoing quantitative calculations if the required outcome is impossible!)

Remember to be clear about what aspect of each force is unknown. If there is a rope at a specified angle, then theforce of tension that it exerts on the object is along that fixed direction, and only its magnitude may be adjusted. Ifthere is a surface of contact, then the normal at that surface points perpendicular away from the surface, and only itsmagnitude may be adjusted. Similarly, if there is friction, then it must be parallel to the surface, but its direction andmagnitude must both be determined.

Above all, do not hesitate to iterate. Producing the “correct” diagram immediately is not a healthy expectation tohold. You are solving a problem and you must be open to exploring alternatives before arriving at a consistent answer.Iteration is a key feature of problem-solving!

Exercises for PPT α10 November 28, 2019 9 Ch.1 Forces

Page 16: Solutions to Exercises for Physics for Physiotherapy ... - Solutions.pdf · SOLUTIONSto Exercises for Physics for Physiotherapy Technology 203-946-DW Fall 2019 by Chris Roderick,

1.1.1 Gravity and Tension

In these first few Free-Body Diagram (FBD) exercises the object is suspended against gravity by ropes. In each exercise,draw the FBD, and use the sum of forces to qualitatively estimate the magnitudes of the tensions (relative to your choiceof the weight of the object).

EXERCISE 1.1.01

~FG

~T

~FG

~T

Physically: The tension must balance the weight, and so has an equal magnitude, as is expected.

EXERCISE 1.1.02

~FG

~TR~TL

~FG

~TR

~TL

Physically: The magnitude of the tension in each rope is slightly greater than half the weight because they are pullinghorizontally against each other in addition to supporting half the weight.

EXERCISE 1.1.03

~FG

~TR

~TL

~FG

~TR

~TL The tension on the rightshould be weaker, andthe tension on the leftslightly stronger, relativeto the weight. ~FG

~TR

~TL

Physically: The rope on the left, that is more vertical, supports more of the weight and thus has a greater magnitudeof tension than the rope on the right.

EXERCISE 1.1.04

~FG

~TR

~TL

~FG

~TR

~TLThe lengths of the ropesare different, but theforces are the same asthe previous exercise.

~FG~TR

~TL

Physically: The length of a rope and the tension in the rope are independent of each other. The angles made by theropes are the same as the previous exercise, and so the tensions are the same as in the previous exercise.

EXERCISE 1.1.05

~FG

~TR

~TL

~FG ~TR

~TL The tension on the leftshould be stronger, rela-tive to the weight.

~FG ~TR

~TL

Physically: The tension in the horizontal rope is independent of the object’s weight. The rope on the left must supportthe weight and oppose the rope on the right, and so is greater in magnitude than either.

Exercises for PPT α10 November 28, 2019 10 Ch.1 Forces

Page 17: Solutions to Exercises for Physics for Physiotherapy ... - Solutions.pdf · SOLUTIONSto Exercises for Physics for Physiotherapy Technology 203-946-DW Fall 2019 by Chris Roderick,

EXERCISE 1.1.06

~FG

~TR

~TL

~FG

~TR

~TLThe tension on the rightshould be slightly weaker,and the tension on theleft stronger, relative tothe weight. ~FG

~TR

~TL

Physically: The rope above and towards the left must support the weight and counter the tension in the rope belowand towards the right. The weight is only pulled away from the vertical by a small angle, so the tension on the rightdoes not need to be large.

EXERCISE 1.1.07

~FG

~TR~TL

~FG

~TR

~TL

EXERCISE 1.1.08

~FG

~TR~TL

~FG

~TR

~TLThe tensions should beslightly stronger, relativeto the weight.

~FG

~TR

~TL

EXERCISE 1.1.09

~FG

~TR~TL

~FG ~TR

~TL

Both tensions should bemuch stronger, relative tothe weight.

Physically: The vertical components of the tensions in the ropes must balancethe weight. Since they are both so close to being horizontal they must both belarge in magnitude to achieve the required vertical balance.

~FG

~TR

~TL

EXERCISE 1.1.10

~FG

~TR~TL

~FG

~TR

~TL The tension on the leftshould be slightly weaker,and the tension on theright stronger, relative tothe weight. ~FG

~TR

~TL

Exercises for PPT α10 November 28, 2019 11 Ch.1 Forces

Page 18: Solutions to Exercises for Physics for Physiotherapy ... - Solutions.pdf · SOLUTIONSto Exercises for Physics for Physiotherapy Technology 203-946-DW Fall 2019 by Chris Roderick,

Physically: The rope on the right, that is more vertical, supports more of the weight and thus has a greater magnitudeof tension than the rope on the left.

EXERCISE 1.1.11

~FG

~TR

~TL

~FG~TR

~TLThe tension on the rightshould be much weaker,and the tension on theleft stronger, relative tothe weight. ~FG ~TR

~TL

Physically: The rope above must support the weight and counter the tension in the rope towards the right. The weightis only pulled away from the vertical by very a small angle, so the tension on the right does not need to be large.

EXERCISE 1.1.12

~FG

~TR

~TL

~FG

~TR

~TL

The tension on theright should be slightlystronger, and the tensionon the left much stronger,relative to the weight.

~FG

~TR

~TL

Physically: The rope above must support the weight and counter the tension in the rope towards the right. The ropesare almost straight, aligned with each other, so the tensions must be larger than the weight.

Exercises for PPT α10 November 28, 2019 12 Ch.1 Forces

Page 19: Solutions to Exercises for Physics for Physiotherapy ... - Solutions.pdf · SOLUTIONSto Exercises for Physics for Physiotherapy Technology 203-946-DW Fall 2019 by Chris Roderick,

1.1.2 With an External Force

In these cases an external force ~P (a push or a pull) is being applied to an object suspended by ropes. In each of thesecases the applied force never causes the tension of any rope to become zero.

EXERCISE 1.1.13

~FG

~P

~T

~FG

~P

~T

Physically: The tension must balance the weight and the applied push. Since the push is upwards, this reduces therequired tension.

EXERCISE 1.1.14

~FG

~P

~T

~FG

~P

~T

Physically: The tension must balance the weight and the applied pull. Since the pull is downwards, this increases therequired tension.

EXERCISE 1.1.15

~FG

~P

~T

~FG ~P

~T The tension int herope should be slightlystronger, relative to theweight. ~FG ~P

~T

EXERCISE 1.1.16

~FG

~T~P

~FG

~T

~P We have over-estimated thestrength of the push relativeto the weight. The tensionshould be stronger, relative tothe weight. ~FG

~T

~P

EXERCISE 1.1.17

~FG

~P~T

~FG~P

~TThe applied push andthe tension should bestronger, relative to theweight.

~FG

~P

~T

EXERCISE 1.1.18

Exercises for PPT α10 November 28, 2019 13 Ch.1 Forces

Page 20: Solutions to Exercises for Physics for Physiotherapy ... - Solutions.pdf · SOLUTIONSto Exercises for Physics for Physiotherapy Technology 203-946-DW Fall 2019 by Chris Roderick,

~FG

~P

~T

~FG

~P

~T We have over-estimated the ten-sion relative to the weight. Thepush should be stronger, rela-tive to the weight.

~FG

~P~T

Physically: The applied push is almost vertical, and so supports most of the weight. The tension does not need to bevery large to keep the object in static equilibrium.

EXERCISE 1.1.19

~FG

~P

~T

~FG

~P

~TBoth the tension and thepush should be muchstronger, relative to theweight.

~FG

~P

~T

Physically: Since the applied force is not horizontal it must be large to hold the object away from the vertical. Theapplied force, almost parallel to the rope, increases the tension significantly.

EXERCISE 1.1.20

~FG

~P

~TR~TL

~FG ~P

~TR

~TLBoth the tension on the rightand the push should be weaker,and the tension on the leftshould be slightly stronger, rela-tive to the weight.

~FG ~P

~TR

~TL

Physically: Since the applied force is horizontal the rope that can oppose it (the rope on the left) must have a highertension. This also decreases the tension in the rope on the right.

EXERCISE 1.1.21

~FG~P

~TR~TL

~FG

~P

~TR

~TL Both the tension on the rightand the push should be weaker,and the tension on the leftshould be slightly stronger, rela-tive to the weight. ~FG

~P

~TR

~TL

Physically: Since the applied force is almost parallel to the rope on the left that rope must have an increased tension.This also decreases the tension in the rope on the right.

EXERCISE 1.1.22

~FG

~P

~TR~TL

~FG

~P

~TR

~TL

Both the tension on the rightand the push should be muchweaker, and the tension on theleft should be stronger, relativeto the weight.

~FG~P~TR

~TL

Physically: Since the applied force is perpendicular to the rope on the left the tension in that rope will not be changedrelative to the situation without the push. Since the push is upwards and towards the right it supports some of theweight, and reduces the tension on the right.

Exercises for PPT α10 November 28, 2019 14 Ch.1 Forces

Page 21: Solutions to Exercises for Physics for Physiotherapy ... - Solutions.pdf · SOLUTIONSto Exercises for Physics for Physiotherapy Technology 203-946-DW Fall 2019 by Chris Roderick,

EXERCISE 1.1.23

~FG

~P

~TR~TL

~FG

~P

~TR

~TL

Both tensions should bestronger, relative to the weight.

~FG

~P

~TR

~TL

Physically: The applied force contributes downwards, with gravity. This increases the tension in both ropes, symmet-rically.

EXERCISE 1.1.24

~FG

~TR~P~TL

~FG

~TR

~P

~TL

Both tensions should be weaker,relative to the weight.

~FG

~TR

~P

~TL

Physically: Since the applied force supports a part of the weight the tension in both ropes is decreased.

EXERCISE 1.1.25

~FG ~P

~TR~TL

~FG

~P

~TR

~TLBoth the tension on the leftshould be stronger, relative tothe weight.

~FG

~P

~TR

~TL

Physically: Since the applied force is parallel to the rope on the left that rope must have an increased tension. Thisdoes not change the tension in the rope on the right.

EXERCISE 1.1.26

~FG

~TR~P

~TL

~FG

~TR

~P

~TL

Both the tension on the left andthe push should be weaker, rela-tive to the weight.

~FG

~TR

~P

~TL

Physically: Since the applied force is parallel to the rope on the left that rope must have a decreased tension. This doesnot change the tension in the rope on the right.

Exercises for PPT α10 November 28, 2019 15 Ch.1 Forces

Page 22: Solutions to Exercises for Physics for Physiotherapy ... - Solutions.pdf · SOLUTIONSto Exercises for Physics for Physiotherapy Technology 203-946-DW Fall 2019 by Chris Roderick,

1.1.3 Gravity and Contact

In this section we will practice analyzing objects in contact with a surface. Friction is present between all surfaces ofcontact. The exceptions of there being negligible friction will be noted as µ= 0.

1.1.4 Level Surfaces

In these situations the force of gravity and the normal will point opposite each other, but, because of other forcespresent, they may not be of equal magnitude. Careful!

EXERCISE 1.1.27

~FG

~n

~FG

~n

Physically: The normal must balance the weight, and so has an equal magnitude, as is expected.

EXERCISE 1.1.28

~FG

~P

~n

~FG

~P

~n

Physically: The normal must balance the weight and the applied push. Since the push is downwards, this increasesthe required normal.

EXERCISE 1.1.29

~FG

~P

~n

~f

~FG ~P

~n~f

Physically: Friction opposes the push.

EXERCISE 1.1.30

~FG

~P

~n

~f~FG

~P

~n~f

The normal shouldbe larger, relative tothe weight. The fric-tion should be slightlysmaller, relative to thepush.

~FG

~P

~n~f

Physically: The normal is larger than the weight since it must also oppose the vertical component of the applied push.Friction opposes the horizontal component of the applied push.

EXERCISE 1.1.31

Exercises for PPT α10 November 28, 2019 16 Ch.1 Forces

Page 23: Solutions to Exercises for Physics for Physiotherapy ... - Solutions.pdf · SOLUTIONSto Exercises for Physics for Physiotherapy Technology 203-946-DW Fall 2019 by Chris Roderick,

~FG

~T

~n

~f

~FG ~T

~n~f

Physically: Friction opposes the pull applied by the tension in the rope.

EXERCISE 1.1.32

~FG

~T

~n

~f

~FG~T

~n~f

The normal shouldbe smaller, relative tothe weight. The fric-tion should be slightlysmaller, relative to thetension.

~FG~T

~n~f

Physically: The normal is smaller than the weight since the vertical component of the applied tension supports aportion of the weight. Friction opposes the horizontal component of the tension.

EXERCISE 1.1.33

~FG

~T

~n

~FG~T

~n

Physically: The normal must balance the weight and the applied tension. Since the tension is upwards, this decreasesthe required normal.

EXERCISE 1.1.34 An object is attached to the surface by a rope. There is tension in the rope.

T 6= 0 N

~FG

~T

~n

~f~FG

~T

~n~f

The normal shouldbe larger, relative tothe weight. The fric-tion should be slightlysmaller, relative to thetension.

~FG

~T

~n~f

Physically: The normal is larger than the weight since it must also oppose the vertical component of the tension thatis trying to pull the object towards the surface. Friction opposes the horizontal component of the tension that is tryingto pull the object the towards the right.

Exercises for PPT α10 November 28, 2019 17 Ch.1 Forces

Page 24: Solutions to Exercises for Physics for Physiotherapy ... - Solutions.pdf · SOLUTIONSto Exercises for Physics for Physiotherapy Technology 203-946-DW Fall 2019 by Chris Roderick,

1.1.5 Inclined Surfaces

In these exercises friction remains strong enough to keep the object in equilibrium on the surface. The normal is still(as always) perpendicular to the surface, but with the surface not horizontal the normal will not point opposite gravity,and will almost certainly not have the same magnitude. Be very careful finding the normal!

EXERCISE 1.1.35

~FG

~f

~n

~FG

~f

~n We have over-estimated thestrength of the push relativeto the weight. The tensionshould be stronger, relative tothe weight. ~FG

~f

~n

Physically: The normal prevents the object from moving through the surface, but it is of the wrong magnitude anddirection to counter gravity. The force of friction completes the sum to zero, preventing the object from moving acrossthe surface.

EXERCISE 1.1.36 The applied force is small in comparison to all other forces.

~FG

~P~f

~n

~FG~P

~f

~nWe have over-estimated thestrength of friction, and the nor-mal should be stronger, relativeto the weight.

~FG~P

~f

~n

Physically: The push is insufficient to counter the component of gravity parallel to the surface. Thus friction must stillpoint upwards along the surface.

EXERCISE 1.1.37 The applied force is large, and the object almost starts sliding UP the incline.

~FG

~P

~n

~f

~FG

~P

~n~f

We have under-estimated thestrength of the push and thenormal, relative to the weight.

~FG

~P

~n~f

Physically: We are told that the object is almost about to begin sliding up the incline. The push upwards along theincline is more than required to support the component of the weight parallel to the surface, so friction must oppose it.This means that friction must point down the incline to oppose that incipient motion.

EXERCISE 1.1.38 The applied force is small in comparison to all other forces.

~FG

~P

~f

~n

~FG ~P

~f

~n We have over-estimated thestrength of friction, and the nor-mal should be stronger, relativeto the weight.

~FG ~P~f

~n

Physically: The push, being into the surface and upwards along the incline, decreases friction and increases the normal.

EXERCISE 1.1.39 The applied force is large, and the object almost starts sliding UP the incline.

Exercises for PPT α10 November 28, 2019 18 Ch.1 Forces

Page 25: Solutions to Exercises for Physics for Physiotherapy ... - Solutions.pdf · SOLUTIONSto Exercises for Physics for Physiotherapy Technology 203-946-DW Fall 2019 by Chris Roderick,

~FG

~P

~n

~f~FG ~P

~n~f

We have under-estimated thestrength of the push and thenormal, and over-estimated thestrength of friction, relative tothe weight. ~FG ~P

~n~f

Physically: The push, being strongly into the surface and upwards along the incline, reverses the direction of friction,and increases the normal significantly.

EXERCISE 1.1.40

µ= 0

~FG

~P

~n

~FG~P

~n

Physically: The normal prevents the object from moving through the surface, but it is of the wrong magnitude anddirection to counter gravity. The applied push, in the place of friction, completes the sum to zero, preventing the objectfrom moving across the surface.

EXERCISE 1.1.41

µ= 0

~FG

~P

~n

~FG ~P

~n

Physically: The applied push, directed into and upwards along the incline, increases the normal and balances thecomponent of gravity parallel to the surface.

EXERCISE 1.1.42

µ= 0

~FG

~T

~n

~FG~T

~n

Physically: The applied tension replaces friction as the balancing force.

EXERCISE 1.1.43

µ= 0

~FG

~T~n

~FG

~T

~n

Exercises for PPT α10 November 28, 2019 19 Ch.1 Forces

Page 26: Solutions to Exercises for Physics for Physiotherapy ... - Solutions.pdf · SOLUTIONSto Exercises for Physics for Physiotherapy Technology 203-946-DW Fall 2019 by Chris Roderick,

EXERCISE 1.1.44

~FG

~P

~f

~n

~FG

~P

~f

~n The normal should bestronger, relative to theweight.

~FG

~P

~f

~n

Physically: The push, directed perpendicular to the surface, only increases the strength of the normal required to stopthe object from moving through the surface.

EXERCISE 1.1.45

~FG

~f

~n

~P~FG

~f

~n

~P

We have under-estimated thestrength of friction and the nor-mal, relative to the weight.

~FG

~f

~n

~P

Physically: The push, being into the surface and downwards along the incline, increases both friction and the normal.

EXERCISE 1.1.46

~FG

~f

~n

~P

~FG

~f

~n

~P We have under-estimated thestrength of friction, relative tothe push.

~FG

~f

~n~P

Physically: The push, being downwards along the incline, increases the friction.

Exercises for PPT α10 November 28, 2019 20 Ch.1 Forces

Page 27: Solutions to Exercises for Physics for Physiotherapy ... - Solutions.pdf · SOLUTIONSto Exercises for Physics for Physiotherapy Technology 203-946-DW Fall 2019 by Chris Roderick,

1.1.6 More General Cases

Cases of vertical and inverted surface.Spherical objects on surfaces.Including two-surface contacts.

EXERCISE 1.1.47µ= 0

µ> 0

Involving Springs

Exercises for PPT α10 November 28, 2019 21 Ch.1 Forces

Page 28: Solutions to Exercises for Physics for Physiotherapy ... - Solutions.pdf · SOLUTIONSto Exercises for Physics for Physiotherapy Technology 203-946-DW Fall 2019 by Chris Roderick,

1.1.7 Indeterminate Problems

It is possible for there to be more unknown forces than there are are equations. These cases are called indeterminate.The possible solutions are subdivided into cases categorized by assumed values for one (or more) of the unknowns.

In these exercises there is friction between the ropes and any surfaces that they lay across. This means that, inany cases where a segment of rope lays on a surface, the tension may vary along that length of the rope!

EXERCISE 1.1.48

(Above) Solve the system for the unknown tensions in these three cases:(1) The tension in the horizontal rope is zero.(2) The tension in the rope that points upwards to the right is zero.(3) The tension in the rope that points upwards to the left equals the object’s weight mg.

EXERCISE 1.1.49

EXERCISE 1.1.50

EXERCISE 1.1.51

EXERCISE 1.1.52

EXERCISE 1.1.53

Exercises for PPT α10 November 28, 2019 22 Ch.1 Forces

Page 29: Solutions to Exercises for Physics for Physiotherapy ... - Solutions.pdf · SOLUTIONSto Exercises for Physics for Physiotherapy Technology 203-946-DW Fall 2019 by Chris Roderick,

Exercises for PPT α10 November 28, 2019 23 Ch.1 Forces

Page 30: Solutions to Exercises for Physics for Physiotherapy ... - Solutions.pdf · SOLUTIONSto Exercises for Physics for Physiotherapy Technology 203-946-DW Fall 2019 by Chris Roderick,

1.2 Solving problems using the Process

The Process

0. Identify the Object!

1. Identify the forces acting on the Object.

2. Draw the Free-Body Diagram.

3. Separately, for each force acting on the Object:

• draw the coordinates• draw the force (vector)• determine the components.

4. Use Newton’s 1st Law to write the equations to be solved.

5. Solve the equations for the unknown quantities.

Remember to take your time and to work carefully through Steps 0, 1 and 2 – that’s where you are doing thephysics! Working on these problems is not a race. If you rush through (or skip) those steps, then you will risk gettingit completely wrong. Working on these problems is where you should practice working methodically through each stepof the Process.

Exercises for PPT α10 November 28, 2019 24 Ch.1 Forces

Page 31: Solutions to Exercises for Physics for Physiotherapy ... - Solutions.pdf · SOLUTIONSto Exercises for Physics for Physiotherapy Technology 203-946-DW Fall 2019 by Chris Roderick,

PROBLEM 1.2.01: On the force table a 250 gram mass is hanging at 72 and a 300 gram mass is hanging at 345.What mass must we hang (and at what direction) to keep the ring at the center in static equilibrium? (The weight ofthe ring may be ignored since it is much smaller than the tension in the strings will be.)

Solution: We are asked to find a mass and a direction. From our experience in the lab with the force table we canexpect that the mass will be something between 200 grams and 600 grams. Also – thinking physically – with the firsttwo strings pulling towards the right and slightly upwards we expect the third string should point towards the left andslightly downwards.

Step 0: The object is the ring at the center of the table.

Step 1: The ring is attached to three separate strings, each of which will exert a tension on it. While the ring doeshave mass, and the gravitational force is not actually zero, its weight will be so much smaller in magnitude than thetensions acting on it that we can neglect the effect of its weight.

Step 2: The Free-Body Diagram and check of sum of forces:

~T1

~T2~T3

~T1

~T2

~T3

From these qualitative constructions we can see that the third string should point towards the left and slightly down-wards, into the third quadrant.

Step 3: The components of the known and unknown forces:

x

y~T1

72

T1,x = (0.250 kg)(9.81 N/kg) cos72 =+0.758 N

T1,y = (0.250 kg)(9.81 N/kg) sin72 =+2.332 N

x

y

~T2

345

T2,x = (0.300 kg)(9.81 N/kg) cos345 =+2.843 N

T2,y = (0.300 kg)(9.81 N/kg) sin345 =−0.762 N

The third tension is completely unknown. We will solve for the components, from which we can get the magnitude anddirection. From our FBD and check-of-sum of forces we expect that this force will point into the third quadrant.

Step 4: Newton’s 1st Law applies to the ring at the center of the table:∑~F =~0 N (1.1)

In this specific situation the sum of forces is~T1 +~T2 +~T3 =~0 N (1.2)

The x-component of that equation is:

T1,x +T2,x +T3,x = 0 N (1.3)

(+0.758 N)+ (+2.843 N)+T3,x = 0 N (1.4)

The y-component of that equation is:

T1,y +T2,y +T3,y = 0 N (1.5)

(+2.332 N)+ (−0.762 N)+T3,y = 0 N (1.6)

Exercises for PPT α10 November 28, 2019 25 Ch.1 Forces

Page 32: Solutions to Exercises for Physics for Physiotherapy ... - Solutions.pdf · SOLUTIONSto Exercises for Physics for Physiotherapy Technology 203-946-DW Fall 2019 by Chris Roderick,

Step 5: Solving the equations we had above gives

T3,x =−3.601 N (1.7)

T3,y =−1.571 N (1.8)

From these components we find that the magnitude is

T3 =√

(T3,x)2 + (T3,y)2 =√

(−3.601 N)2 + (−1.571 N)2 = 3.928 N (1.9)

From this magnitude of tension we can determine the mass hanging from the end of the string: m3 = T3/

g = 3.928 N/

9.81 N/kg=0.400 kg.

The components tell us that the normal points into the third quadrant (that the angle θ in the diagram is between180 and 270). Using the inverse tangent function, we obtain

tan−1(T3,y/T3,x)= tan−1 ((−1.571 N)

/(−3.601 N)

)= 23.57 (1.10)

The calculator’s answer is in the first quadrant, formed by a triangle whose x and y sides are both positive. Wetranslate this result into the correct quadrant by adding 180:

x

y

~T3

23203

Answer: Thus, the mass hanging from, and the direction of, the third string are m3 = 0.400 kg = 400 grams andθ = 203. Both of these results meet our expectations.

Exercises for PPT α10 November 28, 2019 26 Ch.1 Forces

Page 33: Solutions to Exercises for Physics for Physiotherapy ... - Solutions.pdf · SOLUTIONSto Exercises for Physics for Physiotherapy Technology 203-946-DW Fall 2019 by Chris Roderick,

PROBLEM 1.2.02: A student looking at their cellphone has their head bent over, as shown. The weight of their head is50N, and the tension in the muscles attached to the base of their skull is 60 N. What force is exerted by the 1st cervicalvertebrae onto the base of their skull? (Specify magnitude and direction.)

Solution:

Step 0: The Object is the student’s head.

Step 1: Gravity pulls downwards on the student’s head (the 50 N force). The muscles on the back of their neck pull(the 60 N force) their head towards the vertebrae, which produces a reaction (the unknown force).

Step 2: The Free-Body Diagram and check of sum of forces:

~FG

~Fm

~Fv~FG

~Fm

~Fv

From these qualitative constructions we can see that the force exerted by the vertebrae (~Fv) should be upwards andtowards the left. Also it should be larger in magnitude than either of the other forces since those forces both contributein the same general direction (downwards).

Step 3: The components of the known and unknown forces:

x

y

~FG

FG,x = 0 N

FG,y =−50 N

x

y

~Fm

35325

Fm,x = 60 N cos325 =+49.15 N

Fm,y = 60 N sin325 =−34.42 N

Even though we expect that the force of the vertebrae on the head will be upwards and towards the left we will pretendtotal ignorance of this force’s magnitude and direction. Both components, Fv,x and Fv,x, are unknowns to be solved for.

Newton’s 1st Law applies to the student’s head: ∑~F =~0 N (1.11)

In this specific situation the sum of forces is~FG +~Fm +~Fv =~0 N (1.12)

The x-component of that equation is:

FG,x +Fm,x +Fv,x = 0 N (1.13)

(0 N)+ (+49.15 N)+Fv,x = 0 N (1.14)

The y-component of that equation is:

FG,y +Fm,y +Fv,y = 0 N (1.15)

(−50 N)+ (−34.42 N)+Fv,y = 0 N (1.16)

Step 5: Solving those equations above gives Fv,x =−49.15 N and Fv,y =+84.42 N.

Exercises for PPT α10 November 28, 2019 27 Ch.1 Forces

Page 34: Solutions to Exercises for Physics for Physiotherapy ... - Solutions.pdf · SOLUTIONSto Exercises for Physics for Physiotherapy Technology 203-946-DW Fall 2019 by Chris Roderick,

The components tell us that this force points into the second quadrant (that the angle θ in the diagram is between90 and 180). Using the inverse tangent function, we obtain

tan−1(Fv,y/Fv,x)= tan−1 ((+84.42 N)

/(−49.15 N)

)=−59.79 ≈−60 (1.17)

The calculator’s answer is in the fourth quadrant, formed by a triangle whose x side is positive and whose y side isnegative. We translate this result into the correct quadrant by adding 180:

x

y~Fv

−60

120

Answer: The magnitude of the force of the vertebrae on the head is 97.68 N, and the direction (angle measuredcounter-clockwise from the +x-axis) is 120. This meets both of the expectations we generated from constructing thecheck of the sum of forces.

Exercises for PPT α10 November 28, 2019 28 Ch.1 Forces

Page 35: Solutions to Exercises for Physics for Physiotherapy ... - Solutions.pdf · SOLUTIONSto Exercises for Physics for Physiotherapy Technology 203-946-DW Fall 2019 by Chris Roderick,

PROBLEM 1.2.03: An object of weight 3.70 N is hanging by two ropes, as shown. Find the magnitude of the tension ineach rope.

50

3.70 N

Solution:

Step 0: The Object is the sphere.

Step 1: Gravity pulls downwards on the object (the weight of 3.70 N). Each rope will pull on the object. The object isnot touching a surface, so there will not be any normal nor any friction.

Step 2: The Free-Body Diagram and check of sum of forces:

~FG

~TL

~TR ~FG

~TR

~TL

From these qualitative constructions we can see that the weight is supported by the vertical component of the tensionon the left, and the tension towards the right is countered by the horizontal component of the tension on the left. Forthese reasons we can expect that the magnitude of the tension in the rope on the left will be greater than the weight(TL > mg) and greater than in the rope on the right (TL > TR).

Step 3: The components of the known and unknown forces:

x

y

~FG

FG,x = 0 N

FG,y =−3.70 N

x

y~TL

50130

TL,x = TL cos130

TL,y = TL sin130

x

y

~TR

TR,x =+TR

TR,y = 0 N

The magnitudes of both tensions are unknowns that we must solve for.

Step 4: Newton’s 1st Law applied to the Object is

~FG +~TL +~TR =~0 N (1.18)

The x-component of this equation is

FG,x +TL,x +TR,x = 0 N (1.19)

(0 N)+ (TL cos130)+ (+TR)= 0 N (1.20)

The y-component of this equation is

FG,y +TL,y +TR,y = 0 N (1.21)

(−3.70 N)+ (TL sin130)+ (0 N)= 0 N (1.22)

Step 5: In the y-component equation we can see that there is only one unknown, which can be easily solved for (if weare careful with the signs!):

(−3.70 N)+ (TL sin130)+ (0 N)= 0 N (1.23)

TL =− (−3.70 N)sin130 =+4.83 N (1.24)

Exercises for PPT α10 November 28, 2019 29 Ch.1 Forces

Page 36: Solutions to Exercises for Physics for Physiotherapy ... - Solutions.pdf · SOLUTIONSto Exercises for Physics for Physiotherapy Technology 203-946-DW Fall 2019 by Chris Roderick,

This gets substituted into the x-component equation, which then solves the other unknown:

(0 N)+ (TL cos130)+ (+TR)= 0 N (1.25)

TR =−TL cos130 =−(+4.83 N) cos130 =+3.11 N (1.26)

Answer: We found that TL = 4.83 N and that TR = 3.11 N. This meets our expectations that TL > mg and thatTL > TR .

Exercises for PPT α10 November 28, 2019 30 Ch.1 Forces

Page 37: Solutions to Exercises for Physics for Physiotherapy ... - Solutions.pdf · SOLUTIONSto Exercises for Physics for Physiotherapy Technology 203-946-DW Fall 2019 by Chris Roderick,

PROBLEM 1.2.04: An object of weight 4.20 N is hanging from two ropes that are symmetric (as shown). Find themagnitude of the tension in each rope.

30

4.20 N

Solution: Since the ropes are attached symmetrically we can expect that the two tensions will have the same magni-tude.

Step 0: The Object is the sphere.

Step 1: Gravity pulls downwards on the object (the weight of 4.20 N). Each rope will pull on the object. The object isnot touching a surface so there will not be a normal, nor any friction.

Step 2: The Free-Body Diagram and check of sum of forces:

~FG

~TL ~TR

~FG~TR

~TL

From these qualitative constructions we can see that the tensions should have the same magnitude TL = TR . (Becareful and note that the forces are along different directions, so that the vectors are not equal ~TL 6= ~TR .) Each ropewill support half the weight, but will also have to counter the horizontal component of tension in the other rope. Socan expect that each tension will have a greater magnitude than half the weight.

Step 3: The components of the known and unknown forces:

x

y

~FG

FG,x = 0 N

FG,y =−4.20 N

x

y~TL

30 120

TL,x = TL cos120

TL,y = TL sin120

x

y ~TR

30

60

TR,x = TR cos60

TR,y = TR sin60

The magnitudes of both tensions are unknowns that we must solve for.

Step 4: Newton’s 1st Law applied to the Object is

~FG +~TL +~TR =~0 N (1.27)

The x-component of this equation is

FG,x +TL,x +TR,x = 0 N (1.28)

(0 N)+ (TL cos120)+ (+TR cos60)= 0 N (1.29)

The y-component of this equation is

FG,y +TL,y +TR,y = 0 N (1.30)

(−4.20 N)+ (TL sin120)+ (TR sin60)= 0 N (1.31)

Step 5: We see that both unknowns appear in each equation. There is no single equation where we can get anunknown by itself. To solve we need to first express one unknown in terms of the other, then substitute that into theother equation.

Exercises for PPT α10 November 28, 2019 31 Ch.1 Forces

Page 38: Solutions to Exercises for Physics for Physiotherapy ... - Solutions.pdf · SOLUTIONSto Exercises for Physics for Physiotherapy Technology 203-946-DW Fall 2019 by Chris Roderick,

From the x-component equation we can obtain

(0 N)+ (TL cos120)+ (+TR cos60)= 0 N (1.32)

TL =− cos60

cos120 TR =+TR (1.33)

(This shows that the two tension have equal magnitudes, as we expected from the symmetry of the system.) That factcan then be substituted in the y-component equation: The y-component of this equation is

(−4.20 N)+ (TL sin120)+ (TR sin60)= 0 N (1.34)

(−4.20 N)+ (TL sin120)+ (TL sin60)= 0 N (1.35)

TL (sin120+sin60)=+4.20 N (1.36)

TL =+2.43 N (1.37)

Answer: Both tension have a magnitude of 2.43 N. (As expected the tensions were equal in magnitude, and greaterthan half the weight.)

Exercises for PPT α10 November 28, 2019 32 Ch.1 Forces

Page 39: Solutions to Exercises for Physics for Physiotherapy ... - Solutions.pdf · SOLUTIONSto Exercises for Physics for Physiotherapy Technology 203-946-DW Fall 2019 by Chris Roderick,

PROBLEM 1.2.05: A spherical object of weight 11.50 N rests in a corner of a frictionless surface. Find the magnitudeof the normal exerted by each surface.

frictionless

11.50 N

25

Solution:

Step 0: The Object is the sphere.

Step 1: Gravity pulls downwards on the object (the weight of 11.50 N). Each part of the surface in contact with theobject will exert a contact force. The surface is frictionless, so these contact forces will be just the normals. There areno ropes, nor any other external forces acting on the object.

Step 2: The Free-Body Diagram and check of sum of forces:

~FG

~n1

~n2

~FG

~n1

~n2

From these qualitative constructions we can see that the two normals are just opposites to the components of theweight, as measured relative to the incline. This encourages us to use a coordinate system that is tilted along thesurface when finding the components in the next step.

Another feature we notice is that, with the incline at only 25, the part of the surface that is close to horizontal willsupport more of the weight, and should have a larger magnitude than the other, more vertical, surface.

Step 3: The components of the known and unknown forces:

x

y

horizontal

25

~FG

245

FG,x = 11.50 N cos245 =−4.86 N

FG,y = 11.50 N sin245 =−10.42 N

x

y~n1

n1,x =+n1

n1,y = 0 N

x

y~n2

n2,x = 0 N

n2,y =+n2

Here we can see that the choice of coordinates will make the algebra very simple, with only one unknown along eachof the axes.

Step 4: Newton’s 1st Law applied to the Object is

~FG +~n1 +~n2 =~0 N (1.38)

The x-component of this equation is

FG,x +n1,x +n2,x = 0 N (1.39)

(−4.86 N)+ (+n1)+ (0 N)= 0 N (1.40)

The y-component of this equation is

FG,x +n1,x +n2,x = 0 N (1.41)

(−10.42 N)+ (0 N)+ (+n2)= 0 N (1.42)

Step 5: The tricky part of this problem was the geometry. Here, the algebra is trivial, and we can read the answers

Exercises for PPT α10 November 28, 2019 33 Ch.1 Forces

Page 40: Solutions to Exercises for Physics for Physiotherapy ... - Solutions.pdf · SOLUTIONSto Exercises for Physics for Physiotherapy Technology 203-946-DW Fall 2019 by Chris Roderick,

directly from the equations above:

n1 =+4.86 N (1.43)

n2 =+10.42 N. (1.44)

Answer: The portion of the surface that is most horizontal exerts a normal of magnitude 10.42 N, while the portionthat is mostly vertical exerts a normal of magnitude 4.86 N. (As expected from our qualitative construction in the checkof the sum of forces, the more horizontal surface exerts a larger normal, and supports most of the object’s weight.)

Exercises for PPT α10 November 28, 2019 34 Ch.1 Forces

Page 41: Solutions to Exercises for Physics for Physiotherapy ... - Solutions.pdf · SOLUTIONSto Exercises for Physics for Physiotherapy Technology 203-946-DW Fall 2019 by Chris Roderick,

PROBLEM 1.2.06: Two objects are hanging as shown. Find the magnitude of the tension in each rope.

2.00 N

3.00 N

Solution:

Step 0: There are two objects in this system! The Process applies to a single object, so we will have to do the Processonce for one object, and then, separately, do the Process for the other object. Because the two objects are attached toeach other they are interacting. For this reason the results we obtain for one object may be required to solve what ishappening to the other object. In contexts like this it will Newton’s 3rd Law that relates these pairs of interactionstogether.

For the purposes of this problem we will call the object above “object A” and the object below “object B”.

Step 1: Interactions must be listed for each object, separately.

For object A, there is the interaction with Earth (gravity), the rope above it (which connects it to the ceiling), andthe rope below it (through with it interacts with object B).

For object B, there is the interaction with Earth (gravity), and the rope above it (through with it interacts withobject A).

Step 2: When we construct the Free-Body Diagram and check of sum of forces we must remember that Newton’s 3rdLaw relates the force that A exerts on B to the force that B exerts on A.

For object A we will name ~TB the force that object B exerts on it through the connecting rope. The force that theceiling exerts through the rope above object A we will call ~TC .

~FGA

~TB

~TC~FGA

~TB

~TC

For object B we will name ~TA the force that object A exerts on it through the connecting rope.

~FGB

~TA

~FGB ~TA

In the Free-Body Diagrams above the forces of interaction that are related by Newton’s 3rd Law have been high-lightedin yellow. The force that B exerts on A (~TB) is equal in magnitude, but opposite in direction, to the the force that Aexerts on B (~TA).

Step 3: For object A the components of the known and unknown forces are:

x

y

~FGA FGA,x = 0 N

FGA,y =−2.00 N

x

y~TC

TC,x = 0 N

TC,y =+TC

x

y

~TB TB,x = 0 N

TB,y =−TB

For object B the components of the known and unknown forces are:

Exercises for PPT α10 November 28, 2019 35 Ch.1 Forces

Page 42: Solutions to Exercises for Physics for Physiotherapy ... - Solutions.pdf · SOLUTIONSto Exercises for Physics for Physiotherapy Technology 203-946-DW Fall 2019 by Chris Roderick,

x

y

~FGB FGB,x = 0 N

FGB,y =−3.00 N

x

y~TA

TA,x = 0 N

TA,y =+TA

We we are solving for the unknowns we must remember that Newton’s 3rd Law will require that the magnitudes TAand TB equal each other (TB = TA). The directions being opposite (~TB = −~TA) we have already accounted for in thesigns of the components, above.

Step 4: Newton’s 1st Law applied to object A gives

~FGA +~TC +~TB =~0 N (1.45)

and applied to object B gives~FGB +~TA =~0 N (1.46)

(Read these equations carefully, paying close attention to the sub-scripts.) In each of those equations the x-componentsof every vector is zero. Consequently only the y-components of those equations are relevant. The y-component ofNewton’s 1st Law applied to object A is

FGA,y +TC,x +TB,y = 0 N (1.47)

−2.00 N+TC −TB = 0 N (1.48)

while the y-component of Newton’s 1st Law applied to object B is

FGB,y +TA,y = 0 N (1.49)

−3.00 N+TA = 0 N (1.50)

Step 5: Solving the y-component equation for object B tells us that TA = 3.00 N. This is the magnitude of the tension inthe segment of rope between the two objects. Recalling that Newton’s 3rd Law requires TB = TA, we obtain TB = 3.00 N.Using this fact we can solve the y-component equation for object A to obtain TC = 5.00 N, which is the magnitude ofthe tension in the segment of rope between object A and the ceiling.

Answer: The rope above the objects supports the weight of both objects with a tension of magnitude 5.00 N. The ropebetween the objects supports the weight of the lower object with a tension of magnitude 3.00 N.

Exercises for PPT α10 November 28, 2019 36 Ch.1 Forces

Page 43: Solutions to Exercises for Physics for Physiotherapy ... - Solutions.pdf · SOLUTIONSto Exercises for Physics for Physiotherapy Technology 203-946-DW Fall 2019 by Chris Roderick,

PROBLEM 1.2.07: The tendon from the quadraceps muscle (thigh) passes over the patella (kneecap) to attach to thetibia (shin bone). The tension in the tendon is 1333N. What are the magnitude and direction of the contact forcebetween the patella and the femur (thigh bone)? (There is essentially no friction between the patella and femur. Themass of the patella is only a few grams, so gravity may be ignored relative to the tension in the tendon.)

Solution:

Step 0: The Object is the patella (the kneecap).

Step 1: The patella is attached to a tendon both above and below, so there will be two tensions. These tensions pressthe patella against the end of the femur, so there will be a normal force, and we are told there is no friction.

The patella is a small piece of bone with a mass on the order of 10 grams. Its weight (roughly 0.1 N) is completelynegligible in comparison to the tensions and contact forces exerted on it, so we can forget about gravity.

Step 2: The Free-Body Diagram and check of sum of forces:

~n~T1

~T2

~T1

~T2

~n

Step 3: The components of the known and unknown forces:

x

y~T1

37

143

T1,x = 1333 N cos143 =−1065 N

T1,y = 1333 N sin143 =+802 N

x

y

~T2

80

260

T2,x = 1333 N cos260 =−231 N

T2,y = 1333 N sin260 =−1313 N

x

y~n

θ

nx = nx

ny = ny

The components of the normal, nx and ny, are unknowns for which we must solve. From their values we can calculatethe magnitude and direction, which we are asked for.

Step 2: This is a situation Newton’s 1st Law applies to the Object:

∑~F =~0 N (1.51)

In this specific situation the sum of forces is

~T1 +~T2 +~n =~0 N (1.52)

The components of this constraint are the equations we must solve. The x-component is:

T1,x +T2,x +nx = 0 N (1.53)

(−1065 N)+ (−231 N)+nx = 0 N (1.54)

Exercises for PPT α10 November 28, 2019 37 Ch.1 Forces

Page 44: Solutions to Exercises for Physics for Physiotherapy ... - Solutions.pdf · SOLUTIONSto Exercises for Physics for Physiotherapy Technology 203-946-DW Fall 2019 by Chris Roderick,

The y-component is:

T1,y +T2,y +ny = 0 N (1.55)

(+802 N)+ (−1313 N)+ny = 0 N (1.56)

Step 5: Solving the equations above, we get

nx =+1296 N (1.57)

ny =+511 N (1.58)

From these components we find that the magnitude is

n =√

n2x +n2

y =√

(+1296 N)2 + (+511 N)2 = 1393 N (1.59)

The components tell us that the normal points into the first quadrant (that the angle θ in the diagram is between 0and 90). Using the inverse tangent function, we obtain

tan−1(ny/nx)= tan−1 ((+511 N)

/(+1296 N)

)= 21.5 (1.60)

This is in the first quadrant, as required.

Answer: Thus, the normal force of the end of the femur acting on the patella has a magnitude n = 1393 N ≈ 1.4 kN,and points away from the femur at θ = 21.5 above the horizontal.

Exercises for PPT α10 November 28, 2019 38 Ch.1 Forces

Page 45: Solutions to Exercises for Physics for Physiotherapy ... - Solutions.pdf · SOLUTIONSto Exercises for Physics for Physiotherapy Technology 203-946-DW Fall 2019 by Chris Roderick,

PROBLEM 1.2.08: As I push a chair away from me (as shown below) what are the normal and friction forces at my feetif I do not slide while pushing?

Solution:

Step 0: The problem is asking about the forces acting on me, from which we can conclude that I am the object, not thechair.

Step 1: The force that we are told about is the force that I am exerting on the chair. But the question is about theforces acting on me. So we need to know the force that the chair exerts on me. By Newton’s 3rd Law, the chair exertsa force that is equal in magnitude (125 N) and opposite in direction (30 above the horizontal, towards the left). If thechair is pushing me towards the left, then friction will have to point towards the right.

Present as always will also be gravity (due to my interaction with the Earth) and the normal force of the floor uponmy feet.

Step 2: The Free-Body Diagram and the check of sum of forces:

~FG

~n

~f

~P

~FG

~f

~n

~P

Notice how my pushing slightly downwards on the chair has it pushing slight upwards on me. For this reason thenormal is slightly reduced in magnitude to something less than my weight.

Step 3: The components of the known and unknown forces:

x

y

~FG

FG,x = 0 N

FG,y =−(90 kg)(9.81 N/kg)=−882.9 N

x

y

~f

fx =+ ff y = 0 N

x

y~n

nx = 0 N

ny =+n

x

y~P

30

150

Px = 125 N cos150 =−108.3 N

Py = 125 N sin150 =+62.5 N

The magnitudes of the normal (n) and friction ( f ) are unknowns for which we must solve.

Step 4: In this situation Newton’s 1st Law applies to the Object:

∑~F =~0 N (1.61)

In this specific situation the sum of forces is

~FG +~f +~n+~P =~0 N (1.62)

Exercises for PPT α10 November 28, 2019 39 Ch.1 Forces

Page 46: Solutions to Exercises for Physics for Physiotherapy ... - Solutions.pdf · SOLUTIONSto Exercises for Physics for Physiotherapy Technology 203-946-DW Fall 2019 by Chris Roderick,

The components of this constraint are the equations we must solve. The x-component of this equation is:

FG,x + fx +nx +Px = 0 N (1.63)

(0 N)+ (+ f )+ (0 N)+ (−108.3 N)= 0 N (1.64)

The y-component of this equation is:

FG,y + f y +ny +Py = 0 N (1.65)

(−882.9 N)+ (0 N)+ (+n)+ (+62.5 N)= 0 N (1.66)

Step 5: Solving the x-component equation for the magnitude of the friction:

(0 N)+ (+ f )+ (0 N)+ (−108.3 N)= 0 N (1.67)

f =+108.3 N (1.68)

Note that, if we had assumed the incorrect direction for the friction, we would have obtained a negative number atthis step. Since a magnitude must be positive, that would have signaled that something need to be corrected in someprevious step.

Solving the y-component equation for the magnitude of the normal:

(−882.9 N)+ (0 N)+ (+n)+ (+62.5 N)= 0 N (1.69)

n =+820.4 N (1.70)

As expected from the sum of forces check done with the Free-Body Diagram, the normal is less than the weight since(the reaction to) the push acts upwards on the Object (me).

Answer: The normal acting on me is 820.4 N upwards, and the frictional force acting on me is 108.3 N towards theright.

Exercises for PPT α10 November 28, 2019 40 Ch.1 Forces

Page 47: Solutions to Exercises for Physics for Physiotherapy ... - Solutions.pdf · SOLUTIONSto Exercises for Physics for Physiotherapy Technology 203-946-DW Fall 2019 by Chris Roderick,

PROBLEM 1.2.09: An object remains at rest on an incline when a force is applied as shown in the diagram. The massof the object is 2.70 kg and the magnitude of the applied pull is 1.37 N. What is the magnitude of the normal acting onthe object? What is the magnitude and direction of the static friction acting on the object?

Solution:

Step 0: The rectangular Object is given to us.

Step 1: The object is interacting with the Earth (gravity), the surface (normal and friction), and is being pulled. Sothere will be four distinct forces acting on the object. Since the applied pull is pointed up the incline we will guess thatthe friction points down the incline.

Step 2: The Free-Body Diagram and check of sum of forces:

~FG

~n

~f

~P

~FG

~P

~n~f

Step 3: The components of the known and unknown forces are:

xy

~FG

30240

FG,x = mg cos240 =−13.244 N

FG,y = mg sin240 =−22.938 N

xy

~f

fx =− ff y = 0 N

xy ~n

nx = 0 N

ny =+n

xy

~P

30

Px = 1.37 N cos30 =+1.186 N

Py = 1.37 N sin30 =+0.685 N

The magnitudes of the normal (n) and friction ( f ) are unknowns for which we must solve. If we are incorrect in ourguess about the direction of friction, then we will obtain a negative value for “ f ”. We choose a tilted coordinate systemsince it simplifies the algebra for the unknowns by not mixing each unknown onto both axes.

Step 4: In this situation Newton’s 1st Law applies to the Object:∑~F =~0 N (1.71)

In this specific situation the sum of forces is~FG +~f +~n+~P =~0 N (1.72)

The components of this constraint are the equations we must solve. The x-component of this equation is:

FG,x + fx +nx +Px = 0 N (1.73)

(−13.244 N)+ (− f )+ (0 N)+ (+1.186 N)= 0 N (1.74)

The y-component of this equation is:

FG,y + f y +ny +Py = 0 N (1.75)

(−22.253 N)+ (0 N)+ (+n)+ (+0.685 N)= 0 N (1.76)

Exercises for PPT α10 November 28, 2019 41 Ch.1 Forces

Page 48: Solutions to Exercises for Physics for Physiotherapy ... - Solutions.pdf · SOLUTIONSto Exercises for Physics for Physiotherapy Technology 203-946-DW Fall 2019 by Chris Roderick,

Step 5: The x-component equation can be solved for the magnitude of the friction:

(−13.244 N)+ (− f )+ (0 N)+ (+1.186 N)= 0 N (1.77)

f =−12.057 N (1.78)

Since a magnitude must be positive, this signaled that our assumption about the direction of friction was incorrect. Tomaintain static equilibrium, friction must point upwards along the incline.

The y-component equation can be solved for the magnitude of the normal:

(−22.253 N)+ (0 N)+ (+n)+ (+0.685 N)= 0 N (1.79)

n =+22.253 N (1.80)

As expect from the Free-Body Diagram, the normal is less than the weight since the pull acts slightly away from thesurface.

Answer: The normal acting on the object is 22.3 N. The force of static friction acting on the object is 12.1 N pointingupwards along the incline.

Exercises for PPT α10 November 28, 2019 42 Ch.1 Forces

Page 49: Solutions to Exercises for Physics for Physiotherapy ... - Solutions.pdf · SOLUTIONSto Exercises for Physics for Physiotherapy Technology 203-946-DW Fall 2019 by Chris Roderick,

1.3 Pulleys

A pulley is a machine that changes the direction of a rope but not the tension. The following exercises investigate themechanical consequences of that fact. The context, as always, is static equilibrium.

In each of the exercises below find the tension in each rope, when possible. Where asked, find the unknownexternally applied force (appearing as a red vector in the diagrams).

To solve for the tension trace along the rope and treat each pulley as if it was its own object. Each pulley being instatic equilibrium means that the sum of forces acting on each pulley must sum to zero, individually.

Pay very close attention to how the system of pulleys attaches to weights in the problem; you will quite often findthat the tension in the rope is only a fraction of the object’s weight.

1.3.1 A Single Pulley

EXR 1.3.01

12 N

T = 12 N= F.

EXR 1.3.02

45

12 N

T = 12 N= F.

EXR 1.3.03

28 N

T = 14 N.

EXR 1.3.04

28 N

T = 14 N.

EXR 1.3.05

60

28 N

T = 16.17 N.

EXR 1.3.06

28 N

T = 14 N= F.

EXR 1.3.07

60

28 N

T = 16.17 N= F.

EXR 1.3.08

T = 16 N= F.

EXR 1.3.09

T = 8 N.

EXR 1.3.10

T = 5.49 N= F.

EXR 1.3.11

T = 5.49 N= F.

Exercises for PPT α10 November 28, 2019 43 Ch.1 Forces

Page 50: Solutions to Exercises for Physics for Physiotherapy ... - Solutions.pdf · SOLUTIONSto Exercises for Physics for Physiotherapy Technology 203-946-DW Fall 2019 by Chris Roderick,

EXR 1.3.12

T = 2.59 N= F.

EXR 1.3.13

T = 2.77 N= F.

EXR 1.3.14

T = 23.33 N= F.

1.3.2 Two Pulleys

EXR 1.3.15

30 N

T = 15 N= F.

EXR 1.3.16

30 N

T = 15 N= F.

EXR 1.3.17

18 N

T = 18 N= F.

EXR 1.3.18

45

7 NT = 4.94 N= F.

EXR 1.3.19

T = 10 N= F.

EXR 1.3.20

T = 9 N= F.

Exercises for PPT α10 November 28, 2019 44 Ch.1 Forces

Page 51: Solutions to Exercises for Physics for Physiotherapy ... - Solutions.pdf · SOLUTIONSto Exercises for Physics for Physiotherapy Technology 203-946-DW Fall 2019 by Chris Roderick,

EXR 1.3.21

T = 9 N,F = 18 N.

EXR 1.3.22

T = 18 N,F = 36 N.

EXR 1.3.23

T = 13 N,F = 26 N.

EXR 1.3.24

T = 12 N.

EXR 1.3.25

T = 5 N.

EXR 1.3.26

T = 5 N.

1.3.3 Three or more Pulleys

EXR 1.3.27

96 NT = 24 N.

EXR 1.3.28

75 N

T = 25 N= F.

1.3.4 Two or more Ropes

In each of these systems find the tension in each of the ropes.

Exercises for PPT α10 November 28, 2019 45 Ch.1 Forces

Page 52: Solutions to Exercises for Physics for Physiotherapy ... - Solutions.pdf · SOLUTIONSto Exercises for Physics for Physiotherapy Technology 203-946-DW Fall 2019 by Chris Roderick,

EXR 1.3.29

TA = 16 N,TB = 8 N= F.

EXR 1.3.30

TA = 21 N,TB = 10.5 N.

EXR 1.3.31

TA = 5 N= F,TB = 10 N.

EXR 1.3.32

TA = 14 N,TB = 7 N,F = 21 N.

EXR 1.3.33

TA = 500 N,TB = 250 N,TC = 125 N= F.

EXR 1.3.34

TB = 13 N= F,θA = 22.5,TA = 24.02 N.

EXR 1.3.35

TB = 7 N= F,TA = 10.00 N.

Exercises for PPT α10 November 28, 2019 46 Ch.1 Forces

Page 53: Solutions to Exercises for Physics for Physiotherapy ... - Solutions.pdf · SOLUTIONSto Exercises for Physics for Physiotherapy Technology 203-946-DW Fall 2019 by Chris Roderick,

Chapter 2

Torques

2.1 Torque : Qualitative Exercises

2.1.1 Sign of Torque

In each of the following exercises find the sign of the torque about the pivot produced by the single applied force. Ifthe applied force produces a torque of magnitude zero, say so. Remember that the sign of the torque is the sign ofthe rotation that would happen if that torque was the only one being applied. (The sign of rotation follows the signconvention of angles, with counter-clockwise being positive and clockwise being negative.)

EXR 2.1.01

Neg

EXR 2.1.02

Pos

EXR 2.1.03

Pos

EXR 2.1.04

Neg

EXR 2.1.05

Pos

EXR 2.1.06

Pos

EXR 2.1.07Neg

EXR 2.1.08

Pos

EXR 2.1.09

Pos

EXR 2.1.10Neg

EXR 2.1.11

ZERO

EXR 2.1.12

Pos

EXR 2.1.13Neg

EXR 2.1.14

Pos

EXR 2.1.15

ZERO

2.1.2 Line of Action

In the following exercises we use the line of action to reason qualitatively about the magnitude of torque. The contextis static equilibrium, in which

∑~τ=~0 N·m about any axis. With the object in the xy-plane (the page), the z-axis will be

through the chosen pivot (perpendicular to the page), and∑τz = 0 N·m about that axis.

In the cases where we are given a force, the exercise is to find the line of action along which it must act so thatthe object remains in static equilibrium. In the cases where we are given a line of action, the exercise is to find(qualitatively) the force that would, acting along that line, keep the object in static equilibrium. In all cases the centerof mass of the object is at the geometric center of the object.

Exercises for PPT α10 November 28, 2019 47 Ch.2 Torques

Page 54: Solutions to Exercises for Physics for Physiotherapy ... - Solutions.pdf · SOLUTIONSto Exercises for Physics for Physiotherapy Technology 203-946-DW Fall 2019 by Chris Roderick,

Thin rectangular rod

EXR 2.1.16

EXR 2.1.17

EXR 2.1.18

EXR 2.1.19

Square and rectangular objects

Circular objects

Irregularly-shaped objects

2.2 Torque : Quantitative Exercises

2.2.1 Single applied Force

In each of these exercises calculate:(1) the angle θ from the direction of~r to the direction of ~F;(2) the component F⊥ of the force that exerts a torque; and(3) the z-component of the torque (τz) exerted about the pivot by the applied force.

Right Angles

EXERCISE 2.2.01

Exercises for PPT α10 November 28, 2019 48 Ch.2 Torques

Page 55: Solutions to Exercises for Physics for Physiotherapy ... - Solutions.pdf · SOLUTIONSto Exercises for Physics for Physiotherapy Technology 203-946-DW Fall 2019 by Chris Roderick,

2m

10N

90

(1) The angle is given as θ =+90, so the torque τz will be positive.(2) F⊥ = F sinθ = 10 N×sin(+90)=+10 N.(3) τz = F⊥ r = (+10 N)(2 m)=+20 N·m.

EXERCISE 2.2.022m

10N

90

(1) The angle is given as θ =−90 (clockwise), so the torque τz will be negative.(2) F⊥ = F sinθ = 10 N×sin(−90)=−10 N.(3) τz = F⊥ r = (−10 N)(2 m)=−20 N·m.

EXERCISE 2.2.03

5m

10N

90

(1) The angle is given as θ =+90, so the torque τz will be positive.(2) F⊥ = F sinθ = 10 N×sin(+90)=+10 N.(3) τz = F⊥ r = (+10 N)(5 m)=+50 N·m.

EXERCISE 2.2.046m

5N

90

(1) The angle is given as θ =−90 (clockwise), so the torque τz will be negative.(2) F⊥ = F sinθ = 5 N×sin(−90)=−5 N.(3) τz = F⊥ r = (−5 N)(6 m)=−30 N·m.

EXERCISE 2.2.05

8m

5N

90

(1) The angle is given as θ =+90, so the torque τz will be positive.(2) F⊥ = F sinθ = 5 N×sin(+90)=+5 N.(3) τz = F⊥ r = (+5 N)(8 m)=+40 N·m.

EXERCISE 2.2.06

Exercises for PPT α10 November 28, 2019 49 Ch.2 Torques

Page 56: Solutions to Exercises for Physics for Physiotherapy ... - Solutions.pdf · SOLUTIONSto Exercises for Physics for Physiotherapy Technology 203-946-DW Fall 2019 by Chris Roderick,

6m 5N

90

(1) The angle is given as θ =−90 (clockwise), so the torque τz will be negative.(2) F⊥ = F sinθ = 5 N×sin(−90)=−5 N.(3) τz = F⊥ r = (−5 N)(6 m)=−30 N·m.

Notice how, in this exercise, we have r y > 0 m and Fx > 0 N, but τz < 0 N·m due to the relation between theirdirections. Remember this example for how it shows that the vectors (specifically the signs of their components)independent of each other do not give the sign of the torque. The sign of τz is determined by how the directions of~r and~F relate to each other.

EXERCISE 2.2.07

5m7N

90

(1) The angle is given as θ =+90, so the torque τz will be positive.(2) F⊥ = F sinθ = 7 N×sin(+90)=+7 N.(3) τz = F⊥ r = (+7 N)(5 m)=+35 N·m.

EXERCISE 2.2.08

7m 4N

90

(1) The angle is given as θ =+90, so the torque τz will be positive.(2) F⊥ = F sinθ = 4 N×sin(+90)=+4 N.(3) τz = F⊥ r = (+4 N)(7 m)=+28 N·m.

EXERCISE 2.2.09

Exercises for PPT α10 November 28, 2019 50 Ch.2 Torques

Page 57: Solutions to Exercises for Physics for Physiotherapy ... - Solutions.pdf · SOLUTIONSto Exercises for Physics for Physiotherapy Technology 203-946-DW Fall 2019 by Chris Roderick,

3m11N

90

(1) The angle is given as θ =−90 (clockwise), so the torque τz will be negative.(2) F⊥ = F sinθ = 11 N×sin(−90)=−11 N.(3) τz = F⊥ r = (−11 N)(3 m)=−33 N·m.

Non-Right Angles

EXERCISE 2.2.10

6m

5N

45

(1) The angle is given as θ =+45, so the torque τz will be positive.(2) F⊥ = F sinθ = 5 N×sin(+45)=+3.536 N.(3) τz = F⊥ r = (+3.536 N)(6 m)=+21.2 N·m.

EXERCISE 2.2.11

6m

5N

45

135

(1) In the diagram the angle is found to be θ =+135, so the torque τz will be positive.(2) F⊥ = F sinθ = 5 N×sin(+135)=+3.536 N.(3) τz = F⊥ r = (+3.536 N)(6 m)=+21.2 N·m.

General Cases

In these cases you must solve the geometry to find the angle θ from the direction of~r towards the direction of ~F. Recallhow to identify complementary angles, and rules like the transverse-parallel theorem. Be very careful to get the signof θ correct. These exercises may require a few steps to solve the geometry. Be sure to draw larger diagrams thanusual so that you can label the parts of the geometry clearly.

EXERCISE 2.2.12

30

30

5m

17N30

120

(1) In the diagram the angle is found to be θ =+120, so the torque τz will be positive.(2) F⊥ = F sinθ = 17 N×sin(+120)=+14.722 N(3) τz = F⊥ r = (+14.722 N)(5 m)=+73.6 N·m.

Exercises for PPT α10 November 28, 2019 51 Ch.2 Torques

Page 58: Solutions to Exercises for Physics for Physiotherapy ... - Solutions.pdf · SOLUTIONSto Exercises for Physics for Physiotherapy Technology 203-946-DW Fall 2019 by Chris Roderick,

EXERCISE 2.2.13

60

20

6m

17N60

80

(1) In the diagram the angle is found to be θ =−80, so the torque τz will be negative.(2) F⊥ = F sinθ = 17 N×sin(−80)=−16.742 N(3) τz = F⊥ r = (−16.742 N)(6 m)=−100. N·m.

EXERCISE 2.2.14 (Challenge)

4520

4m

21N2545

(1) In the diagram the angle is found to be θ =+25, so the torque τz will be positive.(2) F⊥ = F sinθ = 21 N×sin(+25)=+8.875 N(3) τz = F⊥ r = (+8.875 N)(4 m)=+35.5 N·m.

2.3 Solving Problems using the Process

2.3.1 Solving for Forces using Torque

A mechanical system is said to be in static equilibrium when the sum of forces is zero and the sum of torques is alsozero:

∑Fx = 0 N (2.1)∑Fy = 0 N (2.2)∑τz = 0 N·m (2.3)

In the exercises below we will begin by practicing how to solve for forces quantitatively using the equilibrium of torques.We will begin with simple geometric shapes, like squares, rectangles and circles. After that we will practice applyingthe rules of static equilibrium to mechanical objects of various shapes. Follow the steps of The Process!

Rectangular Solids

In each of the exercises that follow determine:

(1) Find the magnitude of the applied force that keeps the box in equilibrium; and

(2) Find the magnitude and direction of the force acting at the pivot.

In all cases, unless otherwise indicated, the force of gravity acts at the geometric center of the object. The magnitudeof ~FG is the value printed on the object.

EXERCISE 2.3.01

Exercises for PPT α10 November 28, 2019 52 Ch.2 Torques

Page 59: Solutions to Exercises for Physics for Physiotherapy ... - Solutions.pdf · SOLUTIONSto Exercises for Physics for Physiotherapy Technology 203-946-DW Fall 2019 by Chris Roderick,

pivot

52 N0.

40m

1.20m

Step 0: The object is the box.

Step 1: There is gravity acting downwards at the center of the object, which exerts a clock-wise torque. And there isthe applied upward force acting at the bottom right-hand corner, which applies a counter-clock-wise torque. There maybe a force at the pivot, but it does not exert a torque.

Step 2: The Free-Body Diagram, and the check of the sum of forces is below. Remember that we are only able to guessthe force at the pivot on the FBD, and that is after we’ve tried to make the sum of forces diagram.

pivot

~FG ~FA~FP~FG

~FA

~FP

Gravity is vertically downwards and the applied force is vertically upwards. Since the applied force is further fromthe pivot, to balance the torques we will expect that the applied force is smaller (in magnitude) than gravity. This leadsus to conclude that the force at the pivot will have to be vertically upwards, too. (We won’t assume this, but will checkour final answer against this reasoning.)

Step 3: The components of the forces, and the contributions to the torque about the pivot are determined below.With the forces being purely vertical, the moment arms are clearly visible on the rectangular object, and we can use“τz =±Fd” formula to calculate each torque.

pivot ~FG0.60m

τG,z =−(52 N)(0.60m)=−31.20 N·mx

y

~FG

FG,x = 0 N

FG,y =−52 N

pivot

~FA

1.20m

τA,z =+FA ×1.20mx

y~FA

FA,x = 0 N

FA,y =+FA

The components of the force at the pivot (FP,x and FP,y) are both unknowns. This force exerts no torque (τP,z = 0 N·m)since it is acting at the pivot (its moment arm is d = 0 m).

Step 4: The conditions for static equilibrium given by Newton’s 1st Law are

FG,x +FA,x +FP,x = 0 N 0 N+0 N+FP,x = 0 N (2.4)

FG,y +FA,y +FP,y = 0 N (−52 N)+ (+FA)+FP,y = 0 N (2.5)

τG,z +τA,z +τP,z = 0 N·m (−31.20 N·m)+ (+FA ×1.20m)+ (0 N·m)= 0 N·m (2.6)

Notice how the force at the pivot does not contribute in the torque equation, leaving only one unknown, and making itvery easy to solve. This will almost always happen in problems of static equilibrium when we choose to calculate thetorques about the pivot.

Exercises for PPT α10 November 28, 2019 53 Ch.2 Torques

Page 60: Solutions to Exercises for Physics for Physiotherapy ... - Solutions.pdf · SOLUTIONSto Exercises for Physics for Physiotherapy Technology 203-946-DW Fall 2019 by Chris Roderick,

Step 5: Solving the torque equation we find that FA = 26.0 N. Substituting that value in the equation for the sum ofthe y-components, we find that FP,y =+26.0 N. Evaluating the equation for the sum of the x-components, we see thatFP,x = 0 N.

Answer: The applied force that maintains static equilibrium is 26.0 N, vertically upwards. The force at the pivot is(also) 26.0 N, vertically upwards. (Checking against our physical reasoning in Step 2, we see that our answer makessense.)

EXERCISE 2.3.02

pivot

52 N

0.40

m

1.20m

0.60m

Answer: The applied force that maintains static equilibrium is 52.0 N, vertically upwards. The force at the pivot iszero since the applied force supports the entire weight.

EXERCISE 2.3.03

pivot

52 N

0.40

m

1.20m

0.40m

Answer: The applied force that maintains static equilibrium is 39.0 N, vertically upwards. The force at the pivot is13.0 N, vertically upwards. Compare this result with the two previous exercises.

EXERCISE 2.3.04

pivot

52 N

0.40

m

1.20m

0.40m

Answer: The applied force that maintains static equilibrium is 78.0 N, vertically upwards. This is larger than theweight because it is closer to the pivot than the weight. The force at the pivot is 26.0 N, vertically downwards. Becausethe applied force is upwards greater than the weight the pivot has to supply a downward force to keep the objectattached to the pivot.

EXERCISE 2.3.05

pivot

52 N

0.40

m

1.20m

Step 0: The object is the box.

Step 1: There is gravity acting downwards at the center of the object, which exerts a clock-wise torque. And there isthe applied force acting towards the left at the top right-hand corner, which applies a counter-clock-wise torque. Theremust be a force at the pivot because the applied force and gravity (being along different directions) can not cancel eachother. The force at the pivot does not exert a torque.

Step 2: The Free-Body Diagram, and the check of the sum of forces is below. Remember that we are only able to guessthe force at the pivot on the FBD, and that is after we’ve tried to make the sum of forces diagram.

Exercises for PPT α10 November 28, 2019 54 Ch.2 Torques

Page 61: Solutions to Exercises for Physics for Physiotherapy ... - Solutions.pdf · SOLUTIONSto Exercises for Physics for Physiotherapy Technology 203-946-DW Fall 2019 by Chris Roderick,

pivot

~FG

~FA

~FP

~FG

~FA

~FP

Gravity is vertically downwards and the applied force is horizontal towards the left. Since the applied force is closerto the pivot, to balance the torques we will expect that the applied force is larger (in magnitude) than gravity. Thisleads us to conclude that the force at the pivot will have to be towards the right and slightly upwards.

Step 3: The components of the forces, and the contributions to the torque about the pivot are determined below. Withgravity and the applied force being vertical and horizontal, respectively, the moment arms are clearly visible on therectangular object, and we can use “τz =±Fd” formula to calculate each torque.

pivot ~FG0.60m

τG,z =−(52 N)(0.60m)=−31.20 N·mx

y

~FG

FG,x = 0 N

FG,y =−52 N

pivot

~FA

0.40

m

τA,z =+FA ×0.40mx

y

~FA

FA,x =−FA

FA,y = 0 N

The components of the force at the pivot (FP,x and FP,y) are both unknowns. This force exerts no torque (τP,z = 0 N·m)since it is acting at the pivot (its moment arm is d = 0 m).

Step 4: The conditions for static equilibrium given by Newton’s 1st Law are

FG,x +FA,x +FP,x = 0 N 0 N+ (−FA)+FP,x = 0 N (2.7)

FG,y +FA,y +FP,y = 0 N (−52 N)+ (0 N)+FP,y = 0 N (2.8)

τG,z +τA,z +τP,z = 0 N·m (−31.20 N·m)+ (+FA ×0.40m)+ (0 N·m)= 0 N·m (2.9)

Step 5: Solving the torque equation we find that FA = 78.0 N. Substituting this into the equation for the sum ofthe x-components, we see that FP,x = +78.0 N. Solving the equation for the sum of the y-components, we find thatFP,y =+52.0 N.

The force at the pivot has magnitude

FP =√

F2P,x +F2

P,y =√

(+78.0 N)2 + (+52.0 N)2 = 93.7 N (2.10)

The vector ~FP points into the first quadrant, making an angle

θ = tan−1(FP,y

FP,x

)= tan−1

(+52.0 N+78.0 N

)= 33.7 (2.11)

above the +x-axis.

Answer: The applied force that maintains static equilibrium is 78 N, towards the left. This is larger than the weightbecause its line of action is closer to the pivot than the weight’s. The force at the pivot is 94 N, to the right andslightly upwards at an angle of 34 above the +x-direction. Because the applied force is towards the left and gravity isdownwards the pivot has to supply a force that balances both these components.

Exercises for PPT α10 November 28, 2019 55 Ch.2 Torques

Page 62: Solutions to Exercises for Physics for Physiotherapy ... - Solutions.pdf · SOLUTIONSto Exercises for Physics for Physiotherapy Technology 203-946-DW Fall 2019 by Chris Roderick,

EXERCISE 2.3.06

pivot

52 N

0.40

m1.20m

0.30m

Answer: The applied force that maintains static equilibrium is 312 N, towards the left. This is much larger than theweight because its line of action is so much closer to the pivot than the weight’s. The force at the pivot is 316 N, to theright and very slightly upwards at an angle of 9.5 above the +x-direction. Because the applied force is towards theleft and gravity is downwards the pivot has to supply a force that balances both these components.

EXERCISE 2.3.07

pivot

52 N

0.40

m

1.20m

45

+45

Answer: The applied force that maintains static equilibrium is 37 N, upwards and to the right at 45 above the +x-axis. This is a little more than half the weight because its perpendicular component (F⊥) is half the weight. The forceat the pivot is 37 N, upwards and to the left at 45 above the −x-axis. Because the applied force has a portion towardsthe right and gravity is downwards the pivot has to supply a force that balances both these components.

EXERCISE 2.3.08

pivot

52 N

0.40

m

1.20m

45

+135

Answer: The applied force that maintains static equilibrium is 37 N, upwards and to the left at 45 above the −x-axis.This is a little more than half the weight because its perpendicular component (F⊥) is half the weight. The force at thepivot is 37 N, upwards and to the right at 45 above the +x-axis. Because the applied force has a portion towards theleft and gravity is downwards the pivot has to supply a force that balances both these components.

EXERCISE 2.3.09

pivot

52 N

0.40m

1.20m

45

~r

+45

Step 0: The object is the box.

Step 1: There is gravity acting downwards at the center of the object, which exerts a clock-wise torque. And there isthe applied force acting towards the left at the top left-hand corner, which applies a counter-clockwise torque. Theremust be a force at the pivot because the applied force and gravity (being along different directions) can not cancel eachother. The force at the pivot does not exert a torque.

Step 2: The Free-Body Diagram, and the check of the sum of forces is below. Remember that we are only able to guessthe force at the pivot on the FBD, and that is after we’ve tried to make the sum of forces diagram.

Exercises for PPT α10 November 28, 2019 56 Ch.2 Torques

Page 63: Solutions to Exercises for Physics for Physiotherapy ... - Solutions.pdf · SOLUTIONSto Exercises for Physics for Physiotherapy Technology 203-946-DW Fall 2019 by Chris Roderick,

pivot

~FG

~FA

~FP ~FG~FA

~FP

Gravity is vertically downwards and the applied force is angled upwards towards the left. Since the applied forceis closer to the pivot, to balance the torques we will expect that the applied force is larger (in magnitude) than gravity.This leads us to conclude that the force at the pivot will have to point towards the right, and may be angled slightlyeither upwards or downwards.

(Advanced: Since the applied force makes a 45 angle with the horizontal axes, its x and y components will have thesame size. Since the vector~r from the pivot to the applied force is small than the moment arm for gravity, perpendicularcomponent F⊥ of the applied force will have to be bigger than the weight. But F⊥ is the same as the x-component ofthe applied force. This means that the force at the pivot will have to point downwards along with gravity to balancethe vertical portion of the applied force.)

Step 3: The components of the forces, and the contributions to the torque about the pivot are determined below. Withgravity the moment arm is clearly visible on the rectangular object, and we can use “τz =±Fd” formula to calculate itstorque. For the applied force will have to use the “τz = Frsinθ” formula to find its contribution to torque.

pivot ~FG0.60m

τG,z =−(52 N)(0.60m)=−31.20 N·mx

y

~FG

FG,x = 0 N

FG,y =−52 N

pivot

~FA

~r

+45

τA,z = FA ×0.40m×sin(+45)x

y~FA

FA,x = FA cos135

FA,y = FA sin135

The components of the force at the pivot (FP,x and FP,y) are both unknowns. This force exerts no torque (τP,z = 0 N·m)since it is acting at the pivot (its moment arm is d = 0 m).

Step 4: The conditions for static equilibrium given by Newton’s 1st Law are

FG,x +FA,x +FP,x = 0 N 0 N+ (FA cos135)+FP,x = 0 N (2.12)

FG,y +FA,y +FP,y = 0 N (−52 N)+ (FA sin135)+FP,y = 0 N (2.13)

τG,z +τA,z +τP,z = 0 N·m (−31.20 N·m)+ (FA ×0.40m×sin(+45)

)+ (0 N·m)= 0 N·m (2.14)

Step 5: Solving the torque equation we find that the magnitude of the applied force is FA = 110.3 N (which is largerthan the weight, as expected). Substituting this into the equation for the sum of the x-components, we see thatFP,x =+78.0 N. Solving the equation for the sum of the y-components, we find that FP,y =−26.0 N.

The force at the pivot has magnitude

FP =√

F2P,x +F2

P,y =√

(+78.0 N)2 + (−26.0 N)2 = 82.2 N (2.15)

Exercises for PPT α10 November 28, 2019 57 Ch.2 Torques

Page 64: Solutions to Exercises for Physics for Physiotherapy ... - Solutions.pdf · SOLUTIONSto Exercises for Physics for Physiotherapy Technology 203-946-DW Fall 2019 by Chris Roderick,

The vector ~FP points into the fourth quadrant (below the +x-axis), making an angle

θ = tan−1(FP,y

FP,x

)= tan−1

(−26.0 N+78.0 N

)=−18.4 (2.16)

below the +x-axis.

Answer: The applied force that maintains static equilibrium is 110 N, to the left and upwards at 45. This is largerthan the weight because its line of action is closer to the pivot than the weight’s. The force at the pivot is 82.2 N, to theright and slightly downwards at an angle of 18 below the +x-direction.

EXERCISE 2.3.10 (Challenge)

pivot

52 N

0.40

m

1.20m

45

~r

θ

Step 0: The object is the box.

Step 1: There is gravity acting downwards at the center of the object, which exerts a clock-wise torque. And there isthe applied force acting towards the left at the top right-hand corner, which applies a counter-clockwise torque. Theremust be a force at the pivot because the applied force and gravity (being along different directions) can not cancel eachother. The force at the pivot does not exert a torque.

Step 2: The Free-Body Diagram, and the check of the sum of forces is below. Remember that we are only able to guessthe force at the pivot on the FBD, and that is after we’ve tried to make the sum of forces diagram.

pivot

~FG

~FA

~FP

~FG

~FA

~FP

Gravity is vertically downwards and the applied force is angled upwards towards the left. Since the applied forceis further from the pivot, to balance the torques we will expect that the applied force is smaller (in magnitude) thangravity. This leads us to conclude that the force at the pivot will have to point towards the right, and is probably angledupwards.

Step 3: The components of the forces, and the contributions to the torque about the pivot are determined below. Withgravity the moment arm is clearly visible on the rectangular object, and we can use “τz =±Fd” formula to calculate itstorque. For the applied force will have to use the “τz = Frsinθ” formula to find its contribution to torque.

pivot ~FG0.60m

τG,z =−(52 N)(0.60m)=−31.20 N·mx

y

~FG

FG,x = 0 N

FG,y =−52 N

Exercises for PPT α10 November 28, 2019 58 Ch.2 Torques

Page 65: Solutions to Exercises for Physics for Physiotherapy ... - Solutions.pdf · SOLUTIONSto Exercises for Physics for Physiotherapy Technology 203-946-DW Fall 2019 by Chris Roderick,

pivot

~FA

~r

18.4

18.4

θ

45

τA,z = FA ×1.265m×sin(+135−18.4)=+FA ×1.131m

rx =+1.20 m

r y =+0.40 m

r =√

r2x + r2

y = 1.265 m

tan−1(+0.40 m+1.20 m )= 18.4x

y~FA

FA,x = FA cos135

FA,y = FA sin135

The length of the radial vector ~r is 1.265 m. The angle between the horizontal and ~r is 18.4. Thus the anglebetween the radial vector and the applied force is θ =+135−18.4 = 116.6.

The components of the force at the pivot (FP,x and FP,y) are both unknowns. This force exerts no torque (τP,z = 0 N·m)since it is acting at the pivot (its moment arm is d = 0 m).

Step 4: The conditions for static equilibrium given by Newton’s 1st Law are

FG,x +FA,x +FP,x = 0 N 0 N+ (FA cos135)+FP,x = 0 N (2.17)

FG,y +FA,y +FP,y = 0 N (−52 N)+ (FA sin135)+FP,y = 0 N (2.18)

τG,z +τA,z +τP,z = 0 N·m (−31.20 N·m)+ (FA ×1.131m)+ (0 N·m)= 0 N·m (2.19)

Step 5: Solving the torque equation we find that the magnitude of the applied force is FA = 27.6 N (which is largerthan the weight, as expected). Substituting this into the equation for the sum of the x-components, we see thatFP,x =+19.5 N. Solving the equation for the sum of the y-components, we find that FP,y =+32.5 N.

The force at the pivot has magnitude

FP =√

F2P,x +F2

P,y =√

(+19.5 N)2 + (+32.5 N)2 = 37.9 N (2.20)

The vector ~FP points into the first quadrant, making an angle

θ = tan−1(FP,y

FP,x

)= tan−1

(+32.5 N+19.5 N

)= 59.0 (2.21)

above the +x-axis.

Answer: The applied force that maintains static equilibrium is 28 N, to the left and upwards at 45. This is smallerthan the weight because its line of action is further from the pivot than the weight’s. The force at the pivot is 38 N,upwards to the right at an angle of 59 above the +x-direction.

Rods, Ropes & Surfaces

In each of the following situations, solve for each of the unknown forces. Find the normal at each surface of contact.In the cases when it is non-zero, find the magnitude and direction of friction. Find the tension in each rope that ispresent.

A hint on how to start problems of this type: Chose a point of contact with a surface as the pivot, preferably onewhere there is friction (when it is present). This will eliminate one (perhaps two!) unknowns from the torque equation,simplifying the algebra.

EXERCISE 2.3.11

Exercises for PPT α10 November 28, 2019 59 Ch.2 Torques

Page 66: Solutions to Exercises for Physics for Physiotherapy ... - Solutions.pdf · SOLUTIONSto Exercises for Physics for Physiotherapy Technology 203-946-DW Fall 2019 by Chris Roderick,

~FG

~n1

~f1

~n2

EXERCISE 2.3.12

~FG

~T

~f

~n

EXERCISE 2.3.13

~FG

~T

~f

~n

EXERCISE 2.3.14

~FG

~T

~f

~n

EXERCISE 2.3.15

~FG

~n

~f

~T

Exercises for PPT α10 November 28, 2019 60 Ch.2 Torques

Page 67: Solutions to Exercises for Physics for Physiotherapy ... - Solutions.pdf · SOLUTIONSto Exercises for Physics for Physiotherapy Technology 203-946-DW Fall 2019 by Chris Roderick,

2.3.2 Biomechanical Equilibrium

In all the problems that follow, after finding the unknown force, find the force exerted at the pivot.

EXERCISE 2.3.16

A person is holding a 7.20 kg mass in their hand. What total amount of force must the muscles of their upper armbe exerting on their lower arm? The pivot in this situation is the elbow joint. The active muscles are the biceps andbrachialis, which are the large muscles on the front side of the upper arm. They attach to the forearm 4.5 cm from theelbow joint, as shown in the diagram. The forearm (including the hand) has a mass of 1.80 kg, and has its center ofmass 15.0 cm from the elbow joint.

0cm 4.5cm 34cm

7.20kg

Step 0: The object is the person’s forearm including the hand.

Step 1: The object is interacting with the following things: The Earth exerts a gravitational pull on the forearm. Themass in the person’s hand is pressing downwards. The muscles of the upper arm are pulling upwards on the forearm,which bring the bones into contact with each other.

Step 2: For the purposes of finding the sum of forces and torques, the Free-Body Diagram for a complex system likethis portion of the human body can be draw schematically (as a cartoon):

~FJ ~FM

~FG

~Fw

With all the forces being vertical, two of which are unknown, we can not (yet) construct a check of the sum of forces. Itis our guess that the contact force at the joint (~FJ) will be pointed downwards.

Step 3: All the forces acting in the situation are vertical, so there are no x-components. The y-components are

FG,y =−9.81 Nkg ×1.80kg=−17.658 N (2.22)

Fw,y =−9.81 Nkg ×7.20kg=−70.632 N (2.23)

(The y-components of ~FJ and ~FM are unknowns to be solved.)

Exercises for PPT α10 November 28, 2019 61 Ch.2 Torques

Page 68: Solutions to Exercises for Physics for Physiotherapy ... - Solutions.pdf · SOLUTIONSto Exercises for Physics for Physiotherapy Technology 203-946-DW Fall 2019 by Chris Roderick,

The contributions from these forces to the torque about the pivot are:

~FM

4.5cm

τM =+0.045m×FM

~FG

15.0cm

τG =−0.150m×9.81 Nkg ×1.80kg

=−2.649 N·m

~Fw

34cm

τw =−0.340m×9.81 Nkg ×7.20kg

=−24.015 N·m

This excludes the torque due to the force at the pivot (~FJ), which is just 0 N·m.

Step 4: With the arm in static equilibrium, the torques acting on the arm must sum to zero:

τM +τG +τw = 0 N·m (2.24)

+0.045m×FM −2.649 N·m−24.015 N·m= 0 N·m (2.25)

(As noted previously, there is no contribution to torque from the force at the joint since the moment arm of that forceis zero.) All horizontal components of force are zero, identically. All vertical components of force must sum to zero:

FJ,y +FM,y +FG,y +Fw,y = 0 N (2.26)

FJ,y +FM −17.658 N−70.632 N= 0 N (2.27)

Step 5: Solving the sum-of-torques equation gives us that FM = 593 N, which points upwards (the direction was given).Using that result to solve the sum-of-forces equation gives us that the vertical component of the force at the joint isFJ,y =−504 N.

Answer: The magnitude of the force exerted by the muscles is 593 N. The force exerted at the joint is 504 N, down-wards.

Why is the force at the joint downwards?

Since the muscles exert a force much closer to the joint than gravity or the weight, and its torque must balance, itwill be a much larger force than either. To balance this large upwards force, there must be a similarly large downwardsforce at the joint.

Exercises for PPT α10 November 28, 2019 62 Ch.2 Torques

Page 69: Solutions to Exercises for Physics for Physiotherapy ... - Solutions.pdf · SOLUTIONSto Exercises for Physics for Physiotherapy Technology 203-946-DW Fall 2019 by Chris Roderick,

EXERCISE 2.3.17

A person is pressing downwards with their hand, exerting 107 N on a surface (not shown). What total amount of forcemust the muscles of their upper arm be exerting on their lower arm? (The active muscles are the cluster referred to asthe triceps, which are the muscles on the rear side of the upper arm.) The forearm (including the hand) has a mass of1.8kg, and has its center of mass 15cm from the elbow joint.

0cm1.2cm 34cm

107N

Step 0: The object is the person’s forearm including the hand.

Step 1: The object is interacting with the following things: The Earth exerts a gravitational pull on the forearm. Theis pressing downwards on a surface, and the reaction to that is the surface pushing upwards against the person. Themuscles of the upper arm are pulling upwards on the forearm, which bring the bones into contact with each other.

Step 2: For the purposes of finding the sum of forces and torques, the Free-Body Diagram for a complex system likethis portion of the human body can be draw schematically (as a cartoon):

~FM ~FJ

~FG ~n

With all the forces being vertical, two of which are unknown, we can not (yet) construct a check of the sum of forces. Itis our guess that the contact force at the joint (~FJ) will be pointed downwards.

Step 3: All the forces acting in the situation are vertical, so there are no x-components. The y-components are

FG,y =−9.81 Nkg ×1.80kg=−17.658 N (2.28)

ny =+107 N (2.29)

(The y-components of ~FJ and ~FM are unknowns to be solved.)

Exercises for PPT α10 November 28, 2019 63 Ch.2 Torques

Page 70: Solutions to Exercises for Physics for Physiotherapy ... - Solutions.pdf · SOLUTIONSto Exercises for Physics for Physiotherapy Technology 203-946-DW Fall 2019 by Chris Roderick,

The contributions from these forces to the torque about the pivot are:

~FM

1.2cm

τM =−0.012m×FM

~FG

15.0cm

τG =−0.150m×9.81 Nkg ×1.80kg

=−2.649 N·m

~n34cm

τn =+0.340m×107N

=+36.380 N·m

This excludes the torque due to the force at the pivot (~FJ), which is just 0 N·m.

Step 4: With the arm in static equilibrium, the torques acting on the arm must sum to zero:

τM +τG +τn = 0 N·m (2.30)

−0.012m×FM −2.649 N·m+36.380 N·m= 0 N·m (2.31)

(As noted previously, there is no contribution to torque from the force at the joint since the moment arm of that forceis zero.) All horizontal components of force are zero, identically. All vertical components of force must sum to zero:

FM,y +FJ,y +FG,y +ny = 0 N (2.32)

+FM +FJ,y −17.658 N+107 N= 0 N (2.33)

Step 5: Solving the sum-of-torques equation gives us that FM = 2811 N, which points upwards (the direction wasgiven). Using that result to solve the sum-of-forces equation gives us that the vertical component of the force at thejoint is FJ,y =−2900 N.

Answer: The magnitude of the force exerted by the muscles is 2811 N. The force exerted at the joint is 2900 N,downwards.

Why is the force at the joint downwards?

Since the muscles exert a force much closer to the joint than the normal, and its torque must balance, it will be amuch larger force. To balance this large upwards force, there must be a similarly large downwards force at the joint.

We pause to note that the forces exerted by the muscle and exerted in the joint are almost thirty times larger thanthose exerted by gravity or exerted at the extremity of the limb. This is a typical situation!

Exercises for PPT α10 November 28, 2019 64 Ch.2 Torques

Page 71: Solutions to Exercises for Physics for Physiotherapy ... - Solutions.pdf · SOLUTIONSto Exercises for Physics for Physiotherapy Technology 203-946-DW Fall 2019 by Chris Roderick,

EXERCISE 2.3.18

A weightlifter is holding a weight above their head. Each arm supports 421N. The muscles of the shoulder collectivelyexert a force along a line that is 20 above the axis of the bone of the upper arm (the humerus). What is the magnitudeof that collective force? (We will neglect the mass of the arm itself in this.)

0cm 13cm 28cm

421N

Step 0: The object is person’s entire arm, including the hand.

Step 1: The object is interacting with the following things: The Earth exerts a gravitational pull on the arm, but weare told to ignore that contribution (presumably because it will be much smaller than the other forces in the situation).The person is holding the weights above their head, so the weights push downwards on the person’s hand. The musclesin the shoulder (primarily the deltoids) pull on the arm at an angle, pressing the bones of the arm (specifically thehumerus) into contact with the bones of the shoulder (specifically the scapula).

Step 2: For the purposes of finding the sum of forces and torques, the Free-Body Diagram for a complex system likethis portion of the human body can be draw schematically (as a cartoon):

~FM

~FJ

~Fw

~Fw

~FJ

~FM

The force at the joint may have to point slightly downwards if the force due to the muscle is very large in comparisonto the weight being supported.

Exercises for PPT α10 November 28, 2019 65 Ch.2 Torques

Page 72: Solutions to Exercises for Physics for Physiotherapy ... - Solutions.pdf · SOLUTIONSto Exercises for Physics for Physiotherapy Technology 203-946-DW Fall 2019 by Chris Roderick,

Step 3: The components of the forces acting are

x

y

~Fw

Fw,x = 0 N

Fw,y =−421 N

x

y~FM

20160

FM,x = FM cos160

FM,y = FM sin160

While we expect that the force at the joint will point towards the right, its components remain as unknowns that wemust solve for.

The contributions to torque from the forces acting on the object are:

~Fw

0.28 m

τw =−(421N)(0.28m)

=−117.9 N·m

~FM

0.13 m

160

τM =+(FM)(0.13m)sin160

=+FM ×0.04446m

Notice how, in the case of the force due to the weight, we were able to use the line of action to find the moment armdirectly, while, in the case of the force due to the muscle, we needed to find the distance to point where the force wasacting and the angle to the line of action.

As is usual, there is no contribution to torque from the force exerted at the joint.

Step 4: Since the arm is in static equilibrium the sum of forces must be zero:

~FJ +~FM +~Fw =~0 N (2.34)

The x-component of the sum is

FJ,x +FM,x +Fw,x = 0 N (2.35)

FJ,x +FM cos160+0 N= 0 N (2.36)

The y-component of the sum is

FJ,y +FM,y +Fw,y = 0 N (2.37)

FJ,y +FM sin160−421 N= 0 N (2.38)

The sum of torques must also be zero:

τM +τw = 0 N·m (2.39)

+FM ×0.04446m−117.9 N·m= 0 N·m (2.40)

Step 5: Solving the sum-of-torques equation gives us FM = 2651 N. Using this in the sum-of-forces equations gives

FJ,x =+2491 N (2.41)

FJ,y =−486 N (2.42)

Answer: The magnitude of the force exerted by the muscles has to be FM = 2651 N. (As expected, since the muscleacts at a point much closer to the joint, the force applied by the muscle is much greater in magnitude than thatof the weight.) The force at the joint has magnitude FJ = 2538 N, and points towards the right and 11 below the

Exercises for PPT α10 November 28, 2019 66 Ch.2 Torques

Page 73: Solutions to Exercises for Physics for Physiotherapy ... - Solutions.pdf · SOLUTIONSto Exercises for Physics for Physiotherapy Technology 203-946-DW Fall 2019 by Chris Roderick,

horizontal. (As noted when we checked the sum-of-forces, because the force exerted by the muscles is so large thecontact force between the bones at the joint must point slightly downwards to counter the large upwards componentfrom the muscles.)

We can end with a note that the weight of the arm (which would have been about 20 N) would have contributed afew percent to the forces already present. Ignoring the weight did not affect the results significantly.

Exercises for PPT α10 November 28, 2019 67 Ch.2 Torques

Page 74: Solutions to Exercises for Physics for Physiotherapy ... - Solutions.pdf · SOLUTIONSto Exercises for Physics for Physiotherapy Technology 203-946-DW Fall 2019 by Chris Roderick,

EXERCISE 2.3.19

A person is doing a pull-up. Each arm support half their mass (total mass 70kg). The pectorals and the muscles of theback collectively exert a force along a line that is 40 to the left of the vertical. What is the magnitude of that collectiveforce? (We will neglect the mass of the arm itself in this.)

0cm 5.2cm

28cm

Step 0: The object is person’s entire arm, including the hand.

Step 1: The object is interacting with the following things: The Earth exerts a gravitational pull on the arm, but weare told to ignore that contribution (presumably because it will be much smaller than the other forces in the situation).The person is holding their own weight by their hands, so the bar they are holding must be pulling upwards on theirhands. The muscles of the torso (primarily the pectorals on the front, and the latissimus dorsi on the back) pull on thearm downwards at an angle, pressing the bones at the joint into contact.

Step 2: For the purposes of finding the sum of forces and torques, the Free-Body Diagram for a complex system likethis portion of the human body can be draw schematically (as a cartoon):

~FM

~FJ

~FH

~FJ

~FH

~FM

Step 3: The components of the forces acting are

x

y~FH

FH,x = 0 N

FH,y = ( 12 ×70kg)(9.81 N

kg )=+343.4 N

x

y

~FM90 +40

230

FM,x = FM cos230

FM,y = FM sin230

The components of the force at the joint remain as unknowns that we must solve for.

The contributions to torque from the forces acting on the object are:

Exercises for PPT α10 November 28, 2019 68 Ch.2 Torques

Page 75: Solutions to Exercises for Physics for Physiotherapy ... - Solutions.pdf · SOLUTIONSto Exercises for Physics for Physiotherapy Technology 203-946-DW Fall 2019 by Chris Roderick,

~FH

0.28 m

τH =+(343.35N)(0.28m)

=+96.15 N·m

~FM

0.052 m

130

τM =−(FM)(0.052m)sin130

=−FM ×0.03983m

As is usual, there is no contribution to torque from the force exerted at the joint.

Step 4: Since the arm is in static equilibrium the sum of forces must be zero:

~FJ +~FM +~FH =~0 N (2.43)

The x-component of the sum is

FJ,x +FM,x +FH,x = 0 N (2.44)

FJ,x +FM cos230+0 N= 0 N (2.45)

The y-component of the sum is

FJ,y +FM,y +FH,y = 0 N (2.46)

FJ,y +FM sin230+343.4 N= 0 N (2.47)

The sum of torques must also be zero:

τM +τH = 0 N·m (2.48)

−FM ×0.03983m+96.15 N·m= 0 N·m (2.49)

Step 5: Solving the sum-of-torques equation gives us FM = 2414 N. Using this in the sum-of-forces equations gives

FJ,x =+1552 N (2.50)

FJ,y =+1506 N (2.51)

Answer: The magnitude of the force exerted by the muscles has to be FM = 2414 N. The force at the joint hasmagnitude FJ = 2163 N, and points towards the right and 44 above the horizontal. As always the forces generated atand around the joint are much larger than the forces at the extremities that are their cause.

Exercises for PPT α10 November 28, 2019 69 Ch.2 Torques

Page 76: Solutions to Exercises for Physics for Physiotherapy ... - Solutions.pdf · SOLUTIONSto Exercises for Physics for Physiotherapy Technology 203-946-DW Fall 2019 by Chris Roderick,

EXERCISE 2.3.20

A person doing a push-up supports 180N at each hand. The pectoral muscle (the big muscle on the chest) attaches atan angle of 30 to the axis of the bone of the upper arm (the humerus). What is the magnitude of the force exerted bythe pectoral? (We will neglect the mass of the arm in this.)

0cm5.0cm28cm

180N

Step 0: The object is person’s entire arm, including the hand.

Step 1: The object is interacting with the following things: The Earth exerts a gravitational pull on the arm, but weare told to ignore that contribution (presumably because it will be much smaller than the other forces in the situation).The person is touching the surface, supporting a portion of their own weight by their hands (the upwards force of180N). The muscles on the chest (the pectorals) pull on the arm downwards at an angle, pressing the bones at the jointinto contact.

Step 2: For the purposes of finding the sum of forces and torques, the Free-Body Diagram for a complex system likethis portion of the human body can be draw schematically (as a cartoon):

~FM

~FJ

~n ~FM

~n

~FJ

Step 3: The components of the forces acting are

x

y~FH

nx = 0 N

ny =+180 N

x

y

~FM

330

30

FM,x = FM cos330

FM,y = FM sin330

The components of the force at the joint remain as unknowns that we must solve for.

The contributions to torque from the forces acting on the object are:

Exercises for PPT α10 November 28, 2019 70 Ch.2 Torques

Page 77: Solutions to Exercises for Physics for Physiotherapy ... - Solutions.pdf · SOLUTIONSto Exercises for Physics for Physiotherapy Technology 203-946-DW Fall 2019 by Chris Roderick,

~n

0.28 m

τn =−(180N)(0.28m)

=−50.40 N·m

~FM

0.050 m

150

τM =+(FM)(0.050m)sin150

=+FM ×0.0250m

As is usual, there is no contribution to torque from the force exerted at the joint.

Step 4: Since the arm is in static equilibrium the sum of forces must be zero:

~FJ +~FM +~n =~0 N (2.52)

The x-component of the sum is

FJ,x +FM,x +nx = 0 N (2.53)

FJ,x +FM cos330+0 N= 0 N (2.54)

The y-component of the sum is

FJ,y +FM,y +ny = 0 N (2.55)

FJ,y +FM sin330+180 N= 0 N (2.56)

The sum of torques must also be zero:

τM +τn = 0 N·m (2.57)

+FM ×0.0250m−50.40 N·m= 0 N·m (2.58)

Step 5: Solving the sum-of-torques equation gives us FM = 2016 N. Using this in the sum-of-forces equations gives

FJ,x =−1746 N (2.59)

FJ,y =+828 N (2.60)

Answer: The magnitude of the force exerted by the muscles has to be FM = 2016 N. The force at the joint hasmagnitude FJ = 1932 N, and points upwards and to the left at 155 counter-clockwise from the +x-axis.

Exercises for PPT α10 November 28, 2019 71 Ch.2 Torques

Page 78: Solutions to Exercises for Physics for Physiotherapy ... - Solutions.pdf · SOLUTIONSto Exercises for Physics for Physiotherapy Technology 203-946-DW Fall 2019 by Chris Roderick,

EXERCISE 2.3.21

A person doing a push-up supports 142N at each shoulder joint. The triceps muscles (the muscles on the back of theupper arm) attaches at an angle of 5 to the axis of the bone of the upper arm (the humerus). What is the magnitudeof the force exerted by the triceps? (We will neglect the mass of the arm in this.)

28cm21cm0cm

142N

Step 0: The object is person’s upper arm, from the elbow to the shoulder.

Step 1: The object is interacting with the following things: The Earth exerts a gravitational pull on the arm, but weare told to ignore that contribution (presumably because it will be much smaller than the other forces in the situation).The muscles on the back of the upper arm (the triceps) pull on the arm towards the left at a very small angle angle,pressing the bones at the elbow joint into contact. The connection at the shoulder joint is transferring a portion of theperson’s weight (the 142N force) to the object.

Step 2: For the purposes of finding the sum of forces and torques, the Free-Body Diagram for a complex system likethis portion of the human body can be draw schematically (as a cartoon):

~FJ

~FM

~Fs

~Fs

~FJ

~FM

Step 3: The components of the forces acting are

x

y

~Fs

Fs,x = 0 N

Fs,y =−142 N

x

y

~FM175

FM,x = FM cos175

FM,y = FM sin175

The components of the force at the joint remain as unknowns that we must solve for.

Exercises for PPT α10 November 28, 2019 72 Ch.2 Torques

Page 79: Solutions to Exercises for Physics for Physiotherapy ... - Solutions.pdf · SOLUTIONSto Exercises for Physics for Physiotherapy Technology 203-946-DW Fall 2019 by Chris Roderick,

The contributions to torque from the forces acting on the object are:

~Fs

28cm

τs =−(142N)(0.280m)

=−39.76 N·m

~FM

21cm

τM =+(FM)(0.210m)sin175

=+FM ×0.01830m

As is usual, there is no contribution to torque from the force exerted at the joint.

Step 4: Since the arm is in static equilibrium the sum of forces must be zero:

~FJ +~FM +~Fs =~0 N (2.61)

The x-component of the sum is

FJ,x +FM,x +Fs,x = 0 N (2.62)

FJ,x +FM cos175+0 N= 0 N (2.63)

The y-component of the sum is

FJ,y +FM,y +Fs,x = 0 N (2.64)

FJ,y +FM sin175−142 N= 0 N (2.65)

The sum of torques must also be zero:

τM +τs = 0 N·m (2.66)

+FM ×0.01830m−39.76 N·m= 0 N·m (2.67)

Step 5: Solving the sum-of-torques equation gives us FM = 2173 N. Using this in the sum-of-forces equations gives

FJ,x =+2164 N (2.68)

FJ,y =−47 N (2.69)

Answer: The magnitude of the force exerted by the muscles has to be FM = 2173 N. The force at the joint hasmagnitude FJ = 2165 N, and points almost directly towards the right (only 1 below the +x-axis).

Exercises for PPT α10 November 28, 2019 73 Ch.2 Torques

Page 80: Solutions to Exercises for Physics for Physiotherapy ... - Solutions.pdf · SOLUTIONSto Exercises for Physics for Physiotherapy Technology 203-946-DW Fall 2019 by Chris Roderick,

EXERCISE 2.3.22

A person is holding a rope (not shown). The reaction force to their pull is the 64N force on their hand, perpendicularto their arm. The weight of their arm is 34N (center of mass 21cm from the shoulder). If their arm is 37 from thevertical, what force must their pectoral (chest muscle) exert horizontally?

0cm

5cm

21cm

56cm

34N

64N

Step 0: The object is the person’s whole arm.

Step 1: The object is interacting with the following things: The Earth exerts a gravitational pull vertically downwardson the arm. The person is pulling on the rope (the action), and so the rope pulls on their fist (the reaction) upwardsand to the right. The pectorals pull towards the left to balance the torque due to the force at the first. This presses thebones into contact in the shoulder joint.

Step 2: For the purposes of finding the sum of forces and torques, the Free-Body Diagram for a complex system likethis portion of the human body can be draw schematically (as a cartoon):

~FJ

~FM

~FG

~P ~FG

~FM

~P

~FJ

Step 3: The components of the forces acting are

x

y

~FG

FG,x = 0 N

FG,y =−34 N

x

y

~FM

FM,x =−FM

FM,y = 0 N

x

y~P

37

Px = 64 N cos37 =+51.11 N

Py = 64 N sin37 =+38.52 N

The components of the force at the joint remain as unknowns that we must solve for.

Exercises for PPT α10 November 28, 2019 74 Ch.2 Torques

Page 81: Solutions to Exercises for Physics for Physiotherapy ... - Solutions.pdf · SOLUTIONSto Exercises for Physics for Physiotherapy Technology 203-946-DW Fall 2019 by Chris Roderick,

The contributions to torque from the forces acting on the object are:

~FG

21cm

−37

τG = (34N)(0.210m) sin(−37)=−4.297 N·m

~FM

5cm

−127

τM =+(FM)(0.050m) sin(−127)=−FM ×0.040m

~P

56cm

τP =+(64 N)(0.560m)

=+35.84 N·m

As is usual, there is no contribution to torque from the force exerted at the joint.

Step 4: Since the arm is in static equilibrium the sum of forces must be zero:

~FG +~FM +~P +~FJ =~0 N (2.70)

The x-component of the sum is

FG,x +FM,x +Px +FJ,x = 0 N (2.71)

0 N−FM +51.11 N+FJ,x = 0 N (2.72)

The y-component of the sum is

FG,y +FM,y +Py +FJ,y = 0 N (2.73)

−34.00 N+0 N+38.52 N+FJ,y = 0 N (2.74)

The sum of torques must also be zero:

τG +τM +τP +τJ = 0 N·m (2.75)

−4.297 N·m−FM ×0.040m+35.84 N·m+0 N·m= 0 N·m (2.76)

Step 5: Solving the sum-of-torques equation gives us FM = 790 N. Using this in the sum-of-forces equations gives

FJ,x =+739 N (2.77)

FJ,y =−5 N (2.78)

Answer: The magnitude of the force exerted by the muscles has to be FM = 790 N. The force at the joint has magnitudeFJ = 739 N, and points almost directly towards the right (less than 1 below the +x-axis).

Exercises for PPT α10 November 28, 2019 75 Ch.2 Torques

Page 82: Solutions to Exercises for Physics for Physiotherapy ... - Solutions.pdf · SOLUTIONSto Exercises for Physics for Physiotherapy Technology 203-946-DW Fall 2019 by Chris Roderick,

EXERCISE 2.3.23

A person is lifting a 137N weight out to their side. The weight of their arm is 34N (center of mass 21cm from theshoulder). If their arm is 37 from the vertical, what force must their deltoid (shoulder muscle) exert? (The deltoidattaches to the humerus 7.0cm from the shoulder joint at an angle of 22.)

0cm

7cm

21cm

56cm

34N

137N

Step 0: The object is the person’s whole arm.

Step 1: The object is interacting with the following things: The Earth exerts a gravitational pull vertically downwardson the arm. The person is pulling upwards on the weight (the action), and so the weight pulls downwards on their fist(the reaction). The deltiods pull upwards to balance the torque due to the force at the first. This presses the bones intocontact in the shoulder joint.

Step 2: Step 3: Step 4: Step 5: Answer:

Exercises for PPT α10 November 28, 2019 76 Ch.2 Torques

Page 83: Solutions to Exercises for Physics for Physiotherapy ... - Solutions.pdf · SOLUTIONSto Exercises for Physics for Physiotherapy Technology 203-946-DW Fall 2019 by Chris Roderick,

EXERCISE 2.3.24

A woman doing core exercises is in “plank position”, as shown in the diagram below. The joint where the spine meetsthe pelvis is the pivot, with the legs on one side and the upper body on the other. It is the abdominal muscles thatkeep the two segments from bending away from each other. The contact force at her feet is 230.0 N, and the mass ofthe lower segment of her body is 30.61 kg. What is the tension in her abdominal muscles that keeps her body in staticequilibrium?

Step 0: The object is the lower portion of the person’s body.

Step 1: The object is interacting with the following things: The Earth exerts a gravitational pull vertically downwardson this portion of their body. Their feet are touching the floor, so there is a normal force there. The abdominals pulltowards the right to balance the torques due to the other forces. This presses the pelvis into contact with the lumbarvertebrae.

Step 2: Using a very crude cartoon of the object (depicting the lower half of their body by a rectangle), the Free-BodyDiagram is:

Step 3: The components of the forces and the contributing torques are as follows:

~n

pivot

70 cm

τnormal =−230.0 N×0.70 m

~Fg

pivot

32 cm

τgravity =+300.0 N×0.32 m

~T

pivot

18 cm

τabs =+T ×0.18 m

Step 4: There is an unknown force (~FP) acting at the pivot. The forces acting on this lower segment of the body mustsum to zero:

nx +Fg,x +Tx +FP,x = 0 N ny +Fg,y +Ty +FP,y = 0 N (2.79)

(0 N)+ (0 N)+Tx +FP,x = 0 N (+230.0 N)+ (−300.0 N)+ (0 N)+FP,y = 0 N (2.80)

The sum of torques about the pivot is zero:

τnormal +τgravity +τabs = 0 N ·m (2.81)

(−230.0 N×0.70 m)+ (+300.0 N×0.32 m)+ (+T ×0.18 m)= 0 N ·m (2.82)

Step 5: In the sum of forces equations there are three unknowns: the tension and both components of the force at thepivot. Thus we must first solve for the tension. Using the sum of torques we can solve for the unknown tension in theabdominal muscles:

(−230.0 N×0.70 m)+ (+300.0 N×0.32 m)+ (+T ×0.18 m)= 0 N ·m (2.83)

T = 361.1 N (2.84)

With that value in the sum of forces equations, we can now solve for the unknown force acting at the pivot:

(0 N)+ (0 N)+ (+361.1 N)+FP,x = 0 N (+230.0 N)+ (−300.0 N)+ (0 N)+FP,y = 0 N (2.85)

FP,x =−361.1 N FP,y =+70.0 N (2.86)

Exercises for PPT α10 November 28, 2019 77 Ch.2 Torques

Page 84: Solutions to Exercises for Physics for Physiotherapy ... - Solutions.pdf · SOLUTIONSto Exercises for Physics for Physiotherapy Technology 203-946-DW Fall 2019 by Chris Roderick,

The magnitude of this force is FP =√

(−361.1 N)2 + (+70.0 N)2 = 367.8 N.

From the signs of the components of this force, we see that it points into the second quadrant (90 < θ < 180).Using the inverse tangent, our calculator gives us

θ = tan−1(FP,y

FP,x

)= tan−1

( +70.0 N−361.1 N

)=−11 (2.87)

which points into the fourth quadrant. The correct angle is θ = 180−11 = 169.

Answer: The tension in her abdominal muscles is 361 N. The force at the joint points towards her feet (to the left) andslightly upwards (169) with a magnitude of 368 N.

Exercises for PPT α10 November 28, 2019 78 Ch.2 Torques

Page 85: Solutions to Exercises for Physics for Physiotherapy ... - Solutions.pdf · SOLUTIONSto Exercises for Physics for Physiotherapy Technology 203-946-DW Fall 2019 by Chris Roderick,

EXERCISE 2.3.25

A person is standing vertically. Using their hamstrings (three muscles, of which the largest is the biceps femoris) theyare holding the lower portion of one of their legs (3.00kg) horizontal, as shown. The muscles are attached 4.50cm fromthe pivot (the knee joint), and the center of mass of this segment is 15.3cm from the joint. What is the tension in thehamstrings?

0cm4.50cm

Answer: The tension in their hamstrings is 100 N.

EXERCISE 2.3.26

A vertical person is holding one of their legs horizontally, as shown. One of the major flexors, the iliopsoas, connectsthe lesser trochanter (a small bump on the femur) to portions of the pelvis and the lumbar region of the spine. Thismuscle exerts a net force pointed at 57 above the horizontal (as shown), acting 5.8 cm from the hip joint. The wholeleg has a mass of 10.40 kg, and the center of mass is 29.2 cm from the hip joint. What is the tension in the muscle?

0cm 5.8cm

Answer: The tension in their iliopsoas is 612 N.

EXERCISE 2.3.27

A vertical person with one of their legs as shown, is holding their lower leg at an angle of 30 from the vertical. Thissegment has a mass of 2.74 kg, with its center of mass 14.8 cm from the knee joint. The patellar ligament (the ligamentthat connects the patella [kneecap] to the tibia [shinbone]) attaches to the tibia at a point 7.7 cm from the joint, and

Exercises for PPT α10 November 28, 2019 79 Ch.2 Torques

Page 86: Solutions to Exercises for Physics for Physiotherapy ... - Solutions.pdf · SOLUTIONSto Exercises for Physics for Physiotherapy Technology 203-946-DW Fall 2019 by Chris Roderick,

the angle between the ligament and the tibia is 19. What is the tension in the ligament?

0cm

7.7cm

Answer: The tension in their patellar ligament is 79.3 N.

Exercises for PPT α10 November 28, 2019 80 Ch.2 Torques

Page 87: Solutions to Exercises for Physics for Physiotherapy ... - Solutions.pdf · SOLUTIONSto Exercises for Physics for Physiotherapy Technology 203-946-DW Fall 2019 by Chris Roderick,

Chapter 3

MaterialsRemember that the symbol σ means “stress” (the way in which force is distributed across the cross-section of the objectσ = F

/A) and that the symbol ε means “strain” (the relative change in length due to the applied stress ε = ∆L

/Li).

Unless otherwise explicitly stated all the stresses are axial tension or axial compression.

In exercises involving circular geometry remember to find the radius in cases when the diameter is specified. Whenthe cross-section is hollow, obtain the area for the stress by subtracting the area of the hollow from the area of thecross-section.

Since strain is the change in length divided by the original length (ε=∆L/

L = (Lf −Li)/

Li), the new length of thedeformed object is

Lf = Li +∆L (3.1)

Lf = Li +εLi (3.2)

Note also that since strain is a ratio of lengths, the units used do not matter, as long as they are the same.

In your calculations: be very careful with the units and powers of ten; keep at least four significant figures in yourintermediate steps; and round only the final result.

3.1 Stress

EXR 3.1.01 A 5.0 N force is applied across a rectangular surface measuring 2.0 cm by 7.0 cm. The stress is σ =3.6 kPa.The cross-sectional area is A = (0.020 m)× (0.070 m) = 1.4× 10−3 m2. The stress is thus σ = F

/A = (5.0 N)

/(1.4×

10−3 m2)= 3.571×10+3 Pa= 3.6 kPa, written to two significant figures.

EXR 3.1.02 A 1060 N force is applied across a rectangular surface measuring 10 cm by 30 cm. The stress is σ =35 kPa.The cross-sectional area is A = (0.10 m)× (0.30 m) = 3.0× 10−2 m2. The stress is thus σ = F

/A = (1060 N)

/(3.0×

10−2 m2)= 3.533×10+4 Pa= 35 kPa, written to two significant figures.

EXR 3.1.03 A 120 kg mass is resting on a rectangular surface measuring 8.0 cm by 27.0 cm. The pressure is P= 55 kPa. Recall that pressure on a surface is just another name for compressive stress. The force applied is theweight: F = mg = (120 kg)(9.81 N/kg)= 1177.2 N. (Remember: do not round your intermediate calculations!) The cross-sectional area is A = (0.080 m)× (0.270 m)= 2.16×10−2 m2. The stress is thus σ= F

/A = (1177.2 N)

/(2.16×10−2 m2)=

5.450×10+4 Pa= 55 kPa, written to two significant figures.

EXR 3.1.04 A force of 33.7 N is shearing across a rectangular surface measuring 5.0 mm by 8.0 mm. The shearstress is σ = 0.84 MPa.The cross-sectional area is A = (0.0050 m)× (0.0080 m) = 4.0×10−5 m2. The stress is thus σ = F

/A = (33.7 N)

/(4.0×

10−5 m2)= 8.425×10+5 Pa= 0.84 MPa, written to two significant figures.

EXR 3.1.05 A 7.16 kg mass is suspended at the end of a horizontal metal rod that is square in cross-section (width1.00 cm). The shear stress is σ = 0.702 MPa.

EXR 3.1.06 A cylindrical rod of diameter 5.00 cm is under 12.0 N of compression. The stress is σ = 6.11 kPa.

EXR 3.1.07 A 621 kg mass hangs at the end of a 12.7 mm diameter metal rod. The stress is σ = 48 MPa.

EXR 3.1.08 A 7.00 kg mass hangs at the end of a cylindrical rod of diameter 6.0 mm. The stress is σ = 2.4 MPa.

EXR 3.1.09 A 7.16 kg mass is suspended at the end of a horizontal metal rod that is circular in cross-section(diameter 1.00 cm). The shear stress is σ = 0.894 MPa.

EXR 3.1.10 A hollow square beam supports 12 507 N of weight. The outer width of the beam is 42.7 mm and theinner width of the hollow is 40.1 mm. The stress in the material of the beam is σ = 58.1 MPa.

Exercises for PPT α10 November 28, 2019 81 Ch.3 Materials

Page 88: Solutions to Exercises for Physics for Physiotherapy ... - Solutions.pdf · SOLUTIONSto Exercises for Physics for Physiotherapy Technology 203-946-DW Fall 2019 by Chris Roderick,

EXR 3.1.11 A 18.755 kg bowling ball rests on the end of a hollow cylinder with outer radius 3.30 cm and inner radius3.20 cm. The stress in the material of the cylinder is σ = 0.901 MPa.

EXR 3.1.12 A hollow metal tube with outer diameter 3.81 mm and inner diameter 3.77 mm is under a tension of522 N. The stress in the material of the tube is σ = 2.19 GPa.

EXR 3.1.13 A 1406 kg car rests on its four tires, each with a contact patch measuring 19 cm by 24 cm. The pressureon each tire is P = 76 kPa.

EXR 3.1.14 A 54.4 kg ballerina stands on one foot (equivalent to a rectangle measuring 20.3 cm by 5.1 cm). Thepressure under their foot is P = 52 kPa.

EXR 3.1.15 A 54.4 kg ballerina stands on the point of her toes of one foot (equivalent to a rectangle measuring6.0 cm by 2.0 cm). The pressure under their toes is P = 0.44 MPa.

EXR 3.1.16 An adult male moose can have masses up to 700 kg! Their femur is a hollow cylinder of bone, outerdiameter 5.5 cm, inner diameter 3.5 cm. The stress in the bone is σ = 1.2 MPa. (Assume that its weight is distributedequally onto each leg.)

3.2 Strain

EXR 3.2.01 A meter stick (length 1.000 m) is subjected to tension and stretches to 1.003 m. The strain is ε =+3×10−3.The amount of length change is ∆L = Lf −Li = 1.003 m−1.000 m =+0.003 m. Notice how there is only one significantfigure here. This means that the strain is ε= ∆L

Li= +0.003 m

1.000 m =+0.003=+3×10−3, to one significant figure.

EXR 3.2.02 An elastic band is stretched from 8.5 cm to 14.2 cm. The strain is ε = +0.67.The amount of length change is ∆L = Lf −Li = 14.2 cm−8.5 cm = +5.7 cm. This means that the strain is ε = ∆L

Li=

+5.7 cm8.5 cm = +0.670588 . . . = +0.67, to two significant figures. Notice how large this is in comparison to the values found

in the other exercises that involve solid objects. Large strains like this are only possible in materials like rubbers andother polymers.

EXR 3.2.03 An object of length 45.00 cm is subjected to a stress and compresses to 44.92 cm. The strain is ε =−2×10−3.The amount of length change is ∆L = Lf−Li = 44.92 cm−45.00 cm=−0.08 cm. Notice how there is only one significantfigure here. This means that the strain is ε = ∆L

Li= −0.08 cm

45.00 cm = −0.001777 . . . = −0.002 = −2×10−3, to one significantfigure.

EXR 3.2.04 A steel ruler (length 12.02 in) is subjected to tension and stretches to 12.03 in. The strain is ε =+8.319×10−4.

EXR 3.2.05 A wooden column of height 67.02 inches is subjected to a stress and compresses to 66.97 inches. Thestrain is ε = −7.460×10−4.

EXR 3.2.06 A ruler (original length 30.22 cm) is subjected to tension and exhibits a strain of ε = 2.0×10−3. Thedeformed length is Lf = 30.28 cm.

EXR 3.2.07 A segment of a bridge (original length 27.052 m) is subjected to a stress and exhibits a strain of ε =+3.5×10−5. The deformed length is Lf = 27.053 m.

EXR 3.2.08 A segment of a bridge (original length 27.052 m) is subjected to a stress and exhibits a strain of ε =−3.5×10−5. The deformed length is Lf = 27.051 m.

EXR 3.2.09 When the ground under a building subsides a concrete wall (original height 13.09 m) is subjected to astress and exhibits a strain of ε=+7.30×10−6. The deformed height is Lf = 13.09 m.The change in height is ∆L = εLi = (+7.30×10−6)(13.09 m) = +95.6×10−6 m. The final height is thus Lf = Li +∆L =(13.09 m)+ (+95.6×10−6 m) = 13.0900956 m = 13.09 m, to four significant figures. The strain is not zero, and is mea-surable with sophisticated equipment, but is not significant on the human scale.

EXR 3.2.10 A person out on a nature hike takes a rest on a granite boulder (original height 75.00 cm). The com-pressive stress due to the person’s weight causes a strain of ε=−5.28×10−8 in the boulder. The deformed height is Lf= 75.00 cm. As with the previous exercise, the deformation is utterly negligible in comparison to the original size.

Exercises for PPT α10 November 28, 2019 82 Ch.3 Materials

Page 89: Solutions to Exercises for Physics for Physiotherapy ... - Solutions.pdf · SOLUTIONSto Exercises for Physics for Physiotherapy Technology 203-946-DW Fall 2019 by Chris Roderick,

3.3 Stress-Strain Curves

In each of the stress-strain graphs of the following exercises a small orange circle around the end of a stress-straincurve indicates the failure of the material.

EXERCISE 3.3.01 Consider the stress-strain curves plotted below.

(a) Calculate the Young’s Modulus for each of the materials. (Be careful with the units!)

(b) For those materials that have a failure in the plotted range of strains, estimate the stress and strain at failure. Forthose graphs also estimate their values at the yield point (if it exists).

[1]

ε(10−3)

σ(kPa)

10

50

0 2 4 6 8

0

10

20

30

40

[2]

ε(10−3)

σ(kPa)

10

50

0 2 4 6 8

0

10

20

30

40

[3]

ε(10−3)

σ(kPa)

10

50

0 2 4 6 8

0

10

20

30

40

[4]

ε(10−6)

σ(kPa)

10

50

0 2 4 6 8

0

10

20

30

40

[5]

ε(10−3)

σ(MPa)

75

100

0 15 30 45 60

0

20

40

60

80

[6]

ε(10−3)

σ(kPa)

20

20

0 4 8 12 16

0

4

8

12

16

[7]

ε(10−6)

σ(Pa)

250

50

0 50 100 150 200

0

10

20

30

40

[8]

ε(%)

σ(MPa)

5

250

0 1 2 3 4

0

50

100

150

200

[9]

ε(10−3)

σ(kPa)

100

125

0 20 40 60 80

0

25

50

75

100

General comments:

Part (a): Since the proportional regime of each of the stress-strain curves here begins at the origin, the Young’sModulus for each can be found by picking one point on its curve and finding the ratio: Y =∆σ/

∆ε= (σ−0Pa)/

(ε−0)=σ

/ε.

Part (b): Only the curves [1], [2] and [4] do not have the failure point of their materials plotted. Those curves alsodo not show their yield point.

[1] (a) Picking the point σ= 30 kPa and ε= 10×10−3, we getY = (30×10+3 Pa)

/(10×10−3)= 3.0×10+6 Pa= 3.0 MPa.

(b) (No answer.)

[2] (a) Picking the point σ= 30 kPa and ε= 4×10−3, we getY = (30×10+3 Pa)

/(4×10−3)= 7.5×10+6 Pa= 7.5 MPa.

(b) (No answer.)

[3] (a) Picking the point σ= 40 kPa and ε= 6×10−3, we getY = (40×10+3 Pa)

/(6×10−3)= 6.7×10+6 Pa= 6.7 MPa.

Exercises for PPT α10 November 28, 2019 83 Ch.3 Materials

Page 90: Solutions to Exercises for Physics for Physiotherapy ... - Solutions.pdf · SOLUTIONSto Exercises for Physics for Physiotherapy Technology 203-946-DW Fall 2019 by Chris Roderick,

(b) Failure happens at σ= 40 kPa and ε= 6×10−3.Of the given curves only [3] is brittle (it has no regime of plastic deformation before failure).

[4] (a) Picking the point σ= 30 kPa and ε= 10×10−6, we getY = (30×10+3 Pa)

/(10×10−6)= 3.0×10+9 Pa= 3.0 GPa.

(b) (No answer.)

[5] (a) Picking the point σ= 60 MPa and ε= 30×10−3, we getY = (60×10+6 Pa)

/(30×10−3)= 2.0×10+9 Pa= 2.0 GPa.

(b) Failure happens at σ= 80 MPa and ε= 60×10−3, andthe yield point looks like it might be near σ= 60 MPa and ε= 30×10−3 = 0.030.

[6] (a) Picking the point σ= 16 kPa and ε= 12×10−3, we getY = (16×10+3 Pa)

/(12×10−3)= 1.3×10+6 Pa= 1.3 MPa.

(b) Failure happens near σ= 16 kPa and ε= 18×10−3, andthe yield point looks like it might be near σ= 16 kPa and ε= 12×10−3 = 0.012.

[7] (a) Picking the point σ= 20 Pa and ε= 50×10−6, we getY = (20 Pa)

/(50×10−6)= 4.0×10+5 Pa= 0.40 MPa.

(b) Failure happens at σ= 30 Pa and ε= 250×10−6, andthe yield point looks like it might be near σ= 20 Pa and ε= 50×10−6.

[8] (a) Picking the point σ= 150 MPa and ε= 1%= 0.01, we getY = (150×10+6 Pa)

/(0.01)= 15.×10+9 Pa= 15. GPa.

(b) Failure happens at σ= 250 MPa and ε= 2.5%= 0.025, andthe yield point looks like it might be near σ= 150 MPa and ε= 0.01.

[9] (a) Picking the point σ= 50 kPa and ε= 10×10−3, we getY = (50×10+3 Pa)

/(10×10−3)= 5.0×10+6 Pa= 5.0 MPa.

(b) Failure happens at σ= 100 kPa and ε= 80×10−3 = 0.080, andthe yield point looks like it might be near σ= 50 kPa and ε= 10×10−3 = 0.010.

Exercises for PPT α10 November 28, 2019 84 Ch.3 Materials

Page 91: Solutions to Exercises for Physics for Physiotherapy ... - Solutions.pdf · SOLUTIONSto Exercises for Physics for Physiotherapy Technology 203-946-DW Fall 2019 by Chris Roderick,

PROBLEM 3.3.01:

ε(10−3)

σ(kPa)

0 1 2 3 4 50

15

30

45

60

Given the stress-strain curve on the right:(a) What is the Young’s modulus of the material?The Young’s modulus of the material is the slope of the stress-strain graph: Y =∆σ/

∆ε= (60×10+3 Pa)/

(5×10−3)= 12×10+6 Pa=12 MPa.

(b) What is stress in the materialwhen the strain is ε= 3.5×10−3?Visually, the line is somewhere near σ≈ 40 kPa when ε= 3.5×10−3,but we can calculate it more precisely.Since the curve begins at the origin ∆ε= ε−0= ε. From the definitionof Y we can write ∆σ=Y ×∆ε=Y ×ε= (12×10+6 Pa)×(3.5×10−3)=42×10+3 Pa= 42 kPa.

(c) What is strain in the materialwhen the stress is σ= 20 kPa?Visually, the line is somewhere between 1.5×10−3 < ε < 2.0×10−3

when σ= 20 kPa, but we can calculate it more precisely.Since the curve begins at the origin ∆σ = σ− 0 Pa = σ. From thedefinition of Y we can write∆ε=∆σ/

Y =σ/Y = (20×10+3 Pa)

/(12×

10+6 Pa)= 1.7×10−3, which is close to the estimate.

Exercises for PPT α10 November 28, 2019 85 Ch.3 Materials

Page 92: Solutions to Exercises for Physics for Physiotherapy ... - Solutions.pdf · SOLUTIONSto Exercises for Physics for Physiotherapy Technology 203-946-DW Fall 2019 by Chris Roderick,

PROBLEM 3.3.02:

ε(10−3)

σ(MPa)

0 2 4 6 8 10 120

3

6

9

12

15

Given the stress-strain curve on the right:(a) What is the strain at the yield point? If the object were 14.0 cmin length when unstressed, what would be its length at this limit?Inspecting the graph, it looks to be at about ε = 6×10−3. From thedefinition of strain ∆L = ε×Li = (6×10−3)× (14.0×10−2 m) = 84×10−5 m= 0.084 cm. So the length would be Lf = Li +∆L = 14.084 cm.

(b) What is the Young’s modulus of the material in the elasticregime?The Young’s modulus of the material is the slope of the Stress-Straingraph in the elastic regime (the straight-line portion near the ori-gin): Y = ∆σ/

∆ε = (12×10+6 Pa)/

(6×10−3) = 2×10+9 Pa = 2 GPa.

(c) What is stress in the materialwhen the strain is ε= 3.0×10−3?Since the curve begins at the origin ∆ε = ε. From the definition ofY we can write ∆σ = Y ×∆ε = Y × ε = (2×10+9 Pa)× (3.0×10−3) =6×10+6 Pa= 6 MPa.

(d) What is the strain at failure?Inspecting the graph, it looks to be at about ε= 11×10−3.

Exercises for PPT α10 November 28, 2019 86 Ch.3 Materials

Page 93: Solutions to Exercises for Physics for Physiotherapy ... - Solutions.pdf · SOLUTIONSto Exercises for Physics for Physiotherapy Technology 203-946-DW Fall 2019 by Chris Roderick,

PROBLEM 3.3.03:

ε(%)

σ(Pa)

0 1 2 3 4 5 60

5

10

15

20

25

30

35

40

Given the stress-strain curve on the right:(a) From the shape of the stress-strain curve,what type of material is this?With the curved portion at small strain before the linear portionwe recognize this as an elastomeric material (like tendons or liga-ments).

(b) Estimate the strain of the materialwhen the stress is σ= 10 Pa.It looks like ε≈ 3.5%.

(c) Estimate the value of Young’s modulus in the regime where thestress-strain curve is linear.The linear portion of the graph looks like it is in the range 4% < ε<6%. (Remember that “ε= 4%” means ε= 0.04.) The Young’s modulusin this regime is the slope of this segment of the graph: Y =∆σ/

∆ε≈(38 Pa−15 Pa)

/(0.06−0.04)= 1.15×10+3 Pa= 1.2 kPa.

Exercises for PPT α10 November 28, 2019 87 Ch.3 Materials

Page 94: Solutions to Exercises for Physics for Physiotherapy ... - Solutions.pdf · SOLUTIONSto Exercises for Physics for Physiotherapy Technology 203-946-DW Fall 2019 by Chris Roderick,

PROBLEM 3.3.04:

ε(10−3)

σ(MPa)

−20 −10

10 20 30

−24

−20

−16

−12

−8

−4

4

8

12

16

The stress-strain curve above shows the behavior of the material both under compression and under tension. Giventhat curve:

(a) Estimate the stress and strain at yield when the material is under tension.ε≈ 5×10−3 and σ≈ 10 MPa.

(b) Estimate the stress and strain at yield when the material is under compression.ε≈−10×10−3 and σ≈−20 MPa.

(c) Estimate the value of strain at which the stress has its largest magnitude. Is that when the material is undertension or compression?Looking at the graph, the material sustains a larger magnitude of stress when it is compressed (achieving σ≈−21 MPaunder compression versus σ≈+13 MPa under tension). This happens at ε≈−13×10−3, approximately.

(d) Estimate the stress and strain when the material fails under tension.ε≈ 27×10−3 and σ≈ 10 MPa.

(e) Estimate the value of Young’s modulus for values of strain between the compressive yield point and the tensileyield point.The linear portion of the graph is between the two yield points, in the range −10×10−3 < ε < +5×10−3. The Young’smodulus in this regime is the slope of this segment of the graph:

Y =∆σ/∆ε≈ (+10 MPa)− (−20 MPa)

(+5×10−3)− (−10×10−3)= 2.0×10+9 Pa= 2.0 GPa

Exercises for PPT α10 November 28, 2019 88 Ch.3 Materials

Page 95: Solutions to Exercises for Physics for Physiotherapy ... - Solutions.pdf · SOLUTIONSto Exercises for Physics for Physiotherapy Technology 203-946-DW Fall 2019 by Chris Roderick,

Chapter 4

Energy

4.1 Thermal Energy

4.1.1 Energy Transferred

Category 1: Using ∆E = mC ∆T to find the thermal energy ∆E transferred.

EXERCISE 4.1.01 How many kilojoules of thermal energy must be added to increase the temperature of 3.00 L ofwater by 2.00 C?

An increase in the temperature of the object means ∆T =+2.00 C (notice the sign). One litre of water has a mass of1 kg (exact), so 3.00 L has a mass of 3.00 kg. Using the relation ∆E = mC ∆T with the value C = 4184 J/kg ·C forwater we get

∆E = mC ∆T (4.1)

= (3.00 kg)(4184 Jkg·C )(+2.00 C) (4.2)

=+25104 J=+25.1 kJ (4.3)

The positive sign here signifies that thermal energy was added to the object.

EXERCISE 4.1.02 How many kilojoules of thermal energy must be removed from 650 mL of water to decrease itstemperature by 16.00 C?

A decrease in the temperature of the object means ∆T =−16.00 C (notice the sign). One millilitre of water has a massof 1 gram, so 650 mL has a mass of 0.650 kg. Using the relation ∆E = mC ∆T with the value C = 4184 J/kg ·C forwater we get

∆E = mC ∆T (4.4)

= (0.650 kg)(4184 Jkg·C )(−16.00 C) (4.5)

=−43500 J=−43.5 kJ (4.6)

The negative sign here signifies that thermal energy was removed from the object.

EXERCISE 4.1.03 A warm bathtub (172 L) cools by 0.25 C: how much thermal energy does it lose?

The change in temperature is ∆T =−0.25 C. The mass is m = 172 L× 1 kgL = 172 kg.

∆E = mC ∆T (4.7)

= (172 kg)(4184 Jkg·C )(−0.25 C) (4.8)

=−179912 J=−180 kJ (4.9)

EXERCISE 4.1.04 To make a cup of tea we raise 0.53 L by 81 C: how much thermal energy does this take, inkilojoules?

Exercises for PPT α10 November 28, 2019 89 Ch.4 Energy

Page 96: Solutions to Exercises for Physics for Physiotherapy ... - Solutions.pdf · SOLUTIONSto Exercises for Physics for Physiotherapy Technology 203-946-DW Fall 2019 by Chris Roderick,

The change in temperature is ∆T =+81 C. The mass is m = 0.53 L× 1 kgL = 0.53 kg.

∆E = mC ∆T (4.10)

= (0.53 kg)(4184 Jkg·C )(+81 C) (4.11)

=+179619 J=+180 kJ (4.12)

Compare this with the previous exercise.

Exercises for PPT α10 November 28, 2019 90 Ch.4 Energy

Page 97: Solutions to Exercises for Physics for Physiotherapy ... - Solutions.pdf · SOLUTIONSto Exercises for Physics for Physiotherapy Technology 203-946-DW Fall 2019 by Chris Roderick,

4.1.2 Temperature Change

Category 2: Using ∆E = mC ∆T to find the change ∆T = Tfinal −Tinitial in temperature.

EXERCISE 4.1.05 By how much does the temperature of 3.00 L of water change if we transfer 50.2 kJ into it?

Transferring energy into the object means that we are adding energy: ∆E =+50.2 kJ=+50200 J (notice the sign). Themass is m = 3.00 L× 1 kg

L = 3.00 kg.

∆E = mC ∆T (4.13)

∆T =∆E/

(mC ) (4.14)

= (+50200 J)/(

(3.00 kg)(4184 Jkg·C )

)(4.15)

=+4.00 C (4.16)

The positive sign here signifies that the temperature increased.

EXERCISE 4.1.06 By how much does the temperature of 650 mL of water change if we transfer 54.4 kJ out of it?

Transferring energy out of the object means that we are removing energy: ∆E =−54.4 kJ=−54400 J (notice the sign).The mass is m = 0.650 L× 1 kg

L = 0.650 kg.

∆E = mC ∆T (4.17)

∆T =∆E/

(mC ) (4.18)

= (−54400 J)/(

(0.650 kg)(4184 Jkg·C )

)(4.19)

=−20.0 C (4.20)

The negative sign here signifies that the temperature decreased.

EXERCISE 4.1.07 By how much does the temperature of 3.00 L of water change if we transfer 753 kJ into it?

Transferring energy into the object means that we are adding energy: ∆E = +753 kJ = +753000 J (notice the sign).The mass is m = 3.00 L× 1 kg

L = 3.00 kg.

∆E = mC ∆T (4.21)

∆T =∆E/

(mC ) (4.22)

= (+753000 J)/(

(3.00 kg)(4184 Jkg·C )

)(4.23)

=+60.0 C (4.24)

The positive sign here signifies that the temperature increased.

EXERCISE 4.1.08 By how much does the temperature of 650 mL of water change if we transfer 109 kJ out of it?

Transferring energy out of the object means that we are removing energy: ∆E =−109 kJ=−109000 J (notice the sign).The mass is m = 0.650 L× 1 kg

L = 0.650 kg.

∆E = mC ∆T (4.25)

∆T =∆E/

(mC ) (4.26)

= (−109000 J)/(

(0.650 kg)(4184 Jkg·C )

)(4.27)

=−40.0 C (4.28)

The negative sign here signifies that the temperature decreased.

Exercises for PPT α10 November 28, 2019 91 Ch.4 Energy

Page 98: Solutions to Exercises for Physics for Physiotherapy ... - Solutions.pdf · SOLUTIONSto Exercises for Physics for Physiotherapy Technology 203-946-DW Fall 2019 by Chris Roderick,

4.1.3 Temperature, initial or final

Category 3: Using ∆E = mC ∆T to find either the initial temperature Tinitial, or the final temperature Tfinal from∆T = Tfinal −Tinitial.

EXERCISE 4.1.09 We add 50.2 kJ of thermal energy to 3.00 L of water that begins at +18 C. What is its finaltemperature?

Adding thermal energy means that E =+50.2 kJ=+50200 J.The mass is m = 3.00 L× 1 kg

L = 3.00 kg.

∆E = mC ∆T (4.29)

∆T =∆E/

(mC ) (4.30)

Tfinal −Tinitial =∆E/

(mC ) (4.31)

Tfinal = Tinitial +∆E/

(mC ) (4.32)

= (+18 C) + (+50200 J)

(3.00 kg)(4184 Jkg·C )

(4.33)

= (+18 C)+ (+4 C) (4.34)

=+22 C (4.35)

Adding thermal energy will increase the temperature, so the final will be higher than the initial.

EXERCISE 4.1.10 730 mL of hot water is allowed to cool. After it has lost 125 kJ of thermal energy the water hascooled to a temperature of +23.1 C. How hot was it to begin? (What was its initial temperature?)

Losing thermal energy means that E =−125 kJ=−125000 J.The mass is m = 0.730 L× 1 kg

L = 0.730 kg.

∆E = mC ∆T (4.36)

∆T =∆E/

(mC ) (4.37)

Tfinal −Tinitial =∆E/

(mC ) (4.38)

Tinitial = Tfinal −∆E/

(mC ) (4.39)

= (+23.1 C) − (−125000 J)

(0.730 kg)(4184 Jkg·C )

(4.40)

= (+23.1 C)− (−40.9 C) (4.41)

=+64.0 C (4.42)

Losing thermal energy will decrease the temperature, so the initial should be higher than the final.

EXERCISE 4.1.11 We remove 301 kJ of thermal energy from 1.80 L of water that begins at +100. C (it has justfinished boiling). What is its final temperature?

Removing thermal energy means that ∆E =−301 kJ=−301000 J.The mass is m = 1.80 L× 1 kg

L = 1.80 kg.

∆E = mC ∆T (4.43)

∆T =∆E/

(mC ) (4.44)

Tfinal −Tinitial =∆E/

(mC ) (4.45)

Tfinal = Tinitial +∆E/

(mC ) (4.46)

= (+100. C) + (−301000 J)

(1.80 kg)(4184 Jkg·C )

(4.47)

= (+100 C)+ (−40 C) (4.48)

=+60 C (4.49)

Removing thermal energy will decrease the temperature, so the final will be lower than the initial.

Exercises for PPT α10 November 28, 2019 92 Ch.4 Energy

Page 99: Solutions to Exercises for Physics for Physiotherapy ... - Solutions.pdf · SOLUTIONSto Exercises for Physics for Physiotherapy Technology 203-946-DW Fall 2019 by Chris Roderick,

EXERCISE 4.1.12 12.5 L of warm water was being heated. After it has gained 1.674 MJ of thermal energy the waterhas reached a temperature of +92.0 C. How warm was it to begin? (What was its initial temperature?)

Gaining thermal energy means that E =+1.674 MJ=+1674000 J.The mass is m = 12.5 L× 1 kg

L = 12.5 kg.

∆E = mC ∆T (4.50)

∆T =∆E/

(mC ) (4.51)

Tfinal −Tinitial =∆E/

(mC ) (4.52)

Tinitial = Tfinal −∆E/

(mC ) (4.53)

= (+92.0 C) − (+1674000 J)

(12.5 kg)(4184 Jkg·C )

(4.54)

= (+92.0 C)− (+32.0 C) (4.55)

=+60.0 C (4.56)

Losing thermal energy will decrease the temperature, so the initial should be higher than the final.

Exercises for PPT α10 November 28, 2019 93 Ch.4 Energy

Page 100: Solutions to Exercises for Physics for Physiotherapy ... - Solutions.pdf · SOLUTIONSto Exercises for Physics for Physiotherapy Technology 203-946-DW Fall 2019 by Chris Roderick,

4.1.4 Equilibrium Temperature

Category 4: Mixing together two amounts of water (mA and mB) at different initial temperatures (TAi and TBi),the thermal energy that one gains will equal the thermal energy lost by the other. This is conservation of energy:∆EA +∆EB = 0. Once the system has reached equilibrium the combined mass of water will be at a common finaltemperature. From conservation of energy we can solve for unknown temperatures or masses:

Tfinal =mATAi +mBTBi

mA +mB(4.57)

(You can practice your algebra skills by deriving this equation, if you want to.)

EXERCISE 4.1.13 We mix together 1.000 L of water at +30 C with 1.000 L of water at +20 C. What will be the finaltemperature of this mixture?

Let’s call the warmer water “A” and the cooler water “B”. Both masses are mA = mB = 1.00 L× 1 kgL = 1.00 kg. When we

mix these volumes of water to total amount of energy in the mixture does not change, so that the energy that A losesas it cools (∆EA < 0 J) will be the amount that B gains as it warms (∆EB =−∆EA > 0 J). This is written as

∆EA +∆EB = 0 J (4.58)

To solve for the final temperature, we write-out the energies:

∆EA +∆EB = 0 J (4.59)

(mAC (Tfinal −TAi))+ (mBC (Tfinal −TBi))= 0 J (4.60)

(mA +mB)C Tfinal = (mATAi +mBTBi)C (4.61)

Tfinal =mATAi +mBTBi

mA +mB(4.62)

(Notice how the units of energy, and the value of C , are not relevant here – they cancel out of the governing equation.)The final temperature must be between the initial temperatures of the two quantities mixed. Since in this case themasses are equal, the final temperature will be half-way between the two initial temperatures. Substituting our values

Tfinal =mATAi +mBTBi

mA +mB(4.63)

= (1.00 kg)(30 C)+ (1.00 kg)(20 C)1.00 kg+1.00 kg

(4.64)

=+25 C (4.65)

As expected, this is the average of the two initial values.

EXERCISE 4.1.14 We mix together 800 mL of water at +30 C with 1.200 L of water at +20 C. What will be the finaltemperature of this mixture?

Let’s call the warmer water “A” and the cooler water “B”. The masses are mA = 800 mL× 1 kg1000mL = 0.800 kg, and

mB = 1.20 L× 1 kgL = 1.20 kg. We will use the equation developed in the previous exercise:

Tfinal =mATAi +mBTBi

mA +mB(4.66)

The final temperature must be between the initial temperatures of the two substances mixed. Since the masses areunequal, the final temperature will be closer to the initial temperature of the portion with greater mass. Substitutingour values

Tfinal =mATAi +mBTBi

mA +mB(4.67)

= (0.800 kg)(30 C)+ (1.20 kg)(20 C)0.800 kg+1.20 kg

(4.68)

=+24 C (4.69)

Exercises for PPT α10 November 28, 2019 94 Ch.4 Energy

Page 101: Solutions to Exercises for Physics for Physiotherapy ... - Solutions.pdf · SOLUTIONSto Exercises for Physics for Physiotherapy Technology 203-946-DW Fall 2019 by Chris Roderick,

As expected, the greater mass (which was cooler) dominates the result.

EXERCISE 4.1.15 We mix together 1.200 L of water at +30 C with 800 mL of water at +20 C. What will be the finaltemperature of this mixture?

Let’s call the warmer water “A” and the cooler water “B”. The masses are mA = 1.20 L× 1 kgL = 1.20 kg, and mB =

800 mL× 1 kg1000mL = 0.800 kg. The final temperature must be between the initial temperatures of the two substances

mixed. Since the masses are unequal, the final temperature will be closer to the initial temperature of the portion withgreater mass. Substituting our values

Tfinal =mATAi +mBTBi

mA +mB(4.70)

= (1.20 kg)(30 C)+ (0.800 kg)(20 C)1.20 kg+0.800 kg

(4.71)

=+26 C (4.72)

As expected, the greater mass (which was warmer) dominates the result.

EXERCISE 4.1.16 We mix together 1.000 L of water at +30 C with 93 mL of water that was just boiled +100 C.What will be the final temperature of this mixture?

Let’s call the warmer water “A” and the cooler water “B”. The masses are mA = 1.000 L× 1 kgL = 1.000 kg, and mB =

93 mL× 1 kg1000mL = 0.093 kg. The final temperature must be between the initial temperatures of the two substances

mixed. Since the masses are unequal, the final temperature will be closer to the initial temperature of the portion withgreater mass. Substituting our values

Tfinal =mATAi +mBTBi

mA +mB(4.73)

= (1.000 kg)(30 C)+ (0.093 kg)(20 C)1.000 kg+0.093 kg

(4.74)

=+36 C (4.75)

As expected, the greater mass (which was cooler) dominates the result – but the large temperature of the smaller massdid shift the temperature significantly.

Exercises for PPT α10 November 28, 2019 95 Ch.4 Energy

Page 102: Solutions to Exercises for Physics for Physiotherapy ... - Solutions.pdf · SOLUTIONSto Exercises for Physics for Physiotherapy Technology 203-946-DW Fall 2019 by Chris Roderick,

4.2 Kinetic Energy

The kinetic energy of an object of mass m moving at a speed v is

K = 12 mv2 (4.76)

When the mass is measured in kilograms and the speed is measured in metres per second, the kinetic energy calcu-lated by the formula is in joules.

EXERCISE 4.2.01 What is the kinetic energy of a base-ball (mass 153 grams) thrown at a speed of 23 m/s?K = 1

2 (0.153 kg)(23 ms )2 = 40 J.

EXERCISE 4.2.02 What is the kinetic energy of a bird(mass 74.8 grams) flying at a speed of 11.6 m/s?K = 1

2 (0.0748 kg)(11.6 ms )2 = 5.03 J.

EXERCISE 4.2.03 What is the kinetic energy of a bowl-ing ball (mass 5.17 kg) moving at 2.21 m/s?K = 1

2 (5.17 kg)(2.21 ms )2 = 12.6 J.

EXERCISE 4.2.04 What is the kinetic energy of a car(mass 1341 kg) driving at a speed of 10.0 km/h?The speed is 10.0 km

h = 10.0× 1000m3600s = 2.78 m

s . The kineticenergy is K = 1

2 (1341 kg)(2.78 ms )2 = 5.17 kJ.

EXERCISE 4.2.05 What is the kinetic energy of a car(mass 1341 kg) driving at a speed of 20.0 km/h?The speed is 20.0 km

h = 20.0× 1000m3600s = 5.56 m

s . The kineticenergy is K = 1

2 (1341 kg)(5.56 ms )2 = 20.7 kJ. (Notice that

in comparison to the previous exercise, since the speed istwice as much, the kinetic energy is four times as much.)

EXERCISE 4.2.06 What is the kinetic energy of a car(mass 1341 kg) driving at a speed of 100.0 km/h?The speed is 100.0 km

h = 100.0× 1000m3600s = 27.8 m

s . The ki-netic energy is K = 1

2 (1341 kg)(27.8 ms )2 = 517. kJ.

EXERCISE 4.2.07 At what speed (in km/h) is the kineticenergy of a 1200 kg car equal to the energy (approximately1 million joules) released by the explosion of one stick ofTNT?Since K = 1

2 mv2, we have that v =√

2K/

m. Thus v =√2(10+6J)

/(1200 kg) = 40.8 m

s . This speed is v = 40.8 ms =

40.8 ms × 1 km

1000 m × 3600 s1 h = 147 km

h .

EXERCISE 4.2.08 What is the kinetic energy of a person(mass 65.2 kg) walking at a speed of 6.44 km/h?The speed is 6.44 km

h = 6.44× 1000m3600s = 1.789 m

s . The kineticenergy is K = 1

2 (65.2 kg)(1.789 ms )2 = 100. J.

EXERCISE 4.2.09 What was the kinetic energy of UsainBolt (mass 94 kg) sprinting at a speed of 10.0 m/s?K = 1

2 (94 kg)(10.0 ms )2 = 4.7 kJ.

EXERCISE 4.2.10 A group of twenty-seven young chil-dren are running around, fueled by Halloween candy. Inthis group the average mass and speed of a child are37.1 kg and 3.87 m/s. What is the kinetic energy presentin this group of children?On average each child has kinetic energy Kone =12 (37.1 kg)(3.87 m

s )2 = 277.8 J. The group thus has twenty-seven times this much kinetic energy Kgroup = 27×Kone =7.50 kJ.

Exercises for PPT α10 November 28, 2019 96 Ch.4 Energy

Page 103: Solutions to Exercises for Physics for Physiotherapy ... - Solutions.pdf · SOLUTIONSto Exercises for Physics for Physiotherapy Technology 203-946-DW Fall 2019 by Chris Roderick,

4.3 Work

When a force ~F is applied to an object, if the object moves a distance~r along a straight line (its displacement), then thechange in the object’s kinetic energy is

∆K = F r cosθ (4.77)

(where ∆K = Kfinal −Kinitial, and θ is the angle between the force ~F and the displacement ~r). This transfer of energy(either into the object or out of the object) is called the work done to the object by the force:

∆K =W (4.78)

W = F r cosθ (4.79)

EXERCISE 4.3.01 What is the work done by a 5.00 Nforce applied across 70.0 cm if the force is parallel to thedisplacement?When the force is parallel to the displacement θ = 0.The work done is W = F r cosθ = (5.00 N)(0.700 m)cos0 =+3.50 J.

EXERCISE 4.3.02 What is the work done by a 1.11 Nforce applied across 1.23 m if the force points opposite thedisplacement?When the force is opposite to the displacement θ = 180.W = F r cosθ = (1.11 N)(1.23 m)cos180 = −1.37 J. Theforce points opposite the motion, trying to slow or stop theobject, so it is removing energy from the object W < 0 J.

EXERCISE 4.3.03 What is the work done by a 32.00 Nforce applied across 2.75 m if the force points 30 above

the direction of the displacement?W = F r cosθ = (32.00 N)(2.75 m)cos30 =+76.2 J.

EXERCISE 4.3.04 What is the work done by a 42.42 Nforce applied across 5.05 m if the force points 69 abovethe direction opposite the displacement?The angle is θ = 180 − 69 = 111. W = F r cosθ =(42.42 N)(5.05 m)cos111 =−76.8 J.

EXERCISE 4.3.05 What amount of work needs to bedone to a fully loaded shopping cart (39.3 kg) to reduceits speed from 1.72 m/s to 0.23 m/s?Since W = ∆K , we need to find the change in kinetic en-ergy. Kinitial = 1

2 (39.3 kg)(1.72 ms )2 = 58.13 J, and Kfinal =

12 (39.3 kg)(0.23 m

s )2 = 1.04 J. Thus W = ∆K = Kfinal −Kinitial = 1.04 J−58.13 J=−57.1 J

Exercises for PPT α10 November 28, 2019 97 Ch.4 Energy

Page 104: Solutions to Exercises for Physics for Physiotherapy ... - Solutions.pdf · SOLUTIONSto Exercises for Physics for Physiotherapy Technology 203-946-DW Fall 2019 by Chris Roderick,

4.3.1 Problems

PROBLEM 4.3.01: A 75.00 kg object moving at 5.165 ms has −400 J of work done to it.

(a) What is its initial kinetic energy?(b) What will be its final kinetic energy?(c) What will be its final speed? (Notice that the work done to the object is negative (we’ve removed energy from it) soits speed will decrease.)(a) The initial kinetic energy was Ki = 1

2 (75.00 kg)(5.165 ms )2 = 1000 J.

(b) The final kinetic energy will be Kf = Ki +∆K = Ki +W = 1000 J−400 J= 600 J.

(c) The final speed of the object will be vf =√

2Kf/

m =√

2(600 J)/

(75.00 kg)= 4.00 ms .

PROBLEM 4.3.02: When one litre of gasoline (primarily octane) is combusted approximately 42 MJ of thermal energyare released. An internal combustion engine is mechanism that converts thermal energy into mechanical energy.Due to the Laws of Thermodynamics (specifically the Law that prevents entropy from decreasing) at most 30% of thethermal energy can be converted into mechanical energy. This mechanical energy is the work that increases the kineticenergy of the car. If 50 mL of gasoline is combusted, what final speed (in km/h) can a 1723 kg minivan achieve if itbegan at rest?If one litre releases 42 MJ, then 50 mL releases 0.050 L

1.000 L × 42 MJ = 2.100 MJ. The portion of thermal energy that isconverted into mechanical energy is 2.100 MJ×0.30 = 0.630 MJ. Starting from rest means that Ki = 0 J, and ∆K =Kf −Ki = Kf. For that reason

vf =√

2Kf/

m =√

2(0.630×10+6J)/

(1723 kg)= 27.04 ms = 97.4 km

h

PROBLEM 4.3.03: A car (mc = 1320 kg) traveling at 60 km/h comes to a stop. Its kinetic energy is converted by frictioninto thermal energy in the disc brakes. These are two circular discs steel (C = 466 J

kg·C ), each of mass md = 9.5 kg. Bywhat amount does their temperature increase?The thermal energy gained by the discs equals the kinetic energy lost by the car:

∆E+∆K = 0 J (4.80)

(2md)C ∆T + (0 J− 12 mcv2)= 0 J (4.81)

∆T =+ mcv2

2(2md)C=+ (1320 kg)( 60

3.6ms )2

2(2×9.5 kg)(466 Jkg·C )

=+20.7C (4.82)

The brakes do not gain all of this energy all in an instant. The kinetic energy is transformed at a rate that is almostbalanced by the rate at which air flowing past the discs cools them.

Exercises for PPT α10 November 28, 2019 98 Ch.4 Energy

Page 105: Solutions to Exercises for Physics for Physiotherapy ... - Solutions.pdf · SOLUTIONSto Exercises for Physics for Physiotherapy Technology 203-946-DW Fall 2019 by Chris Roderick,

4.4 Power

Power is the rate with respect to time of energy being transferred or transformed:

P =∆E/∆t (4.83)

The units of power are watts: 1 W = 1 J/s. Another unit of power that is sometimes used in the context of vehicles andmachinery is the horsepower:

1 hp= 746 W (4.84)

If we know the power and the amount of time, then we have the amount of energy that was transferred or trans-formed:

∆E = P ×∆t (4.85)

In the context of electricity, where power is measured in kilowatts and time is measured in hours, the amount of energyis measured in kilowatt-hours

1 kWh= 1 kW×1 h= 1000 W×3600 s= 3.6 MJ (4.86)

as an exact number. In the context of food energy, the food calorie (or kilocalorie) is meaningful: 1 Cal= 4.184 kJ.

4.4.1 Power, Energy, and Price

EXERCISE 4.4.01 If an old 60 W incandescent light bulbis left on for one hour, how much energy (in kilojoules) doesit dissipate?∆E = P ×∆t = (60 W)× (3600 s)= 216 kJ

EXERCISE 4.4.02 If a 15 W LED light bulb is left onfor one hour, how much energy (in kilojoules) does it dissi-pate?∆E = P ×∆t = (15 W)× (3600 s)= 54 kJ

EXERCISE 4.4.03 A 1200 W microwave is set to run atfull power for 77 s. How much energy (in kilojoules) did ituse?∆E = P ×∆t = (1200 W)× (77 s)= 92 kJ

EXERCISE 4.4.04 A television is left in its “stand-by”mode for eighteen hours. If that mode uses 24.0 W howmuch energy (in megajoules) did it use?∆E = P ×∆t = (24.0 W)× (18×3600 s)= 1.56 MJ

EXERCISE 4.4.05 Hydro Quebec charges 5.91 cents perkilowatt-hour of energy used. If an old 60W incandescentlight bulb is left on for one whole week, how much will itcost?Express the energy in kWh: ∆E = P ×∆t = (0.060 kW)×(7×24 h)= 10.08 kWh.The cost is: 10.08 kWh×$0.0591/kWh = $0.60. (Taxes notincluded, haha.)

Exercises for PPT α10 November 28, 2019 99 Ch.4 Energy

Page 106: Solutions to Exercises for Physics for Physiotherapy ... - Solutions.pdf · SOLUTIONSto Exercises for Physics for Physiotherapy Technology 203-946-DW Fall 2019 by Chris Roderick,

4.4.2 Problems

PROBLEM 4.4.01: How much does it cost to heat a 2.50 litre container of coffee from 4C to 99C using a 1200 Wmicrowave? (Assume the coffee is like water with C = 4.184 J/gram ·C, and that Hydro Quebec charges $0.0591/kWh.)

The amount of heat transferred to the coffee is Q = mC ∆T = (2500 gram)(4.184 Jgram·C )(99C−4C) = 993700 J =

0.276 kWh. This will cost 0.276 kWh×$0.0591/kWh = $0.0163 = $0.02. (Of course we round prices upwards, haha. Ifwe were paying cash, we would have to round to the nearest increment of five cents!)

PROBLEM 4.4.02: A person is using a 2200 W heater to keep their apartment warm. After twenty minutes off, itturns on for five minutes, and that repeats. If they keep it running like this how much will it cost them for a 30-daymonth? (Hydro Quebec charges $0.0591/kWh.)

The amount of time that the heater is on is 5 min during each 25 min. So the total time the heater is on is∆t = 30 day×24 h

day × ( 525 ) = 144 h. The energy consumed will be ∆E = P ×∆t = (2.200 kW)× (144 h) = 316.8 kWh. This

will cost 316.8 kWh×$0.0591/kWh= $18.72.

PROBLEM 4.4.03: One litre of gasoline, when combusted, can release 42 MJ (megajoules) of thermal energy. If mycar used 20 litres of gas to drive to Ottawa, then(a) How much thermal energy did my car produce over the whole trip?(b) If the trip took 2.0 hours, what was the rate (in kilowatts) at which thermal energy was produced by the engine?(c) If only 30% of that thermal energy was converted into mechanical work that actually moved my car, what was thismechanical power output, measured in horsepower?

(a) The thermal energy produced was E = 20 L×42 MJL = 840 MJ.

(b) The rate of energy production was Pthermal = E/∆t = 840×106 J

2.0×3600 s = 117×103 J/s= 117 kW.

(c) Of that power, only Pwork = 0.30× Pthermal = 0.30× 117 kW = 35 kW of power was output as mechanical work.Expressed in horsepower, this is 35 kW× 1 hp

746 W = 47 hp.

PROBLEM 4.4.04: A steam engine expels 81 J of energy to the environment for every 100 J of internal energy it uses.(a) How much work does it generate? (b) What is the engine’s efficiency?

(a) If 81 J of the 100 J was expelled as waste, then only 100 J−81 J= 19 J can manifest as work.

(b) The efficiency is e =W /E = (19 J)/(100 J)= 0.19= 19%.

PROBLEM 4.4.05: One serving of Pad Thai contains about 573 Calories of food energy (1 Calorie = 4.184 kJ). If youhave a body mass of 65 kg, when you walk at 9 km/h you “burn” 9.55 Calories per minute.(a) What is the rate of energy expenditure, measured in watts?(b) At this rate how long (in hours) would it take to “burn off” your Pad Thai?

(a) The power of 9.55 Calories per minute equals 9.55 Calmin × 4184. J

1 Cal × 1 min60 s = 666 W.

(b) The energy to be developed is 573 Cal= 2.40×106 J.The time this will take is ∆t =∆E

/P = 2.40×106 J

666 W = 3.60×103 s, which is one hour.

PROBLEM 4.4.06: Assuming that muscles convert food energy into mechanical energy with an efficiency of 22%, howmuch food energy is converted by an 80-kg man climbing a vertical distance of 15m? Express your answer in kilojoules,and in food Calories.

The mechanical work he does changes is gravitational potential energy:

∆UG = mg∆h = (80 kg)(9.8 Nkg )(+15 m)=+11760 J (4.87)

This work is only 22% of the food energy he consumed. The full 100% is (11760 J)/0.22= 53455 J= 53.5 kJ. Convertingthis to food Calories gives 53.5 kJ× 1 Cal

4.184 kJ = 12.8 Cal.

PROBLEM 4.4.07: An 80-kg man runs up stairs, ascending 6.0 m in 8.0 s. What is his power output in kilowatts, andin horsepower?

The work done by the man is the change in his gravitational potential energy:

∆UG = mg∆h = (80 kg)(9.81 Nkg )(+6.0 m)=+4709 J= 4.709 kJ (4.88)

Exercises for PPT α10 November 28, 2019 100 Ch.4 Energy

Page 107: Solutions to Exercises for Physics for Physiotherapy ... - Solutions.pdf · SOLUTIONSto Exercises for Physics for Physiotherapy Technology 203-946-DW Fall 2019 by Chris Roderick,

This work was performed at a rate of

P = W∆t

= 4.709 kJ8.0 s

= 0.589 kW (4.89)

Expressed in horsepower, this is 589 W× 1 hp746 W = 0.789 hp.

PROBLEM 4.4.08: The chemical process that drives muscles can produce about 20 kJ of mechanical energy for eachlitre of oxygen a human respires. If a sprinter consumes 4.1 L/min of oxygen, what is their maximum possible poweroutput, in watts, and in Calories per minute?

The rate of oxygen volume consumption is 4.1 L/min = 4.1 L/60 s = 0.068333 L/s. Applying the conversion of oxygenvolume to energy produced, the power generated is

P = 0.068333 Ls ×20 kJ

L = 1.367 kJs = 1367 J

s = 1367 W (4.90)

That’s almost two horsepower! Obviously this is not a power that can be sustained for very long – seconds at most –and only by Olympic-level athletes. Since 1 Calorie= 4184 J, this power is also 1367 J

s × 1 Cal4184 J × 60 s

1 min = 19.603 Cal/min.

PROBLEM 4.4.09: What is the rate of metabolic power output of a sitting person who consumes 0.30 L of oxygen perminute?

The conversion of oxygen consumption rate to power is

P = 0.30 Lmin ×20 kJ

L × 1 min60 s = 100 W (4.91)

PROBLEM 4.4.10: What is the rate of oxygen consumption during sleep assuming a metabolic rate of 75 W?

Since the power is 75 W= 75 J/s, the corresponding rate of oxygen consumption is75 J

s × 1 L20000 J = 0.003750 L/s= 3.75 mL/s= 0.225 L/min.

PROBLEM 4.4.11: A 64kg woman is slowly descending the stairs, traveling 16.5m downwards in 37s. (a) At whatrate must her body dissipate gravitational energy so that so descends at a constant rate? (b) If all of that energy wasconverted into thermal energy in her body, by what amount would her temperature increase?

(a) The power is the change divided by the time. The change in gravitational energy is

∆UG = mg∆h = (64 kg)(9.81 Nkg )(−16.5 m)=−10359 J (4.92)

The dissipation power must be

P = −10359 J37 s

=−280 W (4.93)

(b) The gravitational energy that was dissipated becomes thermal energy in her body:

∆E = mC ∆T (4.94)

∆T =∆E/

mC (4.95)

= (+10359 J)/

(64 kg×4184 Jkg·C ) (4.96)

=+0.0387 C ≈+0.04 C (4.97)

This is barely noticeable, and is easily removed by respiration and thermal radiation.

PROBLEM 4.4.12: In the video “Olympic Cyclist vs Toaster”, Robert Förstemann (Germany) sustains a power outputof 700W for one minute, approximately. (Notice: That is almost one horse-power!) (a) What was his total energyoutput, measured in kilowatt-hours? (b) If that output was only 22% of the chemical energy consumed by his muscles,how many Calories did that effort require? (c) Consequently, how much oxygen did he need to respire?

(a) The energy output was ∆E = P ×∆t = (700 W)(60 s)= 42000 J× 1 kWh3.6×10+6 J = 0.012 kWh.

(b) That amount of energy output was 42000 J× 1 Cal4184 J = 10.04 Cal. If that was only 22%, then he actually produced

E = 10.22 ×10.04 Cal= 45.63 Cal.

(c) The relation between oxygen respired and metabolic energy is 1 L20000 J = 1 L

4.780 Cal , he respired 1 L4.780 Cal ×45.63 Cal =

9.55 L.

Exercises for PPT α10 November 28, 2019 101 Ch.4 Energy

Page 108: Solutions to Exercises for Physics for Physiotherapy ... - Solutions.pdf · SOLUTIONSto Exercises for Physics for Physiotherapy Technology 203-946-DW Fall 2019 by Chris Roderick,

Chapter 5

Waves5.1 Periodic Waves5.1.1 Period & Frequency

In these exercises we will be determining the period T (the amount of time between repetitions) and the frequency f(the rate of repetition) of various phenomena. The frequency f of a repeating event is defined to be the ratio of thenumber of repetitions to the time taken for those repetitions to occur, and is measured in hertz:

1 hertz= 1 repetitionsecond (5.1)

1 Hz= 1 reps (5.2)

(Note that “repetition” is not an SI unit, but is a place-holder or reminder. When calculating it does not contributeanything except to help keep track of consistency. Strictly speaking 1 Hz = 1 s−1.) Because of their definitions theyrelate: f = 1/T. So we can also write the units of period as “seconds per repetition”.

EXR 5.1.01 If a person’s heart rate is 70 beats perminute, what is the period and frequency of that oscilla-tion?The frequency f of a repeating event is defined to be theratio of the number of repetitions to the time taken forthose repetitions to occur. In this problem

f = 70 rep60 s

= 1.17 Hz (5.3)

The period T of a repeating event is defined to be the timetaken per repetition

T = 60 s70 rep

= 0.857 s (5.4)

Remember that f = 1/T can be used to get one from theother.

EXR 5.1.02 The human heart rate can fall in the rangefrom 60 beats per minute (very relaxed) to 120 beats perminute (intense exercise). To what range of frequenciesdoes this correspond?The range of heart rates mentioned for humans corre-sponds to frequencies in the range (60 rep)

/(60 s) = 1 Hz

to (120 rep)/

(60 s)= 2 Hz.

EXR 5.1.03 An adult human at rest has a respirationrate between 12 to 16 times a minute. To what range offrequencies does this correspond?The range of respiration rates mentioned correspondsto frequencies in the range (12 rep)

/(60 s) = 0.20 Hz to

(16 rep)/

(60 s)= 0.27 Hz.

EXR 5.1.04 During exercise a person is respiring at arate of 40 breaths per minute. What is period and fre-quency of this rate?

This rate of respiration has period and frequency T =(60 s)

/(40 rep) = 1.5 s and f = (40 rep)

/(60 s) = 0.67 Hz,

respectively.

EXR 5.1.05 The average computer user can type about190 characters per minute, while professional typists cantype as fast as 360 characters per minute. What are thefrequency of keys per second for these two keyboardists?For the average user f = (190 rep)

/(60 s) = 3.17 Hz while

for the professional f = (360 rep)/

(60 s)= 6.00 Hz.

EXR 5.1.06 Players of video games sometimes need toperform “button mashing” when they repeatedly press acontrol a rapidly as possible. A player pressed a button191 times in 12 seconds. What are the period and fre-quency of this rate of button mashing?This rate of button mashing has period and frequencyT = (12 s)

/(191 rep) = 0.0628 s and f = (191 rep)

/(12 s) =

15.92 Hz, respectively.

EXR 5.1.07 The average male voice has an oscillationaround 130 Hz. A bass singer usually has a low notearound 82.4 Hz. One of the lowest bass singers, a fellowby the name Tim Storms, can sing notes lower than B0(30.87 Hz)! What are the periods of oscillation of the vocalchords of men making these three notes?For each of these three cases we need to use T = 1/ f :T = 1/ f = 1

/(130 Hz)= 0.007692 s= 7.69 ms.

T = 1/ f = 1/

(82.4 Hz)= 0.01214 s= 12.1 ms.T = 1/ f = 1

/(30.87 Hz) = 0.03239 s = 32.4 ms (four times

slower than speaking!).

EXR 5.1.08 The average female voice has an oscillationaround 210 Hz. A soprano singer usually has a high notearound 650 Hz. Above the soprano range is the so-called

Exercises for PPT α10 November 28, 2019 102 Ch.5 Waves

Page 109: Solutions to Exercises for Physics for Physiotherapy ... - Solutions.pdf · SOLUTIONSto Exercises for Physics for Physiotherapy Technology 203-946-DW Fall 2019 by Chris Roderick,

whistle register. Mariah Carey was once recorded hittingG7 (3.136 kHz)! What are the periods of oscillation of thevocal chords of women making these three notes?For each of these three cases we need to use T = 1/ f :T = 1/ f = 1

/(210 Hz)= 0.004762 s= 4.76 ms.

T = 1/ f = 1/

(650 Hz)= 0.001538 s= 1.54 ms.T = 1/ f = 1

/(3136 Hz)= 0.0003189 s= 0.319 ms.

It is interesting to note that the biomechanics of hownotes in the whistle register are produced is not clearlyunderstood! It is also not exclusive to female vocalists, asthere are some male vocalists with this range. Also thesenotes are easily produced by young children.

EXR 5.1.09 The heart of a hummingbird can beat asfast as 1260 times a minute. What is this rate measuredin hertz?The frequency of a repeating event is defined to be “num-ber of repetitions / time”. Since 1 minute is 60 seconds, thefrequency is f = (1260 rep)

/(60 s)= 21 Hz.

EXR 5.1.10 The wings of a hummingbird can flap asfast as 10000 times in three minutes! What is this ratemeasured in hertz?The frequency of a repeating event is defined to be “num-ber of repetitions / time”. Since 1 minute is 60 seconds, thefrequency is f = (10000 rep)

/(180 s)= 56 Hz.

EXR 5.1.11 The musical-sounding notes produced bybirds, birdsong, falls in the range 1 kHz to 8 kHz. (Com-pare this with the whistle register of human singing.)What is the range of the periods of oscillation of the vo-cal chords of birds making notes in this range?To calculate the extremes of the range we need to useT = 1/ f :T = 1/ f = 1

/(1000 Hz)= 0.00100 s= 1.00 ms.

T = 1/ f = 1/

(8000 Hz)= 0.000125 s= 0.125 ms.

EXR 5.1.12 Lower-frequency sounds travel further thanhigh-frequency sounds. (This is due to the phenomena ofattenuation that we will study later.) Elephants communi-cate using some frequencies below what humans can hear.The tones they produce fall in the range of 5 Hz to 30 Hz.What is the range of the periods of oscillation correspond-ing to notes in this range?To calculate the extremes of the range we need to useT = 1/ f :T = 1/ f = 1

/(5 Hz)= 0.200 s.

T = 1/ f = 1/

(30 Hz)= 0.0333 s= 33.3 ms.

Note that these sounds are not produced by only thevocal chords of the elephants, but by coupled resonance ofthe air mass in their trunks.

Exercises for PPT α10 November 28, 2019 103 Ch.5 Waves

Page 110: Solutions to Exercises for Physics for Physiotherapy ... - Solutions.pdf · SOLUTIONSto Exercises for Physics for Physiotherapy Technology 203-946-DW Fall 2019 by Chris Roderick,

5.1.2 The Fundamental Relationship for Periodic Waves

These exercises will use the fundamental relationship for periodic waves

λ= v/

f (5.5)

to relate Frequency ( f ), Wavelength (λ) and Wavespeed (v). The SI units to use for these quantities are hertz (Hz) forfrequency, metres (m) for wavelength, and metres per second (m/s) for wavespeed.

EXR 5.1.13 What is the speed of a periodic wave whosefrequency and wavelength are 500 Hz and 0.5 m respec-tively?Here v =λ f = (0.5 m)(500 Hz)= 250 m/s.

EXR 5.1.14 What is the wavespeed on a steel cable if aperiodic wave of frequency 75 Hz has wavelength 1.333 m?

Here v =λ f = (1.333 m)(75 Hz)= 100 m/s.

EXR 5.1.15 What is the speed of an ultrasonic wave inflesh whose frequency and wavelength are 2.20 MHz and705µm respectively?v = λ f = (705 µm)(2.20 MHz) = (705 × 10−6m)(2.20 ×10+6Hz)= 705×2.20 m/s= 1551 m/s.

EXR 5.1.16 What is the speed of a ultrasonic wavein air whose frequency and wavelength are 45.0 kHz and7.63 mm respectively?v = λ f = (7.63 mm)(45.0 kHz) = (7.63 × 10−3m)(45.0 ×10+3Hz)= 7.63×45.0 m/s= 343 m/s.

EXR 5.1.17 What is the wavelength of a periodic waveon a stretched spring whose wavespeed and period of os-cillation are are 75 m/s and 0.005 s respectively?Since f = 1/T we have λ = v

/f = vT. Here λ = vT =

(75 m/s)(0.005 s)= 0.375 m.

EXR 5.1.18 Electrical power is delivered by oscillatingvoltages of frequency 60.0 Hz. These electrical oscillationscan sometimes cause mechanical oscillations of equal fre-quency, which then become sound in the surrounding air.Since the wavespeed in air is 343 m/s what is the wave-length of these sounds?

Here λ= v/

f = (343 m/s)/

(60.0 Hz)= 5.72 m.

EXR 5.1.19 What is the wavelength of an underwatersound wave whose wavespeed and frequency of oscillationare are 1480 m/s and 5.90 kHz respectively?Here λ= v

/f = (1480 m/s)

/(5900 Hz)= 0.251 m.

EXR 5.1.20 Some ultrasonic waves traveling throughfat tissue have wavespeed and frequency of oscillation1450 m/s and 1.55 MHz respectively. What is the wave-length of these waves?λ = v

/f = (1450 m/s)

/(1.55× 10+6 Hz) = 9.35× 10−4 m =

0.935 mm.

EXR 5.1.21 What is the frequency of a periodic wavewhose wavespeed and wavelength are 120 m/s and 30 cmrespectively?Since λ= v

/f we can get that f = v

/λ.

Here f = v/λ= (120 m/s)/(0.30 m)= 400 Hz.

EXR 5.1.22 What is the frequency of a wave on astretched spring whose wavespeed and wavelength are3.91 m/s and 32.5 cm respectively?Here f = v/λ= (3.91 m/s)/(0.325 m)= 12.0 Hz.

EXR 5.1.23 What is the frequency of a sound wavewhose speed and wavelength are 343 m/s and 20.3 cm re-spectively?Here f = v/λ= (343 m/s)/(0.203 m)= 1690 Hz.

EXR 5.1.24 What is the frequency of an ultrasonic wavetraveling through bone whose speed and wavelength are3290 m/s and 1.94 mm respectively?f = v/λ = (3290 m/s)/(0.00194 m) = 1.70 × 10+6 Hz =1.70 MHz.

Exercises for PPT α10 November 28, 2019 104 Ch.5 Waves

Page 111: Solutions to Exercises for Physics for Physiotherapy ... - Solutions.pdf · SOLUTIONSto Exercises for Physics for Physiotherapy Technology 203-946-DW Fall 2019 by Chris Roderick,

5.1.3 Graphs of WavesDetermining the Amplitude, Wavelength, and other parameters, of Pulses and Periodic Waves from Graphs.

Terminology: A pulse is a wave that does not repeat. A pulse travels at the wavespeed of the medium, but does nothave a wavelength or frequency because it does not repeat across space or across time.

Displacement versus Position

EXR 5.1.25 What are the amplitude and wavelength of the periodic wave graphed below?Answer: A = 1.4 cm and λ= 4.0 m.

x(m)

y(cm)

1 2 3 4 5 6 7 8 9 10

−2

−1

0

1

2

EXR 5.1.26 What are the amplitude and wavelength of the periodic wave graphed below?Answer: A = 1.8 cm and λ= 5.0 m.

x(m)

y(cm)

1 2 3 4 5 6 7 8 9 10

−2

−1

0

1

2

EXR 5.1.27 What are the amplitude and wavelength of the periodic wave graphed below?Answer: A = 2.0 cm and λ= 6.0 m.

x(m)

y(cm)

1 2 3 4 5 6 7 8 9 10

−2

−1

0

1

2

Exercises for PPT α10 November 28, 2019 105 Ch.5 Waves

Page 112: Solutions to Exercises for Physics for Physiotherapy ... - Solutions.pdf · SOLUTIONSto Exercises for Physics for Physiotherapy Technology 203-946-DW Fall 2019 by Chris Roderick,

EXR 5.1.28 What are the amplitude and wavelength of the periodic wave graphed below?Answer: A = 1.2 cm and λ= 3.0 m.

x(m)

y(cm)

1 2 3 4 5 6 7 8 9 10

−2

−1

0

1

2

EXR 5.1.29 What are the amplitude and wavelength of the periodic wave graphed below?Answer: A = 0.8 cm and λ= 2.2 m.

x(m)

y(cm)

1 2 3 4 5 6 7 8 9 10

−2

−1

0

1

2

EXR 5.1.30 What are the amplitude and wavelength of the periodic wave graphed below?Answer: A = 1.2 cm and λ= 3.6 m.

x(m)

y(cm)

1 2 3 4 5 6 7 8 9 10

−2

−1

0

1

2

Exercises for PPT α10 November 28, 2019 106 Ch.5 Waves

Page 113: Solutions to Exercises for Physics for Physiotherapy ... - Solutions.pdf · SOLUTIONSto Exercises for Physics for Physiotherapy Technology 203-946-DW Fall 2019 by Chris Roderick,

EXR 5.1.31 What are the amplitude and wavelength of the periodic wave graphed below?Answer: A = 1.4 cm and λ= 14.0 m.

x(m)

y(cm)

1 2 3 4 5 6 7 8 9 10

−2

−1

0

1

2

EXR 5.1.32 What are the amplitude and wavelength of the periodic wave graphed below?Answer: A = 2.0 cm and λ= 16.0 m.

x(m)

y(cm)

1 2 3 4 5 6 7 8 9 10

−2

−1

0

1

2

Exercises for PPT α10 November 28, 2019 107 Ch.5 Waves

Page 114: Solutions to Exercises for Physics for Physiotherapy ... - Solutions.pdf · SOLUTIONSto Exercises for Physics for Physiotherapy Technology 203-946-DW Fall 2019 by Chris Roderick,

Displacement versus Time

EXR 5.1.33 A periodic wave is traveling across a stretched string. Graphed below is the displacement as a functionof time of a single piece of that string. What are the amplitude, period, and frequency of this wave?Answer: A = 1.6 cm, T = 0.34 s, and f = 2.94 Hz.

t(s)

y(cm)

0.1 0.2 0.3 0.4 0.5 0.6 0.7 0.8 0.9 1.0

−2

−1

0

1

2

EXR 5.1.34 A periodic wave is traveling across a stretched string. Graphed below is the displacement as a functionof time of a single piece of that string. What are the amplitude, period, and frequency of this wave?Answer: A = 1.2 cm, T = 0.46 s, and f = 2.17 Hz.

t(s)

y(cm)

0.1 0.2 0.3 0.4 0.5 0.6 0.7 0.8 0.9 1.0

−2

−1

0

1

2

EXR 5.1.35 A periodic wave is traveling across a metal rod. Graphed below is the displacement as a function of timeof a single piece of that object. What are the amplitude, period, and frequency of this wave?Answer: A = 12.0µm, T = 11.0 ms, and f = 90.9 Hz.

t(ms)

y(µm)

10 20 30 40 50

−10

−5

0

5

10

Exercises for PPT α10 November 28, 2019 108 Ch.5 Waves

Page 115: Solutions to Exercises for Physics for Physiotherapy ... - Solutions.pdf · SOLUTIONSto Exercises for Physics for Physiotherapy Technology 203-946-DW Fall 2019 by Chris Roderick,

EXR 5.1.36 A periodic sound wave is traveling through water. Graphed below is the displacement as a function oftime of a single portion of the medium. What are the amplitude, period, and frequency of this wave?Answer: A = 9.0 nm, T = 32.0µs, and f = 31.3 kHz.

t(µs)

y(nm)

20 40 60 80 100

−10

−5

0

5

10

Exercises for PPT α10 November 28, 2019 109 Ch.5 Waves

Page 116: Solutions to Exercises for Physics for Physiotherapy ... - Solutions.pdf · SOLUTIONSto Exercises for Physics for Physiotherapy Technology 203-946-DW Fall 2019 by Chris Roderick,

Pairs of Graphs

PROBLEM 5.1.01: Below are two photographs of a traveling periodic wave, taken at two different times. (The timebetween these photographs is less than half the period of oscillation of the source.) (a) What direction is the wavetraveling, and at what speed? (b) What are the wavelength and frequency of this wave?

Answer: (a) Towards the left at v = 7.14 m/s. (b) λ= 4.0 m and f = 1.79 Hz.

x(m)

y(cm)

1 2 3 4 5 6 7 8 9 10

−2

−1

0

1

2 t1 = 0.210 s

x(m)

y(cm)

1 2 3 4 5 6 7 8 9 10

−2

−1

0

1

2 t2 = 0.350 s

(a) To find the speed of the wave we can track the motion of a single feature of the wave’s shape. For example, lookingat the photograph of the wave at t1 = 0.210 s we can see where it see where it crosses the x-axis at x1 = 3.0 m, slopingupwards. In the later photograph at t2 = 0.350 s we can see that that feature has moved to x2 = 2.0 m. Consequentlywe find that the wave has moved ∆x = x2 − x1 =−1.0 m (one metre towards the left).

That displacement happens over a time interval ∆t = t2− t1 = 0.140 s. The velocity of the wave is thus vx =∆x/∆t =

(−1.0 m)/

(0.140 s)=−7.14 m/s. The wavespeed is v = 7.14 m/s, and the wave is traveling towards the left.

(b) From the second photograph it is easy to see that he wavelength is λ= 4.0 m. Due to the fundamental relation ofperiodic waves (λ= v

/f ) we get that f = v

/λ= (7.14 m/s)

/(4.0 m)= 1.79 Hz.

Exercises for PPT α10 November 28, 2019 110 Ch.5 Waves

Page 117: Solutions to Exercises for Physics for Physiotherapy ... - Solutions.pdf · SOLUTIONSto Exercises for Physics for Physiotherapy Technology 203-946-DW Fall 2019 by Chris Roderick,

PROBLEM 5.1.02: Below are two frames from a video of a traveling periodic wave. (The time between these pho-tographs is less than half the period of oscillation of the source.) (a) What direction is the wave traveling, and at whatspeed? (b) What are the wavelength and frequency of this wave?

Answer: (a) Towards the right at v = 200 m/s. (b) λ= 3.0 m and f = 67 Hz.

x(m)

y(cm)

1 2 3 4 5 6 7 8 9 10

−2

−1

0

1

2 t1 = 1.050 s

x(m)

y(cm)

1 2 3 4 5 6 7 8 9 10

−2

−1

0

1

2 t2 = 1.052 s

(a) Looking at where the displacement of the wave crosses the x-axis we can see that the wave has moved ∆x =+0.4 mover a time interval ∆t = 0.002 s. The velocity of the wave is thus vx = ∆x

/∆t = (+0.4 m)

/(0.002 s) = +200 m/s. The

wavespeed is v = 200 m/s, and the wave is traveling towards the right.

(b) The wavelength is λ = 3.0 m. Due to the fundamental relation of periodic waves (λ = v/

f ) we get that f = v/λ =

(200 m/s)/

(3.0 m)= 67 Hz.

Exercises for PPT α10 November 28, 2019 111 Ch.5 Waves

Page 118: Solutions to Exercises for Physics for Physiotherapy ... - Solutions.pdf · SOLUTIONSto Exercises for Physics for Physiotherapy Technology 203-946-DW Fall 2019 by Chris Roderick,

PROBLEM 5.1.03: Below are two frames from a video of a periodic wave traveling along a thin metal rod. (The timebetween these photographs is less than half the period of oscillation of the source.) (a) What direction is the wavetraveling, and at what speed? (b) What are the wavelength and frequency of this wave?

Answer: (a) Towards the left at v = 400 m/s. (b) λ= 23.0 m and f = 17.4 Hz.

x(m)

y(mm)

10 20 30 40 50

−10

−5

0

5

10 t1 = 7.276 s

x(m)

y(mm)

10 20 30 40 50

−10

−5

0

5

10 t2 = 7.281 s

(a) Looking at where the displacement of the wave crosses the x-axis we can see that the wave has moved ∆x =−2.0 mover a time interval ∆t = 0.005 s. The velocity of the wave is thus vx = ∆x

/∆t = (−2.0 m)

/(0.005 s) = −400 m/s. The

wavespeed is v = 400 m/s, and the wave is traveling towards the left.

(b) The wavelength is λ = 23.0 m. Due to the fundamental relation of periodic waves (λ = v/

f ) we get that f = v/λ =

(400 m/s)/

(23.0 m)= 17.4 Hz.

Exercises for PPT α10 November 28, 2019 112 Ch.5 Waves

Page 119: Solutions to Exercises for Physics for Physiotherapy ... - Solutions.pdf · SOLUTIONSto Exercises for Physics for Physiotherapy Technology 203-946-DW Fall 2019 by Chris Roderick,

PROBLEM 5.1.04: The graphs below show measurements made of a traveling wave on a string. The first graph is thedisplacement (y) as a function of time (t) for a small piece of the string. The second graph is the displacement (y) as afunction of position (x) along the length, taken at t = 0 s.

Find (a) the amplitude, (b) the wavelength, (c) the period, (d) the frequency, and (e) the wavespeed of the wave.

Answers: (a) A = 1.5 cm; (b) λ= 3.0 m; (c) T = 0.500 s; (d) f = 2.00 Hz; and (e) v = 6.0 m/s.

t(s)

y(cm)

0.25 0.50 0.75 1.00

−2

−1

0

1

2

x(m)

y(cm)

1 2 3 4 5 6 7 8 9

−2

−1

0

1

2

(a) The amplitude of the wave is found from the graph just by looking at it! The amplitude is the height of any of thepeaks in either graph: A = 1.5 cm.

(b) The wavelength is the distance between repetitions across space of the wave’s shape. The graph of displacement(y) as a function of position (x) crosses the horizontal axis, sloping upwards, at x = 3.0 m. Tracing along the contour ofthe string the next place where the string crosses the axis sloping upwards is at x = 6.0 m. After that the shape of thewave repeats. Thus λ= 6.0 m−3.0 m= 3.0 m.

(c) The graph of displacement (y) as a function of time (t) crosses the horizontal axis, sloping upwards, at t = 0.375 s.Tracing along the contour of the graph the next place where the line crosses the axis sloping upwards is at t = 0.875 s.After that the oscillation repeats. Thus T = 0.875 s−0.375 s= 0.500 s.

(d) By definition f = 1/T = 1/(0.500 s)= 2.00 Hz.

(e) From the fundamental relation for periodic waves (λ= v/

f ) we have v =λ f = (3.0 m)(2.00 Hz)= 6.0 m/s.

Exercises for PPT α10 November 28, 2019 113 Ch.5 Waves

Page 120: Solutions to Exercises for Physics for Physiotherapy ... - Solutions.pdf · SOLUTIONSto Exercises for Physics for Physiotherapy Technology 203-946-DW Fall 2019 by Chris Roderick,

5.2 Sound

Traveling waves in air, water and people.

5.2.1 Intensity, Power, and Area

Intensity measures the way in which the power transferred by a wave is distributed across an area:

I = P/

A (5.6)

Intensity is typically measured in watts per square metre W/m2, but in some circumstances (when the areas involvedare human-sized) it is measured in watts per square centimetre W/cm2. It is important to remember that while lengthsconvert as 1 m=100 cm, areas convert as 1m2 = (100cm)2 = 10000cm2.

When a wave spreads out in three dimensions, the wavefronts are spherical. The surface area of a sphere is givenby

Asphere = 4π r2 (5.7)

(Yes, this differs from the formula for the area of a flat circle by a factor of four, so be careful not to mix them up.)Because of this, the intensity a distance r from a source of power P (producing either sound or light) is

I = P4π r2 (5.8)

This is known as the inverse-square law for intensity.

EXR 5.2.01 Find the intensity (measured in W/m2) of awave that delivers 16.0 W to a rectangular area that mea-sures 23 cm by 71 cm.Since we are asked for the intensity to be measured persquare metre, we should convert distances to metres first:I = P/A = 16.0W

/(0.23m×0.71m)= 98W/m2.

EXR 5.2.02 Find the intensity (measured in W/m2) ofa wave that delivers 72.5 W to the surface of a door thatmeasures 1.05 m by 1.82 m.I = P/A = 72.5W

/(1.05m×1.82m)= 37.9W/m2.

EXR 5.2.03 Find the intensity (measured in W/m2) of awave that delivers 60.0 W to a rectangular area that mea-sures 2.50 m by 3.75 m.I = P/A = 60.0W

/(2.50m×3.75m)= 6.4W/m2.

EXR 5.2.04 Find the intensity (measured in W/m2) ona soccer field (measuring 73 m by 110 m) illuminated by7.50 kW of lighting.I = P/A = 7500W

/(73m×110m)= 0.934W/m2.

EXR 5.2.05 Find the intensity (measured in W/cm2) of awave that delivers 90.4 W to a rectangular area that mea-sures 7.5 cm by 11.0 cm.I = P/A = 90.4W

/(7.5cm×11.0cm)= 1.1W/cm2.

EXR 5.2.06 Find the intensity (measured in W/cm2) ofa wave that delivers 72.5 W to the surface of a door thatmeasures 1.05 m by 1.82 m.Since we are asked for the intensity to be measuredper square centimetre, we should convert distances tocentimetres first: I = P/A = 72.5W

/(105cm × 182cm) =

0.00379W/cm2 = 3.79mW/cm2.

EXR 5.2.07 Find the intensity (measured in W/cm2) of

light that delivers 15.0 W onto a piece of paper (measuring210 mm by 297 mm).Since we are asked for the intensity to be measuredper square centimetre, we should convert distances tocentimetres first: I = P/A = 15.0W

/(21.0cm× 29.7cm) =

0.0241W/cm2.

EXR 5.2.08 Find the intensity (measured in W/cm2)of a computer monitor that emits 0.500 W of light energy.(measuring 274 mm by 487 mm).Since we are asked for the intensity to be measured persquare centimetre, we should convert distances to cen-timetres first: I = P/A = 0.500W

/(27.4cm × 48.7cm) =

0.000375W/cm2 = 0.375mW/cm2.

EXR 5.2.09 Find the intensity (measured in W/m2) of awave that delivers 13.7 W to a circular area 5.00 m in di-ameter.Always remember that when given the diameter of a cir-cle we will need the radius to find the area A = π r2.I = P/A = 13.7W

/(π( 1

2 ×5.00m)2)= 0.698W/m2.

EXR 5.2.10 Find the intensity (measured in W/m2) ofa wave that delivers 88 mW to a circular area 75.2 cm indiameter.Always remember that when given the diameter of a cir-cle we will need the radius to find the area A = π r2.I = P/A = 0.088W

/(π( 1

2 ×0.752m)2)= 0.20W/m2.

EXR 5.2.11 Find the intensity (measured in W/cm2) ofa wave that delivers 0.42 W to a circular area 6.2 cm in di-ameter.Always remember that when given the diameter of a cir-cle we will need the radius to find the area A = π r2.I = P/A = 0.42W

/(π( 1

2 ×6.2cm)2)= 0.014W/cm2.

Exercises for PPT α10 November 28, 2019 114 Ch.5 Waves

Page 121: Solutions to Exercises for Physics for Physiotherapy ... - Solutions.pdf · SOLUTIONSto Exercises for Physics for Physiotherapy Technology 203-946-DW Fall 2019 by Chris Roderick,

EXR 5.2.12 Find the intensity (measured in W/cm2) ofa wave that delivers 72 W to a circular area 3.20 m in di-ameter.Always remember that when given the diameter of a circlewe will need the radius to find the area A =π r2. I = P/A =72W

/(π( 1

2 ×320cm)2)= 0.00090W/cm2 = 0.90mW/cm2.

EXR 5.2.13 Find the intensity of a wave measured5.00 m from a 4.20 W source.We are given that the surface area of a sphere is A = 4π r2.At the distance r = 5.00m from the source the wavefrontis a sphere with radius r. Thus the intensity is I = P/A =4.20W

/(4π(5.00m)2

)= 0.0134W/m2 = 13.4mW/m2.

EXR 5.2.14 Find the intensity of a wave measured

5.42 m from a 0.37 W source.I = P/A = 0.37W

/(4π(5.42m)2

) = 0.0010W/m2 =1.0mW/m2.

EXR 5.2.15 Find the intensity (measured in units ofW/cm2) of a wave measured 6.2 cm from a 17.4 W source.I = P/A = 17.4W

/(4π(6.2cm)2

)= 0.036W/cm2.

EXR 5.2.16 Find the intensity (measured in units ofW/cm2) of a wave measured 2.56 m from a 0.909 W source.I = P/A = 0.909W

/(4π(256cm)2

) = 1.10×10−6 W/cm2.Since this quantity is so small, it would be better to mea-sure it in units of micro-watts per square centimetre:I = 1.10µW/cm2

Exercises for PPT α10 November 28, 2019 115 Ch.5 Waves

Page 122: Solutions to Exercises for Physics for Physiotherapy ... - Solutions.pdf · SOLUTIONSto Exercises for Physics for Physiotherapy Technology 203-946-DW Fall 2019 by Chris Roderick,

5.2.2 Intensity, Energy, and Time

Since power relates to energy and time, we can relate intensity to the amount of energy that was transferred:

P = I × A (5.9)

∆E = I × A×∆t (5.10)

These relations can also allow us to solve for the time required to deliver a required amount of energy:

∆t =∆E/

(I × A) (5.11)

Where necessary recall that 1Cal= 4184J is the nutritional calorie, and that 1kWh= 3.6MJ is the usual unit of hydro-electrical energy (where each of these conversions are exact numbers).

EXR 5.2.17 Find the energy (measured in J) deliveredby a wave of intensity 2.11 W/m2 to a rectangular areathat measures 1.24 m by 75 cm after 51 seconds.Since the intensity is measured per square metre, the areamust be measured in square metres, and we must convertthe measurements of the area to metres. ∆E = I×A×∆t =2.11W/m2 × (1.24m×0.75m)× (51s)= 100J.

EXR 5.2.18 Find the energy (measured in J) deliveredby a wave of intensity 37.0 mW/m2 to an area of 4.00 m2

after one hour.∆E = I×A×∆t = 0.0370W/m2×4.00m2×(60×60s)= 533J.

EXR 5.2.19 Find the energy (measured in kWh) deliv-ered by a microwave oven (intensity 2.560 kW/m2) to anarea measuring 28 cm by 23 cm after 33 minutes.∆E = I×A×∆t = 2560W/m2×(0.28m×0.23m)×(33×60s)=3.26×105 J = 0.0907kWh. (This calculation converted thepower to watts so that we could do our time in seconds,but we could have kept it in kilowatts and used our timein hours to get kWh directly.)

EXR 5.2.20 Find the energy (measured in kWh) deliv-ered by a wave of intensity 13.0 W/m2 to an area of 15.0 m2

after eight hours.∆E = I × A ×∆t = 13.0W/m2 × 15.0m2 × (8 × 60 × 60s) =5.62×106 J= 1.56kWh.

EXR 5.2.21 Find the energy (measured in J) deliveredby a wave of intensity 0.64 W/cm2 to a rectangular areathat measures 23 cm by 71 cm after 5 minutes.Since we are asked for the energy to be measured mul-tiples of joules, the time must be measured in seconds:∆E = I × A×∆t = 0.64W/cm2 × (23cm×71cm)× (5×60s)=313536J= 0.314MJ.

EXR 5.2.22 Find the energy (measured in J) deliveredby a wave of intensity 0.053 W/cm2 to a rectangular areathat measures 17 cm by 20 cm after 3 minutes and 12 sec-onds.Since we are asked for the energy to be measured multi-ples of joules, the time must be measured in seconds: ∆E =I×A×∆t = 0.053W/cm2×(17cm×20cm)×(12s+3×60s)=3460J= 3.5kJ.

EXR 5.2.23 Find the energy (measured in Cal) de-livered by a wave of intensity 12.0 W/cm2 to an area of200 cm2 after seven and a half minutes.Since we are asked for the energy to be measured mul-tiples of joules, the time must be measured in seconds:∆E = I × A × ∆t = 12.0W/cm2 × 200cm2 × (7.5 × 60s) =1.08×106 J= 258Cal.

EXR 5.2.24 Find the energy (measured in Cal) deliv-ered by a wave of intensity 9.50 W/cm2 to a circular areaof diameter 18.0 cm after fifteen minutes.Since we are asked for the energy to be measured multi-ples of joules, the time must be measured in seconds: ∆E =I × A ×∆t = 9.50W/cm2 × (

π( 12 ×18.0cm)2

)× (15 × 60s) =2.18×106 J= 520Cal.

EXR 5.2.25 Find the time required (measured in min-utes and seconds) to deliver 513 kJ over a rectangular areameasuring 20.6 cm by 19.4 cm using a source with inten-sity 32 W/cm2.∆t = ∆E

/(I × A) = (513×103 J)

/(32W/cm2 × (20.6cm ×

19.4cm))= 40s.

EXR 5.2.26 Find the time required (measured in min-utes and seconds) to deliver 1.20 MJ over a square areameasuring 0.50 m on each side using a source with inten-sity 6.4 W/cm2.∆t = ∆E

/(I × A) = (1.20×106 J)

/(6.4W/cm2 × (50cm)2) =

75s= 1min and 15s.

EXR 5.2.27 Find the time required (measured in min-utes and seconds) to deliver 0.765 Cal over a rectangulararea measuring 21 cm by 23 cm using a source with inten-sity 5.23 mW/cm2.∆t =∆E

/(I × A)

= (0.765Cal×4184J/Cal)/

(0.00523W/cm2×(21cm×23cm)= 1267s= 21min and 7s.

EXR 5.2.28 Find the time required (measured inminutes and seconds) to deliver 82.0 Cal over a circulararea of diameter 15.0 cm using a source with intensity7.00 W/cm2.∆t =∆E

/(I × A)

= (82.0Cal×4184J/Cal)/

(7.00W/cm2 × (π( 1

2 ×15.0cm)2))

= 277s= 4min and 37s.

Exercises for PPT α10 November 28, 2019 116 Ch.5 Waves

Page 123: Solutions to Exercises for Physics for Physiotherapy ... - Solutions.pdf · SOLUTIONSto Exercises for Physics for Physiotherapy Technology 203-946-DW Fall 2019 by Chris Roderick,

Exercises for PPT α10 November 28, 2019 117 Ch.5 Waves

Page 124: Solutions to Exercises for Physics for Physiotherapy ... - Solutions.pdf · SOLUTIONSto Exercises for Physics for Physiotherapy Technology 203-946-DW Fall 2019 by Chris Roderick,

5.2.3 Sound Level

The quantitative measure β that models subjective loudness is sound level, defined by:

β= (10dB) log(I/

I0)

(5.12)

The logarithm returns a number, and the factor 10dB expresses the level as a multiple of the unit decibel (dB) thatmeasures level. The argument of the logarithm is the ratio of the intensity I of the sound wave to I0 the referenceintensity which is defined to be the exact quantity

I0 = 1×10−12 W/m2 (5.13)

which corresponds (roughly) to the quietest sound that an average person can hear.

In cases where the level is known and the intensity is asked for, the relation is

I = I0 ×10β/10dB (5.14)

Take care with the exponents in these calculations! Ideally you will figure out how to get your calculator to express allquantities in scientific notation.

EXR 5.2.29 Find the sound level (in decibels) of a soundthat has intensity 1×10−6 W/m2.From the definition β = (10dB) log

(I/

I0) =

(10dB) log(

1×10−6 W/m2

1×10−12 W/m2

)= (10dB) log(10+6) = (10dB) ×

(+6)= 60dB.

EXR 5.2.30 Find the sound level (in decibels) of a soundthat has intensity 7.5×10−8 W/m2.From the definition β = (10dB) log

(I/

I0) =

(10dB) log(

7.5×10−8 W/m2

1×10−12 W/m2

)= 49dB.

EXR 5.2.31 Find the intensity (measured in µW/m2) ofa sound of level 65dB.I = I0 × 10β/10dB = (1×10−12 W/m2) × 1065dB/10dB =(1×10−12 W/m2)×10+6.5 = 1×10−5.5 W/m2 = 3.16×10−6 W/m2 =3.16µW/m2.

EXR 5.2.32 Find the intensity (measured in µW/m2) ofa sound of level 53dB.I = I0 × 10β/10dB = (1×10−12 W/m2) × 1053dB/10dB =(1×10−12 W/m2)×10+5.3 = 0.20µW/m2.

PROBLEM 5.2.01: Find the time required (measured in minutes and seconds) to deliver 1.37µJ over an area of21.0cm2 using a source with a sound level of 70 dB.

From the level we get the intensity: I = I0 ×10β/10dB = (1×10−12 W/m2)×1070dB/10dB = 1×10−5 W/m2.

From the intensity and area we get power (the rate at which energy is being delivered to the area): P = I × A =(1×10−5 W/m2)× (0.00210m2)= 21.0×10−9 W.

From the power and the target energy, we find the required time: ∆t = ∆E/

P = (1.37×10−6 J)/

(21.0×10−9 W) =65.2s. Just a little more than one minute.

PROBLEM 5.2.02: Find the energy delivered (measured in microjoules) over a circular area of diameter 14.7cm usinga source with a sound level of 80 dB after two and a half minutes.

From the level we get the intensity: I = I0 ×10β/10dB = (1×10−12 W/m2)×1080dB/10dB = 1×10−4 W/m2.

From the intensity and area we get power (the rate at which energy is being delivered to the area): P = I × A =(1×10−4 W/m2)× (

π( 12 ×0.147m)2

)= 1.697×10−6 W.

From the power and the time, we find the delivered energy: ∆E = P ×∆t = (1.697×10−6 W)× (2.5×60s)= 255.µJ.

PROBLEM 5.2.03: Find the energy delivered (measured in microjoules) over a circular area of 0.60cm2 using a sourcewith a sound level of 80 dB after three minutes.

From the level we get the intensity: I = I0 ×10β/10dB = (1×10−12 W/m2)×1080dB/10dB = 1×10−4 W/m2.

From the intensity and area we get power (the rate at which energy is being delivered to the area): P = I × A =(1×10−4 W/m2)× (0.60×10−4 m2)= 0.60×10−8 W.

From the power and the time, we find the delivered energy: ∆E = P ×∆t = (0.60×10−8 W)× (3×60s)= 1.08µJ.

PROBLEM 5.2.04: Ultrasound waves can have very large intensities without posing a danger to human hearing sinceno portion of the inner ear responds to such high frequencies. If we have such a wave, of intensity 10+5 W/m2, whatwould be:

Exercises for PPT α10 November 28, 2019 118 Ch.5 Waves

Page 125: Solutions to Exercises for Physics for Physiotherapy ... - Solutions.pdf · SOLUTIONSto Exercises for Physics for Physiotherapy Technology 203-946-DW Fall 2019 by Chris Roderick,

(a) The sound level of the wave?

(b) The total energy delivered to 1cm2 after one minute?

(a) The level is β = (10dB) log(

10+5 W/m2

10−12 W/m2

)= 170dB. (If this were an audible sound wave, it would instantly destroy a

person’s eardrums.)

(b) The total energy delivered is ∆E = I × A×∆t = (10+5 W/m2) (1×10−4 m2) (60s) = 600J. (This is enough to raise thetemperature of 100 g of water by 1.4 C.)

Exercises for PPT α10 November 28, 2019 119 Ch.5 Waves

Page 126: Solutions to Exercises for Physics for Physiotherapy ... - Solutions.pdf · SOLUTIONSto Exercises for Physics for Physiotherapy Technology 203-946-DW Fall 2019 by Chris Roderick,

5.2.4 Sound Attenuation

The energy content of sound waves decreases with distance traveled due to internal friction in the medium. Thisprocess is called attenuation. Attenuation relates the distance traveled by the wave (∆x) to the change in sound level(∆β):

α=−∆β/∆x (5.15)

The constant α is the attenuation coefficient. It is a property of the medium that depends upon the frequency of thewave. In the context of sound waves traveling through air α is usually measured in units of dB/km. In the context ofultrasound traveling through water or the human body α is usually measured in units of dB/cm.

EXR 5.2.33 If the sound level of a wave has decreasedby 7.5 dB after traveling 31 cm, what is the attenuationcoefficient measured in dB/cm?We are told that the sound level has decreased by 7.5 dB.This means that ∆β = −7.5 dB. From the definitionα = −∆β/

∆x we have that α = −(−7.5 dB)/(31 cm) =0.242 dB/cm. Note that, unlike we would usually, we donot convert the distance to metres since the attenuationcoefficient is measured per centimetre.

EXR 5.2.34 The sound level of a wave traveling throughair decreases by 80 dB due to attenuation after traveling333 m. What is the attenuation coefficient measured indB/km?Note that we were asked to find the attenuation coefficientmeasured in decibels per kilometre, so we will expressthe distance in those units. We have that α = −∆β/

∆x =−(−80 dB)/(0.333 km)= 240 dB/km.

EXR 5.2.35 Ultrasound waves ( f = 4 MHz) pass-ing through muscle decrease by 13.7 dB after traveling2.71 cm. What is the attenuation coefficient measured indB/cm?We have that α = −∆β/

∆x = −(−13.7 dB)/(2.71 cm) =5.06 dB/cm.

EXR 5.2.36 If a wave is propagating through a mate-rial with an attenuation coefficient of 5.4 dB/cm, by whatamount will its sound level have decreased after propagat-ing 3.3 cm?From the definition α = −∆β/

∆x we get ∆β = −α ∆x =−(5.4 dB/cm)(3.3 cm)=−17.8 dB.

EXR 5.2.37 Ultrasonic waves of frequency 1 MHzpropagates through air with an attenuation coefficient of12.0 dB/cm. By what amount will the wave’s sound level

have decreased after propagating 2.20 mm?Being careful to convert the distance to centimetres,from the definition α = −∆β/

∆x we get ∆β = −α ∆x =−(12.0 dB/cm)(0.220 cm)=−2.64 dB.

EXR 5.2.38 Ultrasonic waves of frequency 10 MHzpropagates through water with an attenuation coefficientof 0.232 dB/cm. By what amount will the wave’s soundlevel have decreased after propagating 43.2 cm?Noting that the attenuation coefficient is given per me-tre, we must be careful to convert the distance to me-tres as well. From the definition α = −∆β/

∆x we get∆β=−α∆x =−(23.2 dB/m)(0.432 m)=−10.0 dB.

EXR 5.2.39 If a wave is propagating through a materialwith an attenuation coefficient of 8.0 dB/cm, what distancewould it have to propagate before its sound level has de-creased by 37 dB?From the definition α = −∆β/

∆x we get ∆x = −∆β/α =

−(−37 dB)/

(8.0 dB/cm)=+4.63 cm.

EXR 5.2.40 Ultrasound at 3 MHz propagatesthrough muscle tissue with an attenuation coefficient of4.15 dB/cm. What depth will it penetrate before its soundlevel has decreased by 22.0 dB?From the definition α = −∆β/

∆x we get ∆x = −∆β/α =

−(−22.0 dB)/

(4.15 dB/cm)=+5.30 cm.

EXR 5.2.41 High-frequency audible sound waves of10 kHz propagates through air with an attenuation coeffi-cient of 190 dB/km when the humidity is 40%. What dis-tance would sound of that frequency have to travel beforeits sound level had decreased by 10.0 dB due to attenua-tion alone?From the definition α = −∆β/

∆x we get ∆x = −∆β/α =

−(−10.0 dB)/

(190 dB/km)=+0.0526 km=+52.6 m.

Exercises for PPT α10 November 28, 2019 120 Ch.5 Waves

Page 127: Solutions to Exercises for Physics for Physiotherapy ... - Solutions.pdf · SOLUTIONSto Exercises for Physics for Physiotherapy Technology 203-946-DW Fall 2019 by Chris Roderick,

Problems

Calculating energy deposition from attenuation: A drop in intensity to a fraction I2/I1 = s < 1 leads to a change in levelby ∆β= (10dB)log s. Since s < 1 and log s < 0, we will find that ∆β< 0 dB.

PROBLEM 5.2.05: An ultrasound device is set to produce waves of frequency 3.00 MHz. If the attenuation of thiswave is 3.8 dB/cm as it propagates into muscle tissue, at what depth (measured in millimetres) is 50% of the energydelivered?

If 50% of the energy has been delivered, then the intensity has dropped to 50% (half) of its initial value: I2 = 12 I1. The

change in level is ∆β= (10dB)log(1/2)=−3.0103 dB.

From the definition α=−∆β/∆x the depth at which a given decrease occurs is given by ∆x =−∆β/

α. The decreasein level (the −3.0103 dB found above). This happens over a distance

∆x =−∆β/α=−(−3.0103 dB)

/(3.8 dB/cm)=+0.7922 cm= 7.92 mm.

PROBLEM 5.2.06: An ultrasound device is set to produce waves of frequency 1.00 MHz. If the attenuation of thiswave is 4.95 dB/cm as it propagates through tendon, at what depth (measured in millimetres) is 37% of the energydelivered?

If 37% of the energy has been delivered, then the intensity has dropped to 63% of its initial value: I2 = 0.63 I1. Thechange in level is ∆β= (10dB)log(0.63)=−2.007 dB.

From the definition α=−∆β/∆x the depth at which a given decrease occurs is given by ∆x =−∆β/

α. The decreasein level (the −3.0103 dB found above). This happens over a distance

∆x =−∆β/α=−(−2.007 dB)

/(4.95 dB/cm)=+0.4054 cm= 4.05 mm.

PROBLEM 5.2.07: In the human body sound waves travel at about 1540 m/s. An ultrasound device is set to producewaves of frequency 4.00 MHz.(a) What is the wavelength (in millimetres) of these waves, in the body?(b) If the attenuation of the wave is 8.2 dB/cm, at what depth (measured in millimetres) is 90% the energy delivered?(c) What is this depth, expressed as a multiple of the wavelength?

(a) From the fundamental relation for periodic waves λ= v/ f = (1540 m/s)/

(4.00×10+6 Hz)= 3.85×10−4 m= 0.385 mm.

(b) If 90% of the energy has been delivered, then the intensity has dropped to 10% (one-tenth) of its initial value:I2 = 1

10 I1. The change in level is ∆β= (10dB)log(1/10)=−10 dB.

From the definition α=−∆β/∆x the depth at which a given decrease occurs is given by ∆x =−∆β/

α. The decreasein level (the −10 dB found above). This happens over a distance

∆x =−∆β/α=−(−10 dB)

/(8.2 dB/cm)= 1.22 cm= 12.2 mm.

(c) The depth ∆x = 1.22 cm is ∆x/λ= (12.2 mm)

/(0.385 mm)= 31.7 wavelengths.

PROBLEM 5.2.08: The attenuation coefficient of skin at 2 MHz is approximately 5dB/cm. If 2 MHz ultrasound ofintensity 0.64W/cm2 is applied over an 12 cm-diameter circular area of skin that is 3 mm thick:(a) What is the intensity of the wave just below the skin?(b) What amount of energy is delivered into the portion below skin after seven minutes?

(a) Given the attenuation coefficient and the distance traveled, the change in sound level is∆β=−α∆x =−(5dB/cm)(0.3cm)=−1.5dB. From the relation ∆β= (10dB)log s, the ratio of intensities (s = I1/I2) is equal to

(10dB)log s =∆β (5.16)

s = 10∆β/

(10dB) = 10(−1.5dB)/

(10dB) = 0.708 (5.17)

Consequently the intensity below the skin is

I2/

I1 = s (5.18)

I2 = s× I1 = (0.708)(0.64W/cm2)= 0.45W/cm2 (5.19)

(b) With the intensity found in part (a), the energy delivered is

∆E = I2 × A×∆t = (0.45W/cm2)(π( 1

2 ×12cm)2)

(7×60s)= 2.1375×10+4 J= 21.4kJ (5.20)

That is enough energy to raise the temperature of one kilogram of water by 5 C.

Exercises for PPT α10 November 28, 2019 121 Ch.5 Waves

Page 128: Solutions to Exercises for Physics for Physiotherapy ... - Solutions.pdf · SOLUTIONSto Exercises for Physics for Physiotherapy Technology 203-946-DW Fall 2019 by Chris Roderick,

Chapter 6

Electricity“Electricity” is broad term for the phenomena related to the controlled transfer of electric charge. In the context of

electric circuits this allows for the controlled transfer and transformation of electric energy.

6.1 Electric Current, Voltage and Power

Electric charge changing location transfers electric energy. Interaction of these charges with their surrounding ma-terial can then transform their electric energy into other forms. With current measuring the rate of charge motion itbecomes meaningful to speak of the power transferred by current. The following exercises explore these ideas.

6.1.1 Current, Charge and Time

Electric charge is an intrinsic physical property (like mass) of the fundamental particles of matter.

Electric current measures the rate at which electric charge moves from place to place.

I = |∆Q|∆t

(6.1)

With charge measured in coulombs and time in seconds, current is measured in amperes (1 A=1 C/s). Since a coulombis a large amount of charge we will quite frequently be measuring currents in smaller increments such as milliamperesmA and microamperes µA. Notice that this quantity depends upon the magnitude of the charge, and not its sign.

Because of the relation between charge, time and current, an alternate unit for charge is the millampere-hour(1mAh= 0.001A×3600s= 3.6C). This unit is often used for measuring the capacity of small batteries, like those foundin portable electronics.

EXR 6.1.01 What is the electric current (measuredin milliamperes) when 75.0 C of charge is transferred in120 s?I = |∆Q|/∆t = (75.0C)

/(120s)= 0.625A= 625mA.

EXR 6.1.02 What is the electric current (measured inmicroamperes) when 632µC of charge is transferred in7.02 s?I = |∆Q|/∆t = (632×10−6 C)

/(7.02s)= 90.0µA.

EXR 6.1.03 What is the electric current (measured inamperes) when 37.0× 10+12 electrons are transferred in1.00s?The amount of charge is (37.0×10+12)×(−1.602×10−19 C)=−5.93µC. This is a large number of electrons, but each hasa very small charge, so the total is small.

The corresponding current is I = |∆Q|/∆t =|(−5.93µC)|/(1.00s)= 5.93µA.

EXR 6.1.04 What is the electric current (measured inamperes) when one-tenth of a mole of electrons are trans-ferred in one hour?“One mole” of things is Avogadro’s number of things NA =6.022×10+23. One-tenth of a mole of electrons is a charge

of ( 110 ×6.022×10+23)× (−1.602×10−19 C) =−9647C. The

charge of each electron is very small, but a mole is a pro-foundly large number, so the total charge is significant.

The corresponding current is I = |∆Q|/∆t =|(−9647C)|/(3600s) = 2.68A. This sort of transfer mighthappen when operating an oven to cook a meal at mediumtemperatures, for example.

EXR 6.1.05 What is the electric charge (measured incoulombs) transferred when a current of 0.500 A flows for12.0 s?|∆Q| = I ×∆t = (0.500A)× (12.0s)= 6.00C.

EXR 6.1.06 What is the electric charge (measured incoulombs) transferred when a current of 128 mA flows forseven and a half minutes?The time elapsed is ∆t = 7.5× 60s = 450s. The chargetransferred is thus |∆Q| = I ×∆t = (0.128A) × (450s) =57.6C.

EXR 6.1.07 What is the electric charge (measured inmilliamp-hours) transferred when a current of 88.8 mAflows for 20 minutes and 16 seconds?The time elapsed is ∆t = 16s+ (20× 60s) = 1216s. The

Exercises for PPT α10 November 28, 2019 122 Ch.6 Electricity

Page 129: Solutions to Exercises for Physics for Physiotherapy ... - Solutions.pdf · SOLUTIONSto Exercises for Physics for Physiotherapy Technology 203-946-DW Fall 2019 by Chris Roderick,

charge transferred is thus |∆Q| = I ×∆t = (0.0888A) ×(1216s)= 108C. This equals 108C× 1mAh

3.6C = 30mAh.

EXR 6.1.08 If a current of 210 mA flows for 7 minuteshow many moles of electrons are transferred?The time elapsed is ∆t = 7×60s= 420s. The charge trans-ferred is thus |∆Q| = I ×∆t = (0.210A)× (420s) = 88.2C.This equals 88.2C

/| −1.602×10−19 C| = 5.51×10+20 elec-trons (where we used the magnitude of the charge be-cause we are counting). This equals (5.51×10+20)

/(6.022×

10+23mol−1) = 9.14×10−4mol, a little less than one thou-sandth of a mole.

EXR 6.1.09 What amount of time (measured in minutesand seconds) must elapse for a 1.30 A current to transfer420 C?∆t = |∆Q|/I = (420C)

/(1.30A) = 323s, which is five min-

utes and twenty-three seconds.

EXR 6.1.10 What amount of time (measured in hoursand minutes) must elapse for a 0.750 A current to transfer1200 mAh?∆t = |∆Q|/I = (1200mAh)

/(750mA) = 1.60h, which is one

hour and thirty-six minutes. (Notice that our currentmust be measured in milliamperes when our charge ismeasured in millamp-hours)

EXR 6.1.11 Car batteries have capacities measured inamp-hours (not milliamp-hours). What amount of time(measured in minutes and seconds) must elapse for a833 A current to drain 70 Ah from a car battery?The amount of charge is 70Ah = 70 × (1A × 3600s) =2.52×10+5 C = 0.252MC. The time elapsed will thus be∆t = |∆Q|/I = (0.252MC)

/(833A) = 303s, which is five

minutes and three seconds.

Exercises for PPT α10 November 28, 2019 123 Ch.6 Electricity

Page 130: Solutions to Exercises for Physics for Physiotherapy ... - Solutions.pdf · SOLUTIONSto Exercises for Physics for Physiotherapy Technology 203-946-DW Fall 2019 by Chris Roderick,

6.1.2 Voltage, Charge and Energy

Similar to how there is gravitational energy between separated masses there is electrical energy between separatedcharges. The electrical potential difference, commonly referred to as the voltage difference, is defined by

∆V = ∆E∆Q

(6.2)

This is the ratio of the change in electrical energy to the amount of charge that experienced that change. (The analogousquantity in the case of gravity would be (mg∆y)

/m = g∆y, which only depends upon the interaction and its geometry,

but not the thing that is being interacted with.) The units of voltage difference are volts: 1V= 1J/C.

As an example, if −72mC charge is transferred across a +1.50V difference, the change in electrical energy is∆E =∆V ×∆Q = (+1.50V)× (−0.072C)=−0.108J. It is critical to note that, unlike the definition of current, the signsof the quantities involved are important.

EXR 6.1.12 What voltage difference was crossed by+0.700mC if its electric energy changed by +8.40mJ?∆V =∆E

/∆Q = (+8.40mJ)

/(+0.700mC)=+12.0V.

EXR 6.1.13 What voltage difference was crossed by+0.700C if its electric energy changed by −77.7mJ?Remembering that signs are important, ∆V = ∆E

/∆Q =

(−0.0777J)/

(+0.700C)=−0.111V=−111.mV.

EXR 6.1.14 What voltage difference was crossedby an electron if its electric energy changed by−0.2755×10−15 J?The charge of an electron is ∆Q = −1.602×10−19 C. Thus∆V = ∆E

/∆Q = (−0.2755×10−15 J)

/(−1.602×10−19 C) =

+1720V=+1.720kV.

EXR 6.1.15 What voltage difference was crossed by+0.833mAh if its electric energy changed by −27.0J?The charge is ∆Q = +0.833×3.6C = +3.00C. Thus ∆V =∆E

/∆Q = (−27.0J)

/(+3.00C)=−9.00V.

EXR 6.1.16 If +2.11mC crosses a voltage difference of−0.570V what is the change in the charge’s electrical en-ergy?Remembering that the signs are important: ∆E = ∆V ×∆Q = (−0.570V)× (+2.11mC)=−1.20mJ.

[By conservation of energy a decrease in the charge’selectric energy would result in an equal increase in someother forms of energy in the system; either the charge’s ki-netic energy (like if it we accelerating towards a negativelycharged object), or the thermal energy of its surroundings(if it were moving through a resistor in a circuit).]

EXR 6.1.17 If −73.5µC crosses a voltage difference of−4.44mV what is the change in the charge’s electrical en-

ergy?Remembering that the signs are important: ∆E = ∆V ×∆Q = (−4.44mV)× (−73.5µC)=+326.nJ.

EXR 6.1.18 If −25.6×10−12 C crosses a voltage differ-ence of +5.08×10+4 V what is the change in the charge’selectrical energy?∆E = ∆V × ∆Q = (+5.08×10+4 V) × (−25.6×10−12 C) =−1.30µJ.

[This situation could be something like a large sparkjumping a gap of about 1.6 cm. The decrease in thecharge’s electric energy manifests as an increase in the ki-netic energy of the charges (as they accelerate across thegap), followed by dissipation by heat, light and sound.]

EXR 6.1.19 If +295mAh crosses a voltage difference of+12.0V what is the change in the charge’s electrical en-ergy?The charge is ∆Q = +295× 3.6C = +1062C. Thus ∆E =∆V ×∆Q = (+12.0V)× (+1062C)=+12744J= 12.7kJ.

[This situation would be like the energy delivered by acar battery during starting.]

EXR 6.1.20 What amount of charge needs to cross640 mV to transfer 10.24µJ?∆Q =∆E

/∆V = (10.24µJ)

/(640mV)= 16.0µC.

EXR 6.1.21 What amount of charge (measured in mAh)needs to cross 1.500 V to transfer 15.12 kJ?the amount of charge is ∆Q = ∆E

/∆V =

(15.12kJ)/

1.500V = 10080C. Converting to milliamp-hours ∆Q = 10080C × 1mAh

3.6C = 2800mAh. [This is thecapacity of a standard AA-battery.]

Exercises for PPT α10 November 28, 2019 124 Ch.6 Electricity

Page 131: Solutions to Exercises for Physics for Physiotherapy ... - Solutions.pdf · SOLUTIONSto Exercises for Physics for Physiotherapy Technology 203-946-DW Fall 2019 by Chris Roderick,

6.1.3 Electrical Power

Charges moving across a voltage difference transfer energy. When current flows across a voltage difference, the rate ofcharge motion relates to the rate of energy transfer, which is power:

P = I ×∆V (6.3)

Consider the units of current times voltage:

A×V= Cs× J

C= J

s=W (6.4)

Also, from the relation between power and energy, we have that

∆E = P ×∆t (6.5)

= I ×∆V ×∆t (6.6)

EXERCISE 6.1.22 What power is being delivered by250 mA flowing through a 1.50 V AA-battery?P = I ×∆V = (0.250A)× (1.50V)= 0.375W.

EXERCISE 6.1.23 What power is being delivered by 62 Aflowing through a 12 V car battery?P = I ×∆V = (62A)× (12V) = 744W (which is almost onehorse-power 1hp= 746W).

EXERCISE 6.1.24 What power is being delivered by2.15 A flowing through a resistor with a 5.52 V voltage dif-ference across it?P = I ×∆V = (2.15A)× (5.52V)= 11.9W.

EXERCISE 6.1.25 Charging a 9.00 V battery by forcing0.987 A through it delivers what power?P = I ×∆V = (0.987A)× (9.00V)= 8.88W.

EXERCISE 6.1.26 What energy is delivered by 325 mAflowing across 1.50 V after 30 s?∆E = I ×∆V ×∆t = (0.325A)× (1.50V)× (30s)= 14.6J.

EXERCISE 6.1.27 What energy is delivered by 0.850 mAflowing across 33.3 mV after seventeen minutes?∆E = I×∆V ×∆t = (8.50×10−4 A)×(0.0333V)×(17×60s)=28.9mJ.

EXERCISE 6.1.28 What current must flow across 120 Vis required to deliver 1500 W?I = P

/∆V = (1500W)

/(120V)= 12.5A.

EXERCISE 6.1.29 What current must flow across 9.00 Vis required to deliver 7.20 W?I = P

/∆V = (7.20W)

/(9.00V)= 0.800A.

Exercises for PPT α10 November 28, 2019 125 Ch.6 Electricity

Page 132: Solutions to Exercises for Physics for Physiotherapy ... - Solutions.pdf · SOLUTIONSto Exercises for Physics for Physiotherapy Technology 203-946-DW Fall 2019 by Chris Roderick,

6.1.4 Problems

PROBLEM 6.1.01: A total of 720 mAh of electrons flows through a 1.500 V battery.(a) What amount of charge in coulombs (including its sign) crossed the battery?(b) If this charge gained electrical energy, what was the sign of the voltage difference?(c) What was the change in the charge’s electrical energy?(d) If the battery was providing this energy at a constant rate of 300 mW, what was the current?

(a) The charge is composed of electrons, so it is negative.Converting from milliamp-hours to coulombs: ∆Q =−720mAh=−720×3.6C=−2592C.

(b) If the charge gains energy, then ∆E > 0J. Since ∆V =∆E/∆Q and ∆Q < 0C, we must have ∆V < 0V.

(c) From the definition of voltage difference ∆E =∆V ×∆Q = (−1.500V)× (−2592C)=+3888J.

(d) There are two ways to solve this problem.

Since power, voltage and current relate as P = I∆V , we see that I = P/∆V = (0.300W)

/(1.500V)= 0.200A= 200mA.

Alternatively, Since P =∆E/∆t, we have ∆t =∆E

/P = (3888J)

/(0.300W)= 12960s= 3.60h.

Thus I = |∆Q|/∆t = (720mAh)/

(3.60h)= 200mA.

PROBLEM 6.1.02: Desk-top cup warmer:(a) How many AA batteries would you need to warm a cup of tea by 3.12 C? (Each 1.500 V battery can deliver2800mAh. The cup contains 230 mL of tea, which has a heat capacity roughly that of water.)(b) If we want to achieve this warming in five minutes, what must be the resistance of the heating element?

(a) The amount of water in the cup is m = 230mL× 1kg1L = 0.230kg. The amount of thermal energy required to warm

the cup is∆E = mC ∆T = (0.230kg)(4184 J

kg ·C )(+3.12C)=+3.00kJ

Each battery can deliver ∆E =∆Q×∆V = (2800mAh× 3.6C1mAh )× (1.500V)= 15.1kJ. One battery will do the job.

(b) Since ∆E = I ×∆V ×∆t, the required current will be

I = ∆E∆V ×∆t

= 3.00kJ1.500V× (5×60s)

= 6.67A

From Ohm’s Law the resistance must be R =∆V/

I = (1.500V)/

(6.67A)= 0.225Ω.

Exercises for PPT α10 November 28, 2019 126 Ch.6 Electricity

Page 133: Solutions to Exercises for Physics for Physiotherapy ... - Solutions.pdf · SOLUTIONSto Exercises for Physics for Physiotherapy Technology 203-946-DW Fall 2019 by Chris Roderick,

6.2 Electric Circuits

Electric charge changing location (a current) transfers electric energy. Interaction of these charges with their sur-rounding material can then transform their electric energy into other forms, and vice versa. An electric circuit is anarrangement of circuit elements (like batteries, resistors, LEDs, etc) connected to each other by wires.

6.2.1 Ohm’s Law & the Simple Circuit

Ohm’s Law relates the difference in electrical potential (the voltage difference ∆V ) across a resistor (R measured inohms) to the current (I measured in amperes) that flows through it:

∆V = IR (6.7)

The unit of resistance is the ohm: 1ohm= 1Ω= 1V/A. Since electrical power is given by P = I∆V , we also have

P = I2 R (6.8)

as the rate at which electric energy is dissipated as thermal energy by a resistor. This relation also shows that oneohm is equivalent to 1Ω= 1W/A2. (Algebra can also show that P = (∆V )2

/R.)

The simplest circuit is a battery and a resistor connected in a single loop, as shown in the diagram below:

RE

a

b c

d

The voltage difference sustained by a battery (or generator, or other source of electrical energy) is referred to as theelectro-motive force, or emf for short, denoted by the symbol E . The value of the emf does not depend upon (or varywith) what it is connected to in the circuit. What does vary, and depends upon the other elements in the circuit, is thecurrent that flows through the battery.

EXR 6.2.01 If the emf of the battery is E = 5.00V andthe resistance in the circuit is R = 220Ω, then find(a) the current flowing around the circuit,(b) the power dissipated by the resistor.

(a) Ohm’s Law tells us that I =∆V/

R. The voltage acrossthe resistor is the emf of the battery, so I = 5.00V

/220Ω=

0.0227A= 22.7mA.

(b) The power dissipated by the resistor is P = I2R =(0.0227A)2(220Ω)= 0.114W= 114mW.

EXR 6.2.02 If the emf of the battery is E = 17.4V andthe resistance in the circuit is R = 421Ω, then find(a) the current flowing around the circuit,(b) the power dissipated by the resistor.

(a) Ohm’s Law tells us that I =∆V/

R. The voltage acrossthe resistor is the emf of the battery, so I = 17.4V

/421Ω=

0.0413A= 41.3mA.

(b) The power dissipated by the resistor is P = I2R =(0.0413A)2(421Ω)= 0.719W= 719mW.

EXR 6.2.03 If the emf of the battery is E = 20.0V andthe current flowing around the circuit is I = 5.00A, thenfind

(a) the value of the resistance,(b) the power provided by the battery.

(a) Ohm’s Law tells us that R = ∆V/

I. The voltageacross the resistor is the emf of the battery, so R =20.0V

/5.00A= 4.00Ω.

(b) Since the voltage across the battery is equal to its emf(∆V = E ) the power provided by the battery is P = V I =E I = 20.0V×5.00A= 100W.

EXR 6.2.04 If the emf of the battery is E = 8.37V andthe current flowing around the circuit is I = 792mA, thenfind(a) the value of the resistance,(b) the power provided by the battery.

(a) Ohm’s Law tells us that R = ∆V/

I. The voltageacross the resistor is the emf of the battery, so R =8.37V

/0.792A= 10.6Ω.

(b) Since the voltage across the battery is equal to its emf(∆V = E ) the power provided by the battery is P = V I =E I = 8.37V×0.792A= 6.63W.

EXR 6.2.05 If the emf of the battery is E = 8.00V and itprovides P = 16.0W of power, then find

Exercises for PPT α10 November 28, 2019 127 Ch.6 Electricity

Page 134: Solutions to Exercises for Physics for Physiotherapy ... - Solutions.pdf · SOLUTIONSto Exercises for Physics for Physiotherapy Technology 203-946-DW Fall 2019 by Chris Roderick,

(a) the current around the circuit,(b) the value of the resistance.

(a) The power provided by the battery is P = E I, so thecurrent through the battery (and thus around the circuit)is I = P

/E = (16.0W)

/(8.00V)= 2.00A.

(b) Ohm’s Law tells us that R = ∆V/

I. The voltageacross the resistor is the emf of the battery, so R =8.00V

/2.00A= 4.00Ω.

EXR 6.2.06 If the emf of the power supply is E = 120Vand it provides P = 1.200kW of power, then find(a) the current around the circuit,(b) the value of the resistance.

(a) The power provided by the supply is P = E I, so thecurrent through the power supply (and thus around thecircuit) is I = P

/E = (1200W)

/(120V)= 10.0A.

(b) Ohm’s Law tells us that R = ∆V/

I. The voltageacross the resistance is the emf of the power supply, soR = 120V

/10.0A= 12.0Ω.

EXR 6.2.07 If the resistance is R = 220Ω and a currentI = 33.2mA flows around the circuit, then find(a) the emf of the battery,(b) the power dissipated by the resistor.

(a) Ohm’s Law gives us E =∆V = IR = (0.0332A)(220Ω)=7.30V.

(b) The power dissipated by a resistor is P = I2R =(0.0332A)2 (220Ω)= 0.242W.

EXR 6.2.08 If the resistance is R = 33.33Ω and a cur-rent I = 3.60A flows around the circuit, then find(a) the emf of the battery,(b) the power dissipated by the resistor.

(a) the voltage across the resistor is the emf of the bat-tery: ∆V = E . Thus Ohm’s Law gives us E = IR =(3.60A)(33.33Ω) = 120V. (This is the voltage of a wallplug.)

(b) The power dissipated by a resistor is P = I2R =(3.60A)2 (33.33Ω) = 432W. (Let’s hope this is designed asa heating element, else it is going to melt!)

Exercises for PPT α10 November 28, 2019 128 Ch.6 Electricity

Page 135: Solutions to Exercises for Physics for Physiotherapy ... - Solutions.pdf · SOLUTIONSto Exercises for Physics for Physiotherapy Technology 203-946-DW Fall 2019 by Chris Roderick,

6.2.2 Circuits with Resistors in Series

In series, the current through each element of the circuit (the battery, and each of the resistors) is the same. Whenresistors are connected in series the equivalent resistance of the resulting circuit is given by

Req = R1 +R2 (6.9)

The current that flows through the battery, and each of the resistors, is given by I = E/

Req.

R1

R2E

a

b c

d

EXERCISE 6.2.09 If the emf of the battery in the seriescircuit is E = 5.00V and the resistances are R1 = 220Ω andR2 = 150Ω, then find(a) the current flowing around the circuit,(b) the voltage difference across the small resistor,(c) the power dissipated by the smaller resistor.

(a) The equivalent resistance of this series circuit is Req =R1+R2 = 220Ω+150Ω= 370Ω. The current flowing out ofthe battery (and hence around the circuit) is found usingOhm’s Law: I = E /Req = (5.00V)

/(370Ω) = 0.0135135A =

13.5mA.

(b) In series the current through each element of the cir-cuit is the same. Ohm’s Law thus gives the voltage acrossthe smaller resistor: V2 = IR2 = (0.0135135A)(150Ω) =2.03V.

(c) The power dissipated by the smaller resistor is P2 =I2 R2 = (0.0135135A)2 (150Ω)= 0.0273923W= 27.4mW.

EXERCISE 6.2.10 If the emf of the battery in the seriescircuit is E = 16.3V and the resistances are R1 = 53.0Ωand R2 = 17.0Ω, then find(a) the current flowing around the circuit,(b) the voltage difference across the small resistor,(c) the power dissipated by the smaller resistor.

(a) The equivalent resistance of this series circuit is Req =R1+R2 = 53.0Ω+17.0Ω= 70.0Ω. The current flowing outof the battery (and hence around the circuit) is found usingOhm’s Law: I = E /Req = (16.3V)

/(70.0Ω) = 0.232857A =

233mA.

(b) In series the current through each element of the cir-cuit is the same. Ohm’s Law thus gives the voltage across

the smaller resistor: V2 = IR2 = (0.232857A)(17.0Ω) =3.96V.

(c) The power dissipated by the smaller resistor is P2 =I2 R2 = (0.232857A)2 (17.0Ω)= 0.921782W= 922mW.

EXERCISE 6.2.11 If the emf of the battery in the seriescircuit is E = 70.0V, the resistance R1 = 98.0Ω, and thecurrent flowing around the circuit is I = 345mA, then findthe value of the unknown resistor.

Given the emf and the current, the equivalent resistanceof the circuit must be Req = E /I = (70.0V)

/(0.345A) =

203Ω. Because Req = R1 + R2, we find R2 = Req − R1 =203Ω−98.0Ω= 105Ω.

EXERCISE 6.2.12 If the emf of the battery in the seriescircuit is E = 120V, the resistance R1 = 34.3Ω, and thecurrent flowing around the circuit is I = 1.75A, then findthe value of the unknown resistor.

Given the emf and the current, the equivalent resistanceof the circuit must be Req = E /I = (120V)

/(1.75A)= 68.6Ω.

Because Req = R1 +R2, we find R2 = Req −R1 = 68.6Ω−34.3Ω= 34.3Ω.

EXERCISE 6.2.13 If the emf of the battery in the seriescircuit is E = 1.50V, the resistance R1 = 1.22Ω, and thecurrent flowing around the circuit is I = 551mA, then findthe value of the unknown resistor.

Given the emf and the current, the equivalent resistanceof the circuit must be Req = E /I = (1.50V)

/(0.551A) =

2.72Ω. Because Req = R1 +R2, we find R2 = Req −R1 =2.72Ω−1.22Ω= 1.50Ω.

Exercises for PPT α10 November 28, 2019 129 Ch.6 Electricity

Page 136: Solutions to Exercises for Physics for Physiotherapy ... - Solutions.pdf · SOLUTIONSto Exercises for Physics for Physiotherapy Technology 203-946-DW Fall 2019 by Chris Roderick,

6.2.3 Circuits with Resistors in Parallel

In parallel, the voltage across each element in the circuit (the battery, and each of the resistors) is the same. Whenresistors are connected in parallel the equivalent resistance of the resulting circuit is given by

1Req

= 1R1

+ 1R2

(6.10)

Your must remember to take the reciprocal of this!: Req = (1/Req)−1. The current that flows through the battery isgiven by I = E

/Req.

R1 R2E

a

b c d

ef

EXERCISE 6.2.14 If the emf of the battery in the paral-lel circuit is E = 25.0V and the resistances are R1 = 11.0Ωand R2 = 22.0Ω, then find(a) the equivalent resistance of the circuit,(b) the current out of the battery,(c) the current through each resistor.

(a) The equivalent resistance of this parallel circuit isfound by first calculating (Req)−1 = (R1)−1 + (R2)−1 =(11.0Ω)−1 + (22.0Ω)−1 = 0.13636Ω−1. (Note carefully theunits here, warning us that we are not yet done!) Theequivalent resistance is thus Req = (0.13636Ω−1)−1 =7.33Ω. (To evaluate this, note that it is less than eitherof the individual resistances, as it should be.)

(b) The current out of the battery is I = E/

Req =(25.0V)

/(7.33Ω)= 3.41A.

(c) Remembering that the voltage across each elementis the same in parallel (V1 = V2 = E ), the currentthrough each resistor obeys Ohm’s Law: I1 = E

/R1 =

(25.0V)/

(11.0Ω) = 2.27A; I2 = E/

R2 = (25.0V)/

(22.0Ω) =1.14A.

EXERCISE 6.2.15 If the emf of the battery in the paral-lel circuit is E = 75.0V and the resistances are R1 = 150Ωand R2 = 68.0Ω, then find(a) the equivalent resistance of the circuit,(b) the current out of the battery,(c) the current through each resistor.

(a) The equivalent resistance of this parallel circuit is

found by first calculating (Req)−1 = (R1)−1 + (R2)−1 =(150Ω)−1 + (68.0Ω)−1 = 0.0213725Ω−1, then calculatingReq = (0.0213725Ω−1)−1 = 46.8Ω.

(b) The current out of the battery is I = E/

Req =(75.0V)

/(46.8Ω)= 1.60A.

(c) Remembering that the voltage across each elementis the same in parallel (V1 = V2 = E ), the currentthrough each resistor obeys Ohm’s Law: I1 = E

/R1 =

(75.0V)/

(150Ω) = 0.500A; I2 = E/

R2 = (75.0V)/

(68.0Ω) =1.10A.

EXERCISE 6.2.16 If the emf of the battery in the par-allel circuit is E = 1.50V and the resistances are R1 =0.375Ω and R2 = 0.920Ω, then find(a) the equivalent resistance of the circuit,(b) the current out of the battery,(c) the current through each resistor.

(a) The equivalent resistance of this parallel circuit isfound by first calculating (Req)−1 = (R1)−1 + (R2)−1 =(0.375Ω)−1 + (0.920Ω)−1 = 3.75362Ω−1, then calculatingReq = (3.75362Ω−1)−1 = 0.266Ω.

(b) The current out of the battery is I = E/

Req =(1.50V)

/(0.266Ω)= 5.63A.

(c) Remembering that the voltage across each ele-ment is the same in parallel (V1 = V2 = E ), the cur-rent through each resistor obeys Ohm’s Law: I1 =E

/R1 = (1.50V)

/(0.375Ω) = 4.00A; I2 = E

/R2 =

(1.50V)/

(0.920Ω)= 1.63A.

Exercises for PPT α10 November 28, 2019 130 Ch.6 Electricity

Page 137: Solutions to Exercises for Physics for Physiotherapy ... - Solutions.pdf · SOLUTIONSto Exercises for Physics for Physiotherapy Technology 203-946-DW Fall 2019 by Chris Roderick,

6.3 Time-Varying Currents

When electric current flows charges move through each element of the circuit. In the context of your Electrotherapycourse a portion of the patient’s body is an element of the circuit. In the human body it is a mixture of chargedmolecules, both positive and negative, that move when electric current flows in the body. With the boundaries of cells,organs, and ultimately the skin, electric current flowing in the human body has nowhere to go. Thus a current thatflows at a constant rate through the human body will separate the charged molecules – with the positively chargedmolecules migrating to one side and the negatively charged molecules migrating to the other.

One method to avoid separating the different charges, while still permitting current, is alternate the direction ofcurrent flow. (Think of this like when you rub your hands together to produce heat by friction, and you alternate thedirection of your hand’s motion.)

Another method is to periodically decrease the applied voltage to zero. This gives the partially separated chargestime to move back together and recombine into its original neutral mixture. That process is called relaxation.

Although the current may vary with time, all the relations between current, voltage and resistance still apply,since they apply at each instant in time. The same is true of the relation between current, voltage and power. So theserelations are true at each instant in time:

∆V = I ×R (6.11)

P = I ×∆V (6.12)

What is complicated now are the relations between current and charge transported, and between power and energytransferred or transformed. These totals will now depend upon exactly how the current varies with time, and will oftenbe expressed as averages over intervals of time that are large in comparison to the changes.

6.3.1 Alternating Current

One of the fundamental types of time-varying current is referred to as Alternating Current, which is abbreviated“AC”. The current (and voltage) vary sinusoidally. (This is the type of current provided by electrical wall outlets.) Ifthe current varies as the sine-function, then (as an example) the instantaneous power being dissipated by a resistor(P = RI2) will vary as the square of the sine-function. This is graphed below:

t(s)

I(A)

t(s)

P(W)

The rate of energy dissipation does not depend upon the direction of charge motion, only its rate. So even as thecurrent reverses direction – in the graph, where the current takes negative values over half of each cycle – the poweris same as in the first half of the cycle. Calling the amplitude of the current oscillation Ipeak, the maximum value ofthe power is Ppeak = R (Ipeak)2.

Averaged over many cycles the average power being dissipated is half of the peak value:

Pavg = 12 Ppeak (6.13)

This is defined by the fact that the total energy ∆E delivered by the alternating current will be ∆E = Pavg∆t, where ∆tis the total time that the current is oscillating. (This can be proved rigorously using calculus.)

This relation between the peak and average power is used to define a measure of the “average current”. (Strictlyspeaking, since the current alternates direction, the current averages out to zero. The “average” being defined here ismore like a measure of the amplitude of the oscillating current.) Defining Irms the root mean square (RMS) average ofthe current through Pavg = R (Irms)2, we obtain

Pavg = 12 Ppeak (6.14)

R (Irms)2 = R (Ipeak)2 (6.15)

Irms = 1p2

Ipeak (6.16)

Exercises for PPT α10 November 28, 2019 131 Ch.6 Electricity

Page 138: Solutions to Exercises for Physics for Physiotherapy ... - Solutions.pdf · SOLUTIONSto Exercises for Physics for Physiotherapy Technology 203-946-DW Fall 2019 by Chris Roderick,

Since 1p2≈ 0.7 the RMS (average) value of the current is about 70% of the peak value. It is important to have these

two measures of the alternating current because: (1) the average of the power relates to the target energy we seek todeliver; but (2) the peak current is important to control for safety reasons.

Through Ohm’s Law the RMS (average) value of the oscillating voltage difference that drives the oscillating currentis similarly defined in relation to its peak value:

∆Vrms = 1p2∆Vpeak (6.17)

(If you are aware of it, the “120 V” of the electrical supply from wall plugs is the RMS value of the oscillating voltage.)When oscillating voltages and currents are measured using “multimeters” it is usually the RMS values being reported.These relations hold:

∆V = I ×R (6.18)

∆Vpeak = Ipeak ×R (6.19)

∆Vrms = Irms ×R (6.20)

This first is true at each instant in time. The second relates the values at the instant the current is at its peak. Thethird relates the values when averaged over an interval of time that is long in comparison to the period of oscillation.

Frequency Dependence

The expressions for Ohm’s Law (∆V = I R) and the instantaneous power (P = I∆V ) do not depend upon the frequency ofthe current’s alternation. What does vary with frequency, usually, is the resistance of the material, typically increasingwith frequency – although the truth is quite complicated, and there are many exceptions. In practice, the frequencywill usually be prescribed and the resistance under those conditions either known, or easily determined.

The resistance may vary with frequency, but the peak voltages and currents are independent of the frequency. Justremember that, as we saw for waves, amplitude and frequency are independent of each other.

Exercises for PPT α10 November 28, 2019 132 Ch.6 Electricity

Page 139: Solutions to Exercises for Physics for Physiotherapy ... - Solutions.pdf · SOLUTIONSto Exercises for Physics for Physiotherapy Technology 203-946-DW Fall 2019 by Chris Roderick,

EXR 6.3.01 If an alternating current Irms = 13.0mAflows through a 220Ω resistor, find(a) the RMS voltage being applied,(b) the peak current flowing through the resistor, and(c) the average power being dissipated.

(a) Ohm’s Law tells us that ∆Vrms = Irms × R. Thus∆Vrms = (0.0130A)(220Ω)= 2.9V.

(b) The relation between peak and RMS current is Irms =1p2

Ipeak. The RMS value is about 70% of the peak, so

the peak value should be bigger. Thus Ipeak =p2× Irms =p

2×13.0mA= 18.4mA.

(c) The average power relates to the RMS current byPavg = R (Irms)2 = (220Ω)(0.0130A)2 = 37.2mW.

EXR 6.3.02 If an alternating current Irms = 15.0A flowsthrough a 8.00Ω resistor, find(a) the RMS voltage being applied,(b) the peak current flowing through the resistor, and(c) the average power being dissipated.

(a) Ohm’s Law tells us that ∆Vrms = Irms × R. Thus∆Vrms = (15.0A)(8.00Ω) = 120V. (This is plug voltage. Isthis an appliance?)

(b) The relation between peak and RMS current is Irms =1p2

Ipeak. The RMS value is about 70% of the peak, so

the peak value should be bigger. Thus Ipeak =p2× Irms =p

2×15.0A= 21.2A. (Definitely not a biologically safe cur-rent.)

(c) The average power relates to the RMS current byPavg = R (Irms)2 = (8.00Ω)(15.0A)2 = 1.80kW. (Maybe anoven? Or a heater of some kind?)

EXR 6.3.03 If an alternating voltage difference ∆Vrms =120V is applied across a 68.0Ω resistor, find(a) the RMS current flowing through the resistor,(b) the peak current flowing through the resistor, and(c) the average power being dissipated.

(a) Ohm’s Law tells us that Irms =∆Vrms/

R. Thus Irms =(120V)

/(68.0Ω)= 1.76A.

(b) The relation between peak and RMS current is Irms =1p2

Ipeak. The RMS value is about 70% of the peak, so

the peak value should be bigger. Thus Ipeak =p2× Irms =p

2×1.76A= 2.49A.

(c) The average power relates to the RMS voltage byPavg = (∆Vrms)2

/R = (120V)2

/(68.0Ω)= 212W.

EXR 6.3.04 If an alternating voltage difference ∆Vrms =9.00V is applied across a 725mΩ resistor, find

(a) the RMS current flowing through the resistor,(b) the peak current flowing through the resistor, and(c) the average power being dissipated.

(a) Ohm’s Law tells us that Irms =∆Vrms/

R. Thus Irms =(9.00V)

/(0.725Ω)= 12.4A.

(b) The relation between peak and RMS current is Irms =1p2

Ipeak. The RMS value is about 70% of the peak, so

the peak value should be bigger. Thus Ipeak =p2× Irms =p

2×12.4A= 17.5A.

(c) The average power relates to the RMS voltage byPavg = (∆Vrms)2

/R = (9.00V)2

/(0.725Ω)= 112W.

EXR 6.3.05 If an alternating voltage difference ∆Vrms =75.0V dissipates 210W (average) in a resistor, find(a) the RMS current flowing through the resistor, and(b) the value of the resistance.

(a) The average power defines Pavg = (Irms)2×R. By Ohm’sLaw (∆Vrms = Irms×R) this means that Pavg = Irms×∆Vrms.Thus Irms = Pavg

/∆Vrms = (210W)

/(75.0V)= 2.80A.

(b) By Ohm’s Law R = ∆Vrms/

Irms = (75.0V)/

(2.80A) =26.8Ω.

EXR 6.3.06 If an alternating voltage difference ∆Vrms =120V dissipates 15.3W (average) in a resistor, find(a) the RMS current flowing through the resistor, and(b) the value of the resistance.

(a) The average power defines Pavg = (Irms)2×R. By Ohm’sLaw (∆Vrms = Irms×R) this means that Pavg = Irms×∆Vrms.Thus Irms = Pavg

/∆Vrms = (15.3W)

/(120V)= 128mA.

(b) By Ohm’s Law R = ∆Vrms/

Irms = (120V)/

(0.128A) =938Ω.

EXR 6.3.07 If an alternating voltage difference ∆Vrms =120V is applied across a 22.2Ω resistor, find the time re-quired to dissipate 7.31 kJ.

The average power relates to the energy delivered andtime taken by ∆E = Pavg ×∆t, so that ∆t = ∆E

/Pavg. The

average power is Pavg = (∆Vrms)2/

R = (120V)2/

(22.2Ω) =649W. Thus ∆t =∆E

/Pavg = (7310J)

/(649W)= 11.3s.

EXR 6.3.08 If an alternating voltage difference ∆Vrms =5.00V is applied across a 417mΩ resistor, find the time re-quired to dissipate 29.2 kJ.

The average power relates to the energy deliveredand time taken by ∆E = Pavg × ∆t, so that ∆t =∆E

/Pavg. The average power is Pavg = (∆Vrms)2

/R =

(5.00V)2/

(0.417Ω) = 60.0W. Thus ∆t = ∆E/

Pavg =(29200J)

/(60.0W)= 487s≈ 8minutes.

Exercises for PPT α10 November 28, 2019 133 Ch.6 Electricity